SlideShare una empresa de Scribd logo
MARAVILLOSOS
PROBLEMAS DE
MATEMÁTICAS
Libro 11
http://matemelga.wordpress.com/
El poema “No hay uno que valga” contiene 1000 versos.
El autor los ha numerado por orden, absteniéndose de usar los
números cuya escritura decimal use uno o más dígitos 1.
El primer verso es numerado, así, como 2, el segundo 3, ... de 9 vamos
a 20, ..., de 99 a 200 ...
¿Cuál es el número del último verso?
SOLUCIÓN
Los números válidos para usar son:
De 1 a 99 hay que eliminar los que empiezan por 1 y, de los restantes, los que acaban en 1: el 1, los que
indican decenas son 10 (10 a 19) y los restantes son 8 ( 21 a 91). En total, 80810199 =−−−
De 100 a 199 se eliminan todos pues todos tienen una cifra 1 al menos.
De 200 a 299 hay uno más que de 1 a 99 (se cuenta el 200 que no tiene correspondiente en la lista
anterior) : 81180 =+
En cada centena, hasta 999, hay también 81, por lo que de 1 a 999: 72881880 =×+ (de 1 a 99 y ocho
centenas más).
De 1000 a 1999 tienen todos, al menos, una cifra 1, por lo que no cuentan y cada centena, a partir de
2000 , y quitando la centena de 2100 a 2199 , tienen todas 81 números válidos.
Por tanto, usando tres centenas, de 1 a 2399 hay 971813728 =×+ a los que deberemos añadir la primera
decena válida 2400 a 2409 (todos menos 9:2401 ), la segunda se ignora porque tiene cifra 1 en todos sus
números, y con la tercera y la cuarta tenemos que de 1 a 2439 hay 9982797193971 =+=×+ números.
Para llegar a mil faltan dos: el que hace 999, 2440 , y el que hace 1000,
2442
Halla el cociente entre el área del triángulo equilátero grande PQR y el área del
triángulo equilátero pequeño LMN ?
SOLUCIÓN
Llamamos x al lado del triángulo equilátero pequeño. Como 651 =+ es la longitud
del lado del triángulo equilátero grande, la proporción entre las áreas de ambos es
la misma que entre los cuadrados de las longitudes de los lados: 2
2
6
x
Hallamos la longitud del lado del triángulo pequeño: aplicamos el teorema del
coseno en el triángulo LRN y tenemos que
2121526
2
1
10251º60cos51251 222
==−=×−+=×××−+= xx
La proporción pedida es ==
21
366
2
2
x
12/7
Marta tiene un medallón hecho con 11 hexágonos.
Ha hecho un dibujo del medallón y se divierte rodeándolo con otros
hexágonos que marca con números desde 1, girando en la dirección
opuesta a la de las agujas del reloj.
Se da cuenta que los números 1, 20, 45, 76,… se colocan uno encima del
otro y se pregunta, ¿cuál será el número que ocupe el lugar 100 en esa
serie?
Averígualo.
SOLUCIÓN
Consideramos la sucesión { } { }...,76,45,20,1=na y la sucesión { } { } { } Nnaab nnn ∈∀=−= + ,...,31,25,191
Esta última sucesión es una progresión aritmética de diferencia 6 : Nnnbn ∈∀+= ,613
Como Nnbaaaab nnnnnn ∈∀+=−= ++ ,11 . Podemos escribir entonces
+++++++++=+++++










+=
+=
+=
+=
=
−−
−−
113322114321
11
334
223
112
1
.........1.........
.........
1
nnn
nnn
babababaaaaaa
baa
baa
baa
baa
a
( ) ( ) ( )( ) ( ) 
−×−×++
+=
−×+
+=+++++= −
−
2
1161319
1
2
1
1.........1 11
1321
nn
a
nbb
abbbba n
n
nnn
( ) ( ) Nnnna
nnnn
a nn ∈∀−+=
−+
+=
−×+
+= ,12103
2
26206
1
2
1626
1 2
2
Entonces, =−×+×= 12100101003 2
100a
30988
La sucesión a1 , a2 , a3 , ... verifica que a1 = 19, a2018 = 99 y para n ≥ 3, an es la media
aritmética de los n – 1 primeros términos.
¿Cuál es el valor de a2 ?
SOLUCIÓN
191 =a y 992018 =a . Sea xa =2
Según indica el enunciado,
2
19
2
21
3
xaa
a
+
=
+
= ;
2
19
3
2
19
19
3
321
4
x
x
x
aaa
a
+
=
+
++
=
++
=
Según se observa, podemos suponer que todos los demás términos de la serie serán iguales. Demostrémoslo
por inducción: Suponemos que niNi
x
ai ≤≤∈∀
+
= 3,,
2
19
.
Entonces, 
+
+
+
++
+
++
=
+++++
= −
+
n
xxx
x
n
aaaaa
a nn
n
2
19
2
19
...
2
19
19
... 1321
1
( ) ( ) ( ) ( ) ( )
2
19
2
19
2
1921922
19
219
11
x
a
n
xn
n
xnx
n
x
nx
a nn
+
=
+×
=
+×−++×
=
+
×−++
= ++
Hemos demostrado que todos los términos de la serie, a partir del tercero, valen
=−===+=
+
=
+
191981981999
2
19
2
19
22018 xax
x
a
x
179
Al laberinto adjunto se entra por la [E]ntrada y se sale por la [S]alida.
Cuando se llega a una casilla
• se puede subir si el número es múltiplo de 3
• se puede bajar si el número es múltiplo de 4
• se puede avanzar hacia la derecha si el número es múltiplo de 5
Da un camino que permita ir desde la [E]ntrada hasta la [S]alida.
SOLUCIÓN
El camino adecuado es
→20→9↑35→12↓16↓24↓25→15→36↑21↑30→
Calcula el valor de
SOLUCIÓN
Si
b
a
m
abaabmb
a
bb
mm
a
log
1
log
1
loglog
1
===== , por lo que
( ) ( ) ( ) ( ) =++++=++++ 100log...4log3log2log
)!100(log
1
...
)!100(log
1
)!100(log
1
)!100(log
1
!100!100!100!100
100432
( )( ) ( )( )==××××= !100log100...432log !100!100
1
Juan tiene el doble de la edad de su hija Carmen, y estamos en el año 2018.
En 2029, el mismo día que hoy, tendrán entre los dos un siglo de años.
¿Qué edades tienen ambos ahora mismo?
SOLUCIÓN
Sea x la edad actual de Juan e y la edad actual de Carmen. Teniendo en cuenta que habrán pasado
1120182029 =− años en la segunda afirmación, el enunciado dice que 



=+++
=
1001111
2
yx
yx
52262
26
3
78
2
783
2
782
2
7822100
2
=×=




==
=




=
=




=+
=




=−=+
=
 x
y
yx
y
yx
yy
yx
yx
yx
Actualmente, Juan tiene 52 años y Carmen tiene 26 años
Con centro en el vértice B del cuadrado ABCD se traza un arco de circunferencia
de radio igual a la longitud del lado del cuadrado.
Un punto P de dicho arco dista 8 cm del lado AD y 1 cm del lado DC.
¿Cuál es la longitud, en centímetros, del lado del cuadrado?
SOLUCIÓN
Sea x la longitud, en centímetros, del lado del cuadrado.
Construímos el triángulo rectángulo BPC' y pueden verse, en la figura, las
longitudes de sus lados en función de x
Aplicando el teorema de Piotágoras a dicho triángulo tenemos que
( ) ( ) 0651812641618 2222222
=+−=+−++−=−+− xxxxxxxxxx
De lo anterior,



=
=

±
=
×−±
=
5
13
2
818
2
6541818 2
x
x
x
Desechando, por el contexto del problema, el segundo valor, se deduce que el lado del cuadrado mide
13 centímetros
Angelines, Rocío y Tere viven en la misma comarca.
En línea recta, las casas de Angelines y Rocío están separadas por un
número entero de hectómetros y las casas de Angelines y Tere en un
número entero de kilómetros.
Además, siempre en línea recta, hay exactamente diez kilómetros
más entre la casa de Rocío y la de Tere que entre la casa de Rocío y la
de Angelines.
Finalmente, el triángulo formado por las casas de las tres vecinas es un triángulo rectángulo.
¿Cuál es, al menos, la distancia entre la casa de Angelines y la de Tere?
SOLUCIÓN
Llamamos a a la distancia, en hectómetros, entre las
casas de Angelines y de Rocío, y llamamos b a la
distancia, en kilómetros, entre las casas de Angelines y de
Tere, por lo que en hectómetros será b10
Tanto a como b son valores enteros y se pide el menor
valor posible de b
Pasada a hectómetros, la distancia entre las casas de
Rocío y Tere será a+100
En resumen, lo que sabemos es que la distancia entre
Rocío y Angelines no es la mayor de las tres.
Como las casas forman un triángulo rectángulo, la hipotenusa puede ser cualquiera de los lados que tienen a
la casa de Tere como vértice.
Vemos los dos casos posibles aplicando el teorema de Pitágoras:
1. Hipotenusa Rocío-Tere: ( ) ( ) +=+++=+ 222222
1002001000010100 baaabaa
1002100210020010000 22
+=+==+ ababba entero positivo.
El menor valor posible, siendo también a un valor entero positivo, es 12100222 =+×=b
2. Hipotenusa Angelines-Tere: ( ) ( ) ++=++= 10000200210010010 22222
aabaab
1002
50100
100002002 2
2
2
2
++=
++
= a
a
b
aa
b , debiendo ser a y b valores enteros positivos.
Una simple comprobación para valores probables iniciales permite deducir que 121442
>> bb , si
existe, pues a debería ser múltiplo de 10
En conclusión, la menor distancia posible entre las casas de Angelines y Tere es
12 kilómetros
El rectángulo adjunto está compuesto por dos cuadrados y un
rectángulo pequeño.
Si el rectángulo pequeño es semejante al rectángulo original y el
lado de cada cuadrado mide 1 cm, ¿qué longitud tiene el lado
mayor del rectángulo grande?
SOLUCIÓN
Sea x la longitud, en centímetros, del rectángulo pequeño no
igual a la unidad. La longitud que se busca es xx +=++ 211
Como los dos rectángulos son semejantes se verifica que
( ) 01212
1
1
2 2
=−+=×+=
+
xxxx
x
x
Entonces,
2
222
2
82
2
442 ±−
=
±−
=
+±−
=x . Desechamos, según el contexto, la solución negativa de
la ecuación por lo que 12
2
222
−=
+−
= xx y el lado pedido es =−+=+ 1222 x
1 + √2 cm
Calcula el valor de la integral
SOLUCIÓN
Sea
( )
( ) ( ) −+−
−
=
8
4
.
3ln9ln
9ln
dx
xx
x
I y hacemos el cambio de variable 6+= tx
Entonces, dtdx = y los límites son 264600 −=−=−= xt y 268611 =−=−= xt por lo que
( )
( ) ( )
( )
( ) ( )
]1[.
3ln3ln
3ln
.
36ln69ln
69ln 2
2
2
2  −− ++−
−
=
−++−−
−−
= dt
tt
t
dt
tt
t
I
Hacemos ahora el cambio dudtut −=−= y los límites son 200 =−= tu y 211 −=−= tu por lo que
( )
( ) ( )
( )
( ) ( )
( ) ( )
( ) ( ) −
−
− −++
+
=−
−++
+
=
++−
−
=
2
2
2
2
2
2
]2[.
3ln3ln
3ln
.
3ln3ln
3ln
.
3ln3ln
3ln
du
uu
u
du
uu
u
dt
tt
t
I
Como el nombre de la variable es arbitrario podemos designar, sin perder generalidad, las variables t y u
como y en las igualdades ]1[ y ]2[ :
( )
( ) ( )
( )
( ) ( )
( )
( ) ( )
( )
( ) ( )

−++
+
+
++−
−
=+






−++
+
=
++−
−
=



−−
+
−
− 2
2
2
2
ª2ª1
2
2
2
2
.
3ln3ln
3ln
.
3ln3ln
3ln
.
3ln3ln
3ln
]2[
.
3ln3ln
3ln
]1[
dy
yy
y
dy
yy
y
II
dy
yy
y
I
dy
yy
y
I
( ) ( )
( ) ( )
] ( ) ==−−===
−++
++−
= −−−  IIydydy
yy
yy
I 4222.
3ln3ln
3ln3ln
2
2
2
2
2
2
2
2
Las flotas de la Federación Intergaláctica siempre se componen
de naves dispuestas en formación de cuadrados perfectos
cuando atacan.
El general Black Vader une las flotas A y B para formar una
flota C con la que ataca al sistema planetario Nobelius y sufre
una fuerte derrota. Solo la última fila sobrevive.
Con las naves restantes puede formar una flota D, pero prefiere subdividirla en dos flotas E y F.
E y F van al ataque y, de nuevo, solo las últimas filas de E y F escapan de la matanza. Quedan 23 naves.
¿Cuál fue el número inicial de naves de cada flota A y B de Black Vader?
SOLUCIÓN
Interpretamos las condiciones que nos plantea el problema:
Si e y f es la respectiva cantidad de naves por fila de las flotas E y F , la cantidad de naves de ambas
flotas es, respectivamente, 2
e y 2
f , siendo 222
fed += el número de naves de la flota D . Los tres valores
anteriores deben formar una terna pitagórica con 823 ==+ efe y 15=f , suponiendo que fe < (en
caso contrario sería igual) 17= d
El cuadrado de este último número, 2891722
=== dc , es el número de naves por fila de la flota C : la
cantidad de naves de la flota C es ( ) 835212892222
=== dc
Para finalizar, si las flotas A y B pueden formar la flota C , 2222
28983521===+ cba , siendo
( ) ( ) ( ) 13617828917171715178 2222
=×==×=×+× a y 2551715 =×=b (otra terna pitagórica
derivada de la anterior) el número de naves respectivas, por fila, de las flotas A y B .
La cantidad de naves de cada una de estas últimas flotas es 1849613622
==a y 6502525522
==b
18496 y 65025 naves
Otro resultado posible es 2592116122
==a y 5760024022
==b , pues también verifican que
2222
28983521===+ cba . La cantidad de naves de las flotas A y B puede ser
25921 y 57600 naves
Los puntos A, B y C de la figura dividen a los lados del triángulo MNP en
dos partes que están en la relación 1:3
¿Qué fracción de la superficie del triángulo MNP está coloreada de rojo?
SOLUCIÓN
Según el enunciado, CPCM ×= 3 , BNBP ×= 3 y
3
3
AN
AMAMAN =×=
Llamamos WÁreaCBP = y VÁreaABN =
Si consideramos los triángulos MBC y CBP y la altura común c , la
relación entre sus áreas es
=×=
×
×=
××
=
×
= WÁrea
cCPcCPcCM
Área CBPMBC 33
2
3
2
3
2
WWWÁreaÁreaÁrea CBPMBCMBP 43 =+=+=
Y considerando los triángulos MBA y ABN con altura común a , la relación entre sus áreas es
=×=
×
×=
×
=
×
=
33
1
23
1
2
3
2
V
Área
aAN
a
AN
aAM
Área ABNMBA
3
4
3
V
V
V
ÁreaÁreaÁrea ABNMBAMBN =+=+=
Por último, los triángulos MBP y MBN , con altura común b , tienen sus áreas relacionadas así:
×=
×
×=
××
=
×
= MBNMBP Área
bBNbBNbBP
Área 3
2
3
2
3
2
VW
V
ÁreaWÁrea MBNMBP =×=×==
3
4
334
Entonces, la superficie roja es
3
10
3
3
3
3
WW
W
V
WÁreaÁreaÁrea MBAMBCr =+=+=+= , y la superficie total
del triángulo es
3
16
3
4
4
3
4
4
WW
W
V
WÁreaÁreaÁrea MBNMBPt =+=+=+=
La razón es ==
16
10
3
16
3
10
W
W
5/8
Dos hermanos, Ángel y Benito, heredaron dos campos cuadrados cada
uno, y un lado de cada uno de los cuatro campos forma la orilla del lago
cuadrilátero Raum, que tiene dos ángulos opuestos rectos, precisamente
en donde los hermanos construyeron sus casas.
Por supuesto, ambas superficies heredadas miden lo mismo y las
dimensiones de cada campo es, en decámetros, un número entero.
Un día Benito sale de su casa corriendo a velocidad siempre constante,
entre 15 y 20 km/h, y alcanza el muelle en treinta segundos. Pasa por la
casa de Ángel un minuto y medio después de comenzar a correr y da la
vuelta completa al lago en cuatro minutos.
¿Cuál es la superficie del lago en áreas?
SOLUCIÓN
Llamamos x a la velocidad constante de Benito en su recorrido, medida en
km/h: 2015 ≤≤ x , y dcba ,,, a las longitudes, en decámetros, de los
campos cuadrados según se ve en la figura adjunta.
Según el enunciado, al tener las mismas superficies las herencias se verifica
que 2222
cbda +=+ , lo cual es coherente con la construcción de dos
triángulos rectángulos en el lago con la misma hipotenusa, y la superficie del
lago es
222
bcad
S
bcad
S
+
=+= dam2
La velocidad, en decámetros por minuto, es
3
5
60
100 xx
= por lo que, como en
medio minuto Benito alcanza el muelle ,
6
5
2
1
3
5 x
a
x
a =×= dam [A]
En minuto y medio alcanza llega a casa de Ángel , luego
2
5
2
3
3
5 x
ba
x
ba =+×=+ dam. De lo anterior,
3
5
6
10
6
5
2
5
2
5
6
5 xxxx
b
x
b
x
ba ==−==+=+ dam [B]
Por fin, como en cuatro minutos da la vuelta completa,
3
20
4
3
5 x
dcba
x
dcba =+++×=+++ dam,
luego
6
25
2
5
3
20
3
20
2
5 x
dc
xx
dc
x
dc
x
=+−=+=++ dam
Además, 





−





=−+





=+





+=+
22
222
2
2
2
2222
6
5
3
5
3
5
6
5 xx
cdc
x
d
x
cbda
( ) ( ) ( )
212
25
6
25
12
25
12
25
36
25
9
25 22222
22 x
cd
x
cd
xx
cdcd
xxx
cd =−=−×=−×+=−=−
En resumen,





=
=













+×=






−×=






=−
=+
6
14
6
11
26
25
2
1
26
25
2
1
2
6
25
x
d
x
c
xx
d
xx
c
x
cd
x
dc
, [C] y [D]
Según lo obtenido en [A], [B], [C] y [D], la superficie del lago es 
×+×
=
+
=
2
6
11
3
5
6
14
6
5
2
xxxx
bcad
S
2
5
72
180
2
36
110
36
70
22
22
xx
xx
S ==
+
= dam2
Como
6
5x
a = y
6
11x
c = deben ser valores enteros, siendo 2015 ≤≤ x 18= x , por lo que la superficie
del lago es 810
2
185 2
=
×
=S dam2
= S
810 áreas
En el dibujo aparece un cuarto de circunferencia de radio 1 y dos
semicircunferencias tangentes.
¿Cuál es el radio de la semicircunferencia pequeña?
SOLUCIÓN
Según lo señalado en la imagen adjunta, siendo r el valor pedido, se verifica que
( ) 



−=
=+





−=
++=+++






=+






+=





++
ar
rara
ar
rrrara
ar
rra
12
2
12
4
1
4
1
2
12
2
1
2
1 2222
22
2
( )
( ) 
+−
=
−
×−=




−
=
×−=−=

2
231
2
1
21
2
1
212 2
2
2
aaa
aaa
r
raarra
3
1
0312312 22
==−+−= aaaaa
Y como ===−=−=
6
2
3
2
3
1
112 rar
1/3 unidades
Hace un año, las edades de Rosa y de su madre Luisa tenían las
mismas cifras pero con el orden invertido cada una respecto de
la otra.
Este año sucede lo mismo con las edades de Rosa y de su padre
Juan y, curiosamente, la suma de las edades actuales de sus
padres es 93.
¿Cuál es la edad actual de Rosa?
SOLUCIÓN
Llamamos ab a la edad de Rosa. La edad de su padre es, entonces, ba y es lógico pensar que 9<a , 1>b y
ab >
Además hace un año la edad de Rosa era ( )1−ba y la edad de su madre ab )1( − por lo que este año su
madre tiene ( )1)1( +− ab años.
Así, ( ) ( ) =+=−+=++++−×=++− 10220293922093101110931)1( baabababbaab
baba 10515110 −==+ y, como ambas son cifras, 1510515 =×−== ab
Por lo tanto, Juan tiene 51 años, Luisa tiene 42 años y Rosa tiene
15 años
¿Cuáles de las siguientes desigualdades no tienen soluciones reales?
SOLUCIÓN
La respuesta se encuentra en la representación gráfica de las funciones xy 2= , 2
xy = e x
y 2= :
Pueden apreciarse cinco zonas en las que se
establece el conjunto de valores que cumplen
determinadas desigualdades:
Zona A. Se cumple la desigualdad 1:
2
22 xx x
<< , ( )7665,0, −∞−∈∀x
Zona B. Se cumple la desigualdad 6 :
x
xx 22 2
<< , ( )0,7665,0−∈∀x
Zona C. Se cumple la desigualdad 2 :
x
xx 222
<< , ( )1,0∈∀x
Zona D. Se cumple la desigualdad 4 :
xx x
222
<< , ( )2,1∈∀x
Zona E. Se cumple la desigualdad 1:
2
22 xx x
<< , ( )∞+∈∀ ,2x
En resumen,
las desigualdades 3 y 5 no tienen soluciones reales
Cuatro antiguos piratas, ahora convertidos en comerciantes
pacíficos y honorables, se encuentran y conversan.
− Cada uno de nosotros tiene un hijo, dice el primero,
¡y todos sueñan con ir al mar!
− Sí, pero los hijos de ustedes tres son exactamente
de la misma edad, a diferencia del mío, expone el
segundo.
− Es cierto, y la suma de las edades de los cuatro suman ya 50 años, observa el tercero.
− ¡Cómo pasa el tiempo!, hace diez años la misma suma era únicamente 10 años, manifiesta el
cuarto.
¿Cuál es la edad actual del hijo del segundo pirata?
SOLUCIÓN
Sean dcba ,,, las edades de los hijos de los cuatro piratas en el orden mostrado en el enunciado, que
tienen, evidentemente, más de 10 años.
La conversación se traduce en las siguientes igualdades:
( ) ( ) ( ) ( )





>>
−
=




>>
=+






>>
=+++
==






=−+−+−+−
=+++
==
10,10
3
50
10,10
503
10,10
50
1010101010
50
ba
b
a
ba
ba
ba
aaba
dca
dcba
dcba
dca








=
−
==
=
−
==
=
−
==

11
3
50
,17
12
3
50
,14
13
3
50
,11
b
ab
b
ab
b
ab
, posibilidades admisibles.
El hijo del segundo pirata puede tener, actualmente,
11, 14 o 17 años
El círculo y el rectángulo de la figura tienen el mismo centro.
Si las dimensiones del rectángulo son 6x12 cm y los dos lados pequeños del
rectángulo son tangentes al círculo, ¿cuál es el área de la región común (en
amarillo) al rectángulo y al círculo?
SOLUCIÓN
Evidentemente, el radio del círculo mide 6 cm. Marcamos con letras algunos
vértices para determinar superficies.
El área pedida será el doble de la suma de las áreas del triángulo AOD (de
marrón) y el sector circular AOB (de azul), pues es igual a la superficie restante
(de amarillo).
En el triángulo rectángulo OEA aplicamos el teorema de Pitágoras para obtener
EA : ===−=−=−= 33272793636 22222
EAOEOAEA
362 == EAAD cm
El área del triángulo AOD es, entonces, 39
2
336
2
=
×
=
×OEAD
cm2
[i]
Como el triángulo AOB es equilátero, el ángulo que abarca en el centro del círculo es de º60 , sexta parte
del ángulo completo, por lo que el área del sector circular AOB es la sexta parte del área del círculo:
π
ππ
6
6
6
6
22
==
r
cm2
[ii]
La superficie resultante de sumar las áreas del triángulo AOD y del sector circular AOB es [i]+[ii]:
π639 + cm2
y el área pedida es el doble.
18√3 + 12π cm2
= 68,876 cm2
Carlos ha dibujado, con puntos, la silueta de un barco en un papel con trama triangular
como se ve en la figura adjunta.
¿Cuántos triángulos equiláteros distintos pueden dibujarse con todos los vértices en
puntos de la silueta?
SOLUCIÓN
Los triángulos equiláteros, clasificados según tipos, son
por lo que el total es =+++++++ 244413416
38 triángulos equiláteros
En un triángulo rectángulo la bisectriz de un ángulo agudo corta al cateto
opuesto en dos trozos de longitudes 1 y 2.
¿Cuál es la longitud del segmento de bisectriz interior al triángulo?
SOLUCIÓN
Nombramos puntos, ángulos y longitudes, y debemos hallar x
Es evidente que, en el triángulo BAC ,
b
3
2tan =α y, en el triángulo DAC ,
b
1
tan =α
Entonces, =−
−
=






−
×
=
−
×
= 2
3
3
1
1
2
3
1
1
1
2
3
tan1
tan2
2tan 2
2
22
b
bb
b
bα
α
α
33233 222
===− bbbb
Aplicando el teorema de Pitágoras en el triángulo rectángulo DAC , ( ) ==+=+=+= xbx 413131
2222
2
Se construyen un octógono regular y un hexágono también regular,
aunque se han tenido que utilizar lados ligeramente más grandes que los
del octógono para que, al superponerlos, coincidan los dos vértices como
muestra la figura.
¿Cuál es la amplitud del ángulo x ?
SOLUCIÓN
Designamos α y β a los ángulos interiores respectivos de un hexágono
regular y de un octógono regular y dibujamos diagonales en ambos
polígonos para construir cuadriláteros, cuyos ángulos suman siempre º360
Según el cuadrilátero de la derecha, =+++ º360
223
º360 αα
α
º120º240º120º3602º3602º120 ==−==+ ααα
Según el cuadrilátero de la izquierda, =+++ º360
224
º360 ββ
β
º135º270º90º3602º3602º90 ==−==+ βββ
Entonces, en el cuadrilátero central, −==++=+++ º255º3602º360º135º1202º360 xxxx βα
== xx º1052
52,5o
En el siguiente criptograma cada dígito ha sido reemplazado por una letra. Como siempre a dígitos
distintos corresponden letras distintas y, además, ningún número comienza con cero.
Halla la diferencia entre el valor más pequeño posible y el mayor valor posible de HUIT.
SOLUCIÓN
Para el valor más pequeño posible serán 1=D y 2=H . Además, está claro que 9=U o
0=U , y elegimos el segundo por ser el valor más pequeño. Así, tenemos la suma que
aparece a la derecha.
Como se ve, debe ser 10=+ SE ( 46 + o 37 + ) y 5<X . Si 63 == TX y SE, no
podrían tener valores admisibles == 84 TX
53784 =+=+== ISETX (para dar el menor valor). En resumen, la suma
es la que se ve a la derecha y el menor valor posible de HUIT es 2058
El valor más grande será para 8=D y 9=H . 9=U o 0=U , y elegimos el segundo por
ser tener asignado ya el otro valor. Así, tenemos la suma que aparece a la derecha.
Debe ser 10=+ SE ( 46 + o 37 + ), 5<X y T par. El mayor valor posible 7=I hace
que 146 =+=+ XSE y 2=T . En resumen, la suma es la que se ve a la derecha y el
mayor valor posible de HUIT es 9072
De todo lo anterior deducimos que la diferencia entre los valores menor y mayor de HUIT
es =− 20589072
7014
¿Cuántos números hay que colocar en el recinto de color negro?
SOLUCIÓN
Mostrando la distribución en la imagen derecha se observa que la
cantidad de números a colocar en el recinto negro es
5
En Spanystán las monedas se identifican fácilmente si uno no es
daltónico: 13 monedas lilas valen 17 euros, 13 monedas amarillas
son 18 euros y 13 monedas rojas valen 19 euros.
Juan sale a la calle con 13 monedas de cada tipo para hacer unas
compras.
Compra un libro de 16 euros y luego una camiseta por 15 euros.
Por cada uno de estos dos gastos, Juan da varias monedas y no se
le devuelve ningún cambio. En otra tienda compra una cartera con
2 monedas amarillas y 5 monedas rojas.
Para volver a casa coge un taxi, cuyo recorrido le cuesta un número entero de euros. Juan paga con todas
las monedas amarillas y rojas que tiene y el taxista le da el cambio en monedas lilas.
¿Cuántas monedas lilas le devuelve el taxista?
SOLUCIÓN
Al comprar el libro paga con x monedas lilas, y monedas amarillas y z monedas rojas, por lo que,
convirtiéndolas en euros, 
−−
==++=++
17
1918208
20819181716
13
19
13
18
13
17 zy
xzyx
zyx





=
=
=

−−
+−−=
1
2
9
17
24
12
z
y
x
zy
zyx , al ser todos los valores, evidentemente, enteros positivos.
Le quedan entonces 4913 =− monedas lilas, 11213 =− monedas amarillas y 12113 =− monedas rojas.
Luego compra una camiseta con x monedas lilas, y monedas amarillas y z monedas rojas, por lo que,
convirtiéndolas en euros, 
−−
==++=++
17
1918195
19519181715
13
19
13
18
13
17 zy
xzyx
zyx
17
28
11
zy
zyx
−−
+−−=
En este caso hay dos posibilidades (en los demás casos rebasa el número de monedas lilas que le quedan):
a)





=
=
=
0
8
3
z
y
x
, y le quedan 134 =− moneda lila, 3811 =− monedas amarillas y 12 monedas rojas.
b)





=
=
=
1
6
4
z
y
x
, y le quedan 044 =− monedas lilas, 5611 =− monedas amarillas y 11112 =− monedas
rojas.
Por fin, al comprar la cartera le quedan a) 1 moneda lila, 123 =− moneda amarilla y 7512 =− monedas
rojas o b) 325 =− monedas amarillas y 6511 =− monedas rojas
Analizamos los dos casos al coger el autobús. Sea k el precio, en euros, del billete de autobús y m el
número de monedas lilas que le devuelven (ambos valores enteros positivos):
a) Al pagar con todas las monedas amarillas y rojas que posee, 
×
+=
×
+
×
13
17
13
719
13
118 m
k
92
13
48
11
13
17151
171313318 ==
−
+−=
−
=+=+ km
m
mk
m
kmk
b) Al pagar con todas las monedas amarillas y rojas que posee, 
×
+=
×
+
×
13
17
13
619
13
318 m
k
102
13
48
12
13
17168
171311454 ==
−
+−=
−
=+=+ km
m
mk
m
kmk
En ambos casos, en los que el precio del taxi es 9 o 10 euros, el taxista le devuelve
2 monedas lilas
Si el área del cuadrado BESO es 16 cm2
y el área del triángulo SOL es el doble, ¿cuál es, en cm2
, el
área del trapecio naranja?
SOLUCIÓN
Si la superficie del cuadrado BESO es 16 cm2
, esu lado es 416 ==a cm2
Como la superficie del triángulo SOL es 32 cm2
,
( ) ( ) =
+×
=
+×
32
2
44
32
2
haha
( ) 121643242 ==+=+× hhh cm
Establecemos ahora una relación de semejanza entre las alturas y las bases de los
triángulos SOL y EBL : 3
16
124
4412
12
=
×
==
+
=
+
x
x
a
x
ah
h
cm
De lo anterior, la superficie del trapecio naranja es =×
+
=×
+
4
2
43
2
a
ax
14 cm2
Un aspersor de riego, giratorio, está ubicado en el centro de un
jardín rectangular cuyos lados son números enteros de metros
estrictamente mayores de 2.
Este aspersor, en todo momento, rocía una superficie cuadrada de
la cual es una de las esquinas, siendo el lado del cuadrado regado
más grande que la mitad de la diagonal del jardín.
La porción máxima de jardín, regada en un momento dado (de
verde en el dibujo), tiene un área de 1991 m².
¿Cuál es la superficie del jardín?
SOLUCIÓN
Tomamos x2 y y2 como las medidas del jardín en metros, siendo
yx ≤<1
La porción de jardín máxima regada se produce en el pentágono que se
muestra en la imagen derecha, compuesto por dos trapecios iguales cuyas
bases menor y mayor miden xy − e y metros y la altura x metros.
La superficie es
( ) ( ) 1811119912
2
2 ×==×−=
×+−
×= xxy
xyxy
S m2
22211 == xx m y 1921118118121812 =+=+==− xyxy m
Entonces, la superficie del jardín es =×=× 1922222 yx
4224 m2
El 25% de los libros de Merche son novelas y 1/9 del total son libros de poesía.
Si el número de libros que tiene está entre 50 y 100, ¿cuántos de ellos no son ni novelas ni
de poesía?
SOLUCIÓN
El %25 representa la cuarta parte, por lo que el número de libros de Merche, x , es divisible por 4
Además,
9
1
de los libros son de poesía, por lo que el número de libros también es divisible por 9
En resumen, el número de libros de Merche se puede esxpresar así: nxnx 3694 =××= , siendo Nn∈
Como 72236;21003650 =×==≤=≤ xnnx
Tiene, por tanto, 18
4
72
= novelas y 8
9
72
= libros de poesía.
El resto es ( )=+− 81872
46 libros
La serie
se construye así: se eligen y escriben las cifras 8 y 7 y, después, escribimos las cifras de su producto, 56.
Después se escriben sucesivamente las cifras del producto de 7 (segundo término) y 5 (tercer término), 35,
las cifras del producto de 5 (tercer término) y 6 (cuarto término), 30, … continuando con las cifras (o única
cifra) de los productos determinados por dos términos consecutivos.
Después de un tiempo, solo se obtienen ceros.
¿Cuál es el último dígito distinto de cero?
SOLUCIÓN
La secuencia completa es ...8103536578
Desde el primero cero que aparece hasta el siguiente mostramos la secuencia que aparece y las respectivas
que se deducen, cada una de la anterior:  ...0555880......05151810...
 ...042230......06480......00101010061420......0525204460...
 ...022610......021280......062180......023420......08460...
 ...02140......06220......061210......08260......042160...
...080......0240.. 
El último dígito distinto de cero es
8
El lado del cuadrado ABCD mide 4 cm.
Si el triángulo EAD tiene igual área que el cuadrado, ¿cuál es la
distancia del vértice E a la recta determinada por B y C ?
SOLUCIÓN
Llamamos EFd = a la longitud pedida.
Está claro que 4+= hd , siendo h la altura del triángulo EAD
Como las áreas de triángulo y cuadrado son iguales, = 2
4
2
4h
842 =×= h cm
Por lo tanto, =+=+= 484hd
12 cm
Sea ABCD un cuadrilátero tal que el ángulo en C mide 76o
y el ángulo en D mide
128o
.
Se trazan las bisectrices interiores de los ángulos en A y en B que se cortan en P.
Halla el ángulo APB.
SOLUCIÓN
Dibujamos las bisectrices, el punto P y nombramos los ángulos correspondientes.
Se pide γ=BPA ˆ
La suma de los ángulos de un pentágono es º540º1803 =× pues siempre se puede
descomponer en tres triángulos.
Entonces, en el pentágono APBCD , se verifica que
( ) º24º128º76º360º540º540º128º76º360 −=−−−=−+=+++−+ γβαβγα
En el triángulo APB , γβαγβα −=+=++ º180º180 y sustituyendo en la igualdad anterior se obtiene
que ===+=−=−−−=−+
2
º204
º204º24º1802º24º180º24 γγγγγβα
102o
Con los dígitos 1, 2, 3, 4 y 5, escritos en algún orden, formamos el número
de cinco cifras abcde.
Si el número de tres cifras abc es divisible por 4, el bcd es divisible por 5 y
el cde es divisible por 3, ¿qué cifra representa la letra a ?
SOLUCIÓN
Si bcd es divisible por 55 = d
Entonces, si ec5 es divisible por 133 +=+ nec , y c es par porque abc es divisible por 4
2≠c porque, en caso contrario, dec === 52 para que 713 =+=+ nec , lo cual es una contradicción.
Por lo tanto, 34 == ec para que 713 =+=+ nec y 2=b pues 4ab es divisible por 4
De todo lo anterior, como ===== aedcb 3,5,4,2
1
Sea ABCD un rectángulo y A', B', C' y D' en las prolongaciones de sus
lados tales que AA' = k.AD ; BB' = k.AB ; CC' = k.BC ; DD' = k.CD
Halla k de modo que el área del cuadrilátero A'B'C'D' sea 25 veces el
área del rectángulo ABCD
SOLUCIÓN
Representamos en la figura las longitudes.
Entonces,
×=×+×+= ABCDCDCABAABCDDCBA ÁreaÁreaÁreaÁreaÁrea 2522 ''''''''
( ) ( ) =
+×
×+
+×
×+ ab
kaakbkbbka
ab 25
2
2
2
2
012242224 2222
=−+=+=+++ kkkkababkkababkkab
Entonces, 3
2
71
2
71
2
12411
=
+−
=
±−
=
×+±−
= kk en el contexto del problema: un valor negativo no
tiene sentido.
Por lo tanto, =k
3
En el triángulo ABC las medianas trazadas desde B y desde C son
perpendiculares entre sí.
Si AC mide 15 cm y AB mide 10 cm, calcula cuánto mide BC
SOLUCIÓN
Nombramos, en la figura, los puntos clave y las longitudes. Debemos
calcular la longitud BCx =
En los triángulos equiláteros que se construyen alrededor del punto en
donde se cortan las medianas applicamos el teorema de Pitágoras:
En BOC , 222
xba =+ [i]
En FOC ,
2
22
2
15






=+ da [ii]
En BOE , 222
5=+ cb [iii]
En FOE , 222
EFdc =+ , pero los triángulos AEF y ABC son semejantes (un ángulo igual y los lados que
lo forman son proporcionales en proporción 1 a 2 ) por lo que
2
22
22






=+=
x
dc
x
EF [iv]
Haciendo ahora [i]-[ii]-[iii]+[iv] tenemos ( ) ( ) 





+−





−=+++−+−+
2
2
2
222222222
2
5
2
15 x
xdccbdaba
====+=+=





+−





− xxx
x
x
x
x 65
5
325
325525
4
225
4
0
2
5
2
15 22
2
2
2
2
2
2
√65 cm = 8,06 cm
Se divide un cuadrado de 125 cm2
de área en cinco regiones, cuatro cuadrados y un
polígono en forma de L, de rojo en la figura, todas de igual área.
¿Cuál es la longitud, en cm, del lado más corto del polígono en forma de L?
SOLUCIÓN
Llamamos x a la longitud pedida, en cm, y a a la longitud del lado de los cuadrados,
tambien en cm.
La superficie de cada región es 25
5
125
= cm2
, por lo que 5252
== aa cm
Por otro lado, la región en forma de L , de la misma superficie, se compone de dos
rectángulos, por lo que su área será ( ) =+=+=×++× 2520422 22
xxaxxaxxax
1251025101002520 22
±−=+±−==−+ xxx
En el contexto del problema x debe ser positivo, por lo que =−= 10125x
5√5 – 10 cm = 1,18 cm
Halla la suma de los ángulos de los vértices A, B, C, D y E de la estrella de la figura.
SOLUCIÓN
Sean edcba ,,,, los ángulos interiores del pentágono central. Está claro que
º540º1803 =×=++++ edcba , suma de los ángulos de un pentágono cualquiera.
Observemos que
• en el triángulo de vértice A , dcAdcA +=+=−+−+ º180ˆº180º180º180ˆ
• en el triángulo de vértice B , edBedB +=+=−+−+ º180ˆº180º180º180ˆ
• en el triángulo de vértice C , aeCaeC +=+=−+−+ º180ˆº180º180º180ˆ
• en el triángulo de vértice D , baDbaD +=+=−+−+ º180ˆº180º180º180ˆ
• en el triángulo de vértice E , cbEcbE +=+=−+−+ º180ˆº180º180º180ˆ
Sumando ashora miembro a miembro las cinco igualdades,
( ) =×=+++++++++×=×+++++ º1080º5402º900ˆˆˆˆˆ2º1805ˆˆˆˆˆ EDCBAedcbaEDCBA
=−=++++ º900º1080ˆˆˆˆˆ EDCBA
180o
Escribe en cada casilla de la pirámide un número natural mayor que 1
de modo que
• la casilla superior tenga escrito el número 560105280
• el número escrito en cada casilla sea igual al producto de los
números escritos en las dos casillas sobre las que está apoyada.
SOLUCIÓN
Como 466
7532560105280 ×××= , vamos a intentar rellenarla de abajo hasta arriba con los divisores más
elementales.
Ponemos 5 en una esquina, al ser el factor que menos debe intervenir y 3 en la casilla central. Como hay
que repetir uno de los factores elegimos el 2 para que ocupe un ala completa (dos casillas seguidas).
Así, rellenamos de abajo hacia arriba la pirámide y queda
Que no determina, en la casilla superior, el número indicado, pero que da la pista fundamental para obtener
una pirámide (no la úníca) que se busca.
Colocamos un 2
2 en vez de un 2 en la primjera casilla de abajo y obtenemos
que sí cumple las condiciones del problema.
Una solución es, pues,
Todos los estudiantes de una clase hicieron una prueba. Cinco de ellos
obtuvieron la puntuación máxima, 100 puntos, ninguno obtuvo menos
de 60 puntos y la media de la clase fue de 76 puntos.
¿Cuántos estudiantes, como poco, había en la clase?
SOLUCIÓN
Sea n el número de estudiantes que no obtuvieron 100 puntos. El total de estudiantes pedido es 5+n
Si el promedio de puntos de esos n estudiantes fue de ( )60>x , la media total fue =
+
+×
76
5
1005
n
nx
( )
x
nnxnnx
−
==×−+=+
76
120
1207638076500
A menor valor de ( )60>x corresponde menor valor de n , siempre que ambos valores sean enteros. Esto
corresponde a 8
15
120
6176
120
6061 ==
−
=>= nx , que corresponde a =+=+ 585n
13 estudiantes
Quique intercambia los dígitos de un número de 3 cifras distintas, y distintas
de cero, de modo que ningún dígito queda en su posición original.
Después resta el número viejo menos el nuevo y el resultado es un número de
2 cifras que es un cuadrado perfecto.
Halla todos los resultados que puede obtener Quique.
SOLUCIÓN
Sea el número abc con las tres cifras distintas entre sí. Se pueden dar dos casos:
1. Volteamos el número obteniendo bca y restamos: 2
1010010100 Aacbcbabcaabc =−−−++=− ,
siendo ba > y 2
A un número entero de dos cifras.
De lo anterior, ( ) 3691011999099 222
===−−×=−−
•
AAAcbacba o 812
=A
a) Si ( ) 41110410113610119362
−=+=−−=−−×= acbcbacbaA y se cumple para
los valores
• 8,118422102 ===−=+= cbcba
• 9,229433103 ===−=+= cbcba
b) Si ( ) 91110910118110119812
−=+=−−=−−×= acbcbacbaA y se cumple para los
valores
• 3,113922102 ===−=+= cbcba
• 4,224933103 ===−=+= cbcba
• 5,335944104 ===−=+= cbcba
• 6,446955105 ===−=+= cbcba
• 7,557966106 ===−=+= cbcba
• 8,668977107 ===−=+= cbcba
• 9,779988108 ===−=+= cbcba
2. Volteamos el número obteniendo cab y restamos: 2
1010010100 Abaccbacababc =−−−++=− ,
siendo ca > y 2
A un número entero de dos cifras.
De lo anterior, ( ) 3691110999990 222
===−+×=−+
•
AAAcbacba o 812
=A
a) Si ( ) 41110411103611109362
+=+=−+=−+×= cbacbacbaA y se cumple para
los valores
• 1,881477107 ===+=+= babac
• 2,992488108 ===+=+= babac
b) Si ( ) 91110911108111109812
+=+=−+=−+×= cbacbacbaA y se cumple para los
valores
• 1,331922102 ===+=+= babac
• 2,442933103 ===+=+= babac
• 3,553944104 ===+=+= babac
• 4,664955105 ===+=+= babac
• 5,775966106 ===+=+= babac
• 6,886977107 ===+=+= babac
• 7,997988108 ===+=+= babac
Entonces, todos los resultados que puede obtener Quique son:
218 – 182 = 36; 329 – 293 = 36; 817 – 781 = 36;
928 – 892 = 36; 213 – 132 = 81; 324 – 243 = 81;
435 – 354 = 81; 546 – 465 = 81; 657 – 576 = 81;
768 – 687 = 81; 879 – 798 = 81; 312 – 231 = 81;
423 – 342 = 81; 534 – 453 = 81; 645 – 564 = 81;
756 – 675 = 81; 867 – 786 = 81; 978 – 897 = 81
18 resultados
Los puntos A, B, C y D, en este orden, determinan un cuadrilátero inscrito en una
circunferencia.
Los lados AB y CD son paralelos, el ángulo ADC = 50o
y los ángulos BAC y BCA son
iguales.
¿Cuánto mide el ángulo DAC ?
SOLUCIÓN
Como AB y CD son paralelos, el cuadrilátero inscrito en la circunferencia debe
ser un trapecio isósceles. Por lo tanto, === º50ˆˆº50ˆ CDABCDCDA
º130
2
º50º50º360ˆˆ =
−−
=== αDABABC
Como == βACBCAB ˆˆ el triángulo ABC es isósceles y −= αβ º1802
º25
2
º50
2
º130º180
2
º180
==
−
=
−
= β
α
β
Ahora bien, =−=−==+ º25º130βαγαγβ
105o
La familia Álvarez, la familia Benítez y el matrimonio Cáceres
almorzaron juntos.
Los Álvarez, que comieron 3 chuletones, 2 ensaladas y 5 gaseosas,
gastaron 85 euros, los Benítez, que comieron 5 chuletones, 3 ensaladas
y 9 gaseosas, gastaron 142 euros.
¿Cuánto gastaron los Cáceres que comieron, entre los dos, 1 chuletón,
1 ensalada y 1 gaseosa?
SOLUCIÓN
Llamamos zyx ,, a los precios unitarios respectivos, en euros, de chuletón, ensalada y gaseosa.
Según el enunciado, ( ) ( ) =++−=++−++



=++
=++ −×
zyxzyxzyx
zyx
zyx
1421709351046
142935
85523 ª2ª12
28 euros
Si Presen se sube en una mesa y Roque se queda en el suelo, Presen es 80 cm más alta
que Roque.
Si Roque se sube a la mesa y Presen se queda en el suelo entonces Roque es un metro
más alto que Presen.
¿Cuál es, en centímetros, la altura de la mesa?
SOLUCIÓN
Tomamos yx, como las alturas respectivas, en centímetros, de Roque y Presen y z la altura, en
centímetros, de la mesa.
Según el enunciado tenemos que ( ) 1020220
100
80 ª1ª2
=−=−×+−=−



+=+
+=+ −
yxyxxyyx
yzx
xzy
De la primera condición, =+=+−=+=+ 80108080 yxzxzy
90 centímetros
Halla el menor número natural que es suma de 9 naturales consecutivos, es suma de
10 naturales consecutivos y además es suma de 11 naturales consecutivos.
SOLUCIÓN
Según el enunciado, el número buscado es =
×
=
×
=
×
, siendo , , ∈ N
De lo anterior, = 9 + 36 = 10 + 45 = 11 + 55
Entonces, !"
!" ""
# ⇒ %
%
# ⇒ %
# ⇒
&'()*
*
(
*
&'(+&'
&&
,
Como , , ∈ N ⇒ -
% .
#⇒ -
.
# ⇒
-
&&/)&'
&'
0, siendo 1, 2 ∈ N
Por lo tanto,
.
= 91 ⇒ 112 + 10 = 901 ⇒ 2 =
-%
⇒ 2 = 81 +
-%
El valor natural más pequeño de 1 que hace a 2 natural es 1 = 5 ⇒ 2 = 8 × 5 +
×"%
= 40
De ahí, el valor más pequeño de ⇒ = 91 = 9 × 5 = 45 ⇒ = 10 + 45 = 10 × 45 + 45 =
495
En la figura se observa un triángulo isósceles dividido en cuatro regiones
de las que conocemos las áreas de los tres triángulos: 3, 3 y 6 cm2
.
Si M y N son los puntos medios de los lados iguales, ¿cuál es el área del
cuadrilátero verde?
SOLUCIÓN
Señalámos los vértices del triángulo grande y su altura.
Por semejanza, la altura del triángulo ABC es el doble de la del triángulo
BMC y, como ambos triángulos tienen la misma base, el área del
triángulo ABC es el doble de la del área del triángulo BMC :
( ) 186322 =+×=×= BMCABC ÁreaÁrea unidades cuadradas
Entonces, la superficie del cuadrilátero verde es la superficie del triángulo ABC menos las superficies de los
tres triángulos interiores: =−−− 63318
6 unidades cuadradas
En un hotel de Menorca hay 120 personas distribuidas entre la recepción, el
bar, el comedor y el salón de reuniones. La cantidad de personas que hay en el
bar es un quinto de la que hay en el comedor, y en la recepción hay un octavo
de las que hay en el salón.
Al pasar diez personas del comedor al salón y seis del bar a la recepción, en la
recepción hay un sexto de las que quedan en el comedor.
¿Cuántas personas había inicialmente en cada uno de los lugares
mencionados del hotel?
SOLUCIÓN
Llamamos dcba ,,, a la cantidad de personas que hay, respectivamente, en la recepción, en el bar, en el
comedor y en el salón de reuniones.
Según el enunciado, 120=+++ dcba ,
5
c
b = ,
8
d
a =
Al moverse las personas como indica el enunciado,
6
46
6
6
10
6
10
6
−
=−
−
=
−
=+
cc
a
c
a
Además,
3
1844
6
46
88
−
=
−
==
c
d
cdd
a y, sustituyendo en la primera ecuación, se tiene que
=
−+++−
=
−
+++
−
=+++ 120
30
1840403062305
120
3
1844
56
46
120
ccccc
c
cc
dcba
70
81
5670
5670813600207081 ====− cccc
De lo anterior, 4
6
4670
6
46
=
−
=
−
= a
c
a ; 14
5
70
5
=== b
c
b ; 32
3
184704
3
1844
=
−×
=
−
= d
c
d
En resumen hay
4 personas en la recepción
14 personas en el bar
70 personas en el comedor
32 personas en el salón de reuniones
“¿Qué edad tienes?” le preguntan a Joaquín, y contesta: “Si yo viviera 100 años, mi edad
actual sería los cuatro tercios de la mitad de lo que me quedaría por vivir”.
¿Cuál es la edad de Joaquín?
SOLUCIÓN
Sea x la edad actual de Joaquín. Si viviera 100 años le quedarían por vivir x−100 años.
Entonces, según lo que contesta Joaquín, ( ) −=
−
=−××= xx
x
xxx 22003
3
2200
100
2
1
3
4
===
5
200
2005 xx
40 años
Halla todos los números naturales de dos cifras tales que al elevarlos al cubo se obtienen
números que terminan en dos cifras iguales.
SOLUCIÓN
Sea xy uno de esos números. Como ( ) ( ) 322333
30300100010 yxyyxxyxxy +++=+= , los sumandos que
determinan las dos últimas cifras son 32
30 yxy +
Analizamos los casos:
xyxyy ∀=+= ,0300 32
cifra, por lo que todos los números de dos cifras acabados en cero cumplen la
condición: 90,80,70,60,50,40,30,20,10
130301 32
+=+= xyxyy debe acabar en 711 = x para que 211130 =+x . Sólo hay un número
cumpliendo la condición: 71
8120302 32
+=+= xyxyy debe acabar en



=+=
=+=

10888120;9
4888120;4
88
xx
xx
. Hay dos números cumpliendo
la condición: 92,42
27270303 32
+=+= xyxyy debe acabar en 137727270;577 =+= xx . Hay un número cumpliendo
la condición: 53
64480304 32
+=+= xyxyy debe acabar en



=+=
=+=

294464480;6
54464480;1
44
xx
xx
. Hay dos números
cumpliendo la condición: 64,14
125750305 32
+=+= xyxyy debe acabar en 55: imposible
2161080306 32
+=+= xyxyy debe acabar en 66 : imposible
3431470307 32
+=+= xyxyy debe acabar en 106333431470;733 =+= xx . Hay un número
cumpliendo la condición: 77
5121920308 32
+=+= xyxyy debe acabar en 22 : imposible
7292430309 32
+=+= xyxyy debe acabar en 225997292430;999 =+= xx . Hay un número
cumpliendo la condición: 99
Concluyendo, hay 17 números cumpliendo las condiciones del problema:
10, 14, 20, 30, 40, 42, 50, 53, 60, 64, 70, 71, 77, 80, 90, 92, 99
En la figura adjunta los segmentos AB, FC y ED son paralelos.
Si la longitud del segmento AB es 10 y la del ED es 7, ¿cuál es la longitud del
segmento FC ?
SOLUCIÓN
Como los triángulos AED y AFC son semejantes,
7
7 FCAE
AF
AF
AE
FCAF
AE
FC
ED ×
===
Como los triángulos ABE y FCE son semejantes,
10
10 FCAE
FE
FE
AE
FCFE
AE
FC
AB ×
===
Sumando las dos expresiones,
AE
FCAEAE
FCAEFCAE
FEAF
÷






+××=
×
+
×
=+
10
1
7
1
107
×=
÷
70
17
1 FC
AE
FC = 70/17
Para hacer un castillo de naipes de 1 piso se usan 2 naipes, para hacerlo de 2 pisos se usan 7 naipes, para
hacerlo de 3 pisos se usan 15 naipes…
¿Cuántos naipes hay que usar para hacer un castillo de 100 pisos?
SOLUCIÓN
El problema propone una sucesión na tal que 21 =a , 72 =a , 153 =a , … donde na es la cantidad de naipes
necesarios para completar un castillo de n pisos.
Experimentalmente puede verse que 264 =a , 405 =a , …, siempre un incremento de 3 naipes en la
diferencia con el castillo anterior.
Construímos la sucesión :1 nnn aab −= +
( ) 13213
............
243142640
233111526
2238715
213527
11
454
343
232
121
−=+−×=−=








+×==−=−=
+×==−=−=
+×==−=−=
+×==−=−=
−− nnaab
aab
aab
aab
aab
nnn
Se puede observar que esta última sucesión es una progresión aritmética de diferencia 3 y, sumando sus
1−n primeros términos, tenemos que
( ) ( ) ( ) ( )
( ) ( ) ( ) ( ) ( ) ( ) 




−×+
=
−×−+
=
−×+
=++++
−=−=−++−+−+−=++++
−
−
−−
2
143
2
1135
2
1
......
2............
11
1321
113423121321
nnnnnbb
bbbb
aaaaaaaaaaabbbb
n
n
nnnnn
( ) ( ) ( ) ( )
2
3
2
443
2
2
143
2
143
2
22
nn
a
nnnn
a
nn
a nnn
+
=
+−+
=+
−×+
=
−×+
=−
Por lo tanto, el número de naipes necesarios para construir un castillo de 100 pisos es
=
+×
=
2
1001003 2
100a
15050 naipes
Sabiendo que
calcula el valor de
SOLUCIÓN
1 + + + + … = 2 ⇒ + + + … = 2 − 1= 1
Buscamos el valor de = + 2 + 3 + 4 + … = + + + + 2 + + 3 + … ⇒
⇒ = + + + + … + + 2 + 3 + … = 1 + × + 2 + 3 + … = 1 + ⇒
⇒ − = 1 ⇒ 1 − × = 1 ⇒ =
Como 1 + + + + … es la suma de los términos de una progresión geométrica de razón ∋ | | < 1,
pues de lo contrario la suma no sería un valor real, 1 + + + + … = = = 2 ⇒ = =
2
Calcula, en la figura adjunta, el área de la forma de color rojo.
SOLUCIÓN
Para referenciar todas las formas que aparecen en las operaciones, señalamos los 12
vértices con letras consecutivas del alfabeto.
Calculamos, en principio, la superficie de toda la forma señalada
de amarillo. Dicha superficie es igual a la superficie del sector
circular , de radio 12, menos la superficie del sector circular
, de radio 8, y menos la superficie del cuadrado , de
lado 4: =
×
−
×
− 4 = 20 − 16
La superficie de la forma azul es igual a la superficie del cuadrado , de lado 4,
menos la superficie del sector circular , de radio 4: !" = 4 −
×
= 16 − 4
Hallamos ahora la superficie señalada de verde. Es igual a a la superficie del cuadrado
# , de lado 8, menos la superficie del sector circular # , de radio 8:
$% &% = 8 −
× 8
4
= 64 − 16
La superficie buscada es '( = + !" − $% &% = *20 − 16+ + *16 − 4 + − *64 − 16 + =
32π – 64 = 36,53 unidades cuadradas de superficie
Se inscribe una semicircunferencia en un triángulo isósceles de base 16 cm y altura
15 cm, como muestra la figura.
¿Cuál es su radio?
SOLUCIÓN
Se observa en la imagen derecha que hay tres triángulos rectángulos en los que
aplicamos el teorema de Pitágoras:
22222222
6488 ararra −=−=+=
1728964225815 222
==+=+= xx cm
( ) =+−=−++−+−=
=
225643428922564215
17
222222
aaaaxxrax
x
17
64
34
128
12834 === aa cm
De ahí, =×=×=





−×=





−=−=
17
15
8
289
225
64
289
64
164
17
64
6464 2
2
22
rrar
120/17 = 7,0588 cm
En una circunferencia de centro O, AB es un diámetro y P un punto de AB que
dista 9 cm. de O.
Se trazan dos cuerdas perpendiculares a AB que miden 18 cm y 14 cm
respectivamente, dejan a O entre ambas y distan 8 cm entre sí.
Calcula la medida de la cuerda paralela a las otras que pasa por P.
SOLUCIÓN
Intentamos calcular el radio de la circunferencia según las condiciones del segundo
párrafo.
Siendo r el radio de la circunferencia, aplicamos el teorema de la altura en los
triángulos rectángulos ADB y AEB :
( )
( )




=+
+==−




=+
=−




=+
=+




=×+
=+×
498
44
498
3288
498
818
78
98
2
2
bab
baba
bab
ba
bab
aba
ba
ba
( ) 68549366049124984 2
−=++−==−+=+×+ bbbbbbb cm
De ahí, 28568544 −=−+=+= aba cm
Entonces, 85
2
8685285
2
8
=
+−+−
=
++
= r
ba
r cm
Mostramos ahora la cuerda paralela a las anteriores. Su longitud, x2 , es el doble
de la altura del triángulo rectángulo AFB .
Aplicamos en dicho triángulo en teorema de la altura y tenemos que
( ) ( ) 24985985985 22
==−=−×+= xx cm
Por lo tanto, =x2
4 cm
Halla el resto de la división
SOLUCIÓN
Tengamos en cuenta que, aplicando la regla de Ruffini, ( ) ( )21232
−×−=+− xxxx y que
( ) ( ) ( ) ( ) ( ) ( )
RuffiniR
xxxxxxxxx
.
99999999100
422142114242 +−+−×−=+−×+−=+−×=+−
( ) ( ) ( ) ++−×−×++++++=+− 2211......42 296979899100
.
xxxxxxxxxx
RuffiniR
( ) ( ) 2231......42 2296979899100
+++−×++++++=+− xxxxxxxxxx , luego el resto solicitado es
x + 2
¿Cuántos números naturales de 4 cifras terminan en 36 y son múltiplos de 36?
SOLUCIÓN
Para que sean múltiplos de 9436 ×= deben ser múltiplos de 4 y de 9
Todos los números acabados en 36 son múltiplos de 4 pues el número formado por sus dos últimas cifras
( )36 es múltiplo de 4
Para que sean, también, múltiplos de 9 la suma de sus cifras debe ser múltiplo de 9: si el número es
••
=+=++ 993636 babaab , siendo 0≠a
Esto se cumple en estos casos: 9936,9036,8136,7236,6336,5436,4536,3636,2736,1836
10 números
Halla el valor exacto de
sin usar calculadora.
SOLUCIÓN
Llamamos = 2 + √5 + 2 − √5 ⇒ = 2 + √5 + 2 − √5 ⇒
⇒ = 2 + √5 + 3 × 2 + √5 × 2 − √5 + 3 × 2 + √5 × 2 − √5 + 2 − √5
Ahora bien, como 3 × 2 + √5 × 2 − √5 = 3 × 2 + √5 × 2 + √5 × 2 − √5 =
= 3 × 2 + √5 × 2 + √5 × 2 − √5 = 3 × 2 + √5 × √−1 = −3 × 2 + √5 y
3 × 2 + √5 × 2 − √5 = 3 × 2 + √5 × 2 − √5 × 2 − √5 =
= 3 × 2 + √5 × 2 − √5 × 2 − √5 = 3 × √−1 × 2 − √5 = −3 × 2 − √5 , se sigue que
= 2 + √5 − 3 × 2 + √5 − 3 × 2 − √5 + 2 − √5 = 4 − 3 × 2 + √5 + 2 − √5 ⇒
⇒ = 4 − 3 ⇒ + 3 − 4 = 0
Usando la Regla de Ruffini,
1 0 3 −4
1 ) 1 1 4
1 1 4 ( 0
⇒ + 3 − 4 = ( − 1) × ( + + 4) = 0
Como + + 4 = 0 no posee soluciones reales pues =
±√
, la única solución real se obtiene de
− 1 = 0 ⇒ =
1
Empezando con 46 se forma una secuencia de dígitos colocando en cada
paso, a continuación del ultimo número escrito, el producto de los dos
últimos dígitos que se escribieron.
Los primeros dígitos pueden verse en la imagen adjunta.
Calcula el dígito que está en la posición 1089
SOLUCIÓN
La secuencia es 462483261224832612 … y puede observarse que, a partir del tercer dígito de la secuencia,
se repite la serie, de 8 dígitos, 24832612: hay dos dígitos iniciales y, posteriormente, series iguales de 8
dígitos.
Entonces, como 1089 = 136 × 8 + 1 = 2 + 135 × 8 + 7, el dígito correspondiente a esa posición será el
séptimo de la serie que se repite:
1
La gráfica de la función f está compuesta por tres trozos rectilíneos, como
muestra la figura.
¿Cuántas soluciones tiene la ecuación f ( f ( f ( x ) ) ) = 0 ?
SOLUCIÓN
La función está compuesta de tres tramos rectos. Los dos extremos de pendiente 1 y el del medio de
pendiente −1
Como pasa por los puntos −4 , 0 y 0 , 0 , su expresión analítica es =
+ 4, < −2
– , − 2 ≤ < 0
, ≥ 0
Como −4 = 0 = 0 , = 0 ⇒
⎩
⎪
⎨
⎪
⎧ = −4 ⇒ = −8 ⇒ = −12
= 0 ⇒
= −4 ⇒ = −8
= 0 ⇒
= −4
= 0
La ecuación planteada posee
4 soluciones
Sea t una recta y P un punto exterior.
Sobre la recta se marcan de izquierda a derecha los puntos A, B, C, D, E de
modo que PA=PB, PB=BC, PC=CD y PD=DE.
Se traza por P la paralela a t y se marca en esta paralela el punto Q tal que
PQED es un paralelogramo.
Si los ángulos QED y APB son iguales, ¿cuánto mide el ángulo BAP?
SOLUCIÓN
Construimos el esquema que nos plantea el enunciado:
Llamamos = = y seguimos el siguiente razonamiento:
Como es un paralelogramo, = =
es un triángulo isósceles: = = ⇒ = 180° − − = 180° − 2 ⇒ = 2
es un triángulo isósceles: = = 2 ⇒ = 180° − − = 180° − 4 ⇒ = 4
es un triángulo isósceles: = = 4 ⇒ = 180° − − = 180° − 8 , pero
= ⇒ = 180° − 8 ⇒ 9 = 180° ⇒ = 20° ⇒ = 4 =
80o
Si f es una función periódica de periodo T = 5 y en el intervalo [3 , 8) verifica que
¿cuál es el valor de f (2019) ?
SOLUCIÓN
+ 5 = , y como 2019 = 5 × 403 + 4 ⇒ 2019 = 4 + 5 × 403 = 4 ⇒
2019 = 4 = 4 − 10 × 4 + 25 =
1
El druida Panoramix desea preparar 24 cucharones de una pócima mágica que contenga
las sustancias A, B, C por partes iguales.
Dispone de un recipiente donde hay A y C mezclados por partes iguales; otro en el que
hay A y B mezclados en la proporción 2:3 y un tercero en el que hay B y C mezclados en
la proporción 1:2.
¿Cuántos cucharones de cada recipiente debe mezclar para obtener la pócima deseada?
SOLUCIÓN
Sean , , los cucharones que tienen que contribuir, de cada recipiente, a la mezcla deseada. Está claro,
entonces, que + + = 24
Los cucharones del primer recipiente aportarán de A y de C, medidos en “unidades de cucharón”.
Los cucharones del segundo recipiente aportarán de A y de B, medidos “en unidades de cucharón”.
Los cucharones del tercer recipiente aportarán de B y de C, medidos en “unidades de cucharón”.
Como las tres sustancias deben ser aportadas en la misma cantidad, deberá ser + = + = + ⇒
⇒ 15 + 12 = 18 + 10 = 15 + 20 tomando denominador común.
Se obtiene el sistema
+ + = 24
15 + 12 = 18 + 10
18 + 10 = 15 + 20
⇒
+ + = 24
15 − 6 − 10 = 0
15 − 18 + 10 = 0
ª→ ª ª
ª→ ª ª
!
+ + = 24
30 − 24 = 0
12 − 20 = 0
⇒
⇒
+ + = 24
5 − 4 = 0
3 − 5 = 0
⇒ #
+ + = 24
=
$
=
⇒
⎩
⎪
⎨
⎪
⎧
$
+ + = 24⇒
)
= 24 ⇒ = 10
=
$
⇒ =
$×)+
⇒ = 8
= ⇒ =
×)+
⇒ = 6
Se obtienen así + = + = + = 8 “unidades de cucharón” de cada sustancia A, B y C mezclando
8, 10 y 6 cucharones de cada recipiente, siguiendo el orden
Halla todos los pares de enteros (x, y), con x ≤ y, que verifican que su producto
es igual a cinco veces su suma.
SOLUCIÓN
Sea × = 5 × + . Entonces, ∃ ∈ ∋ = 5 . Elegimos esta incógnita porque los resultados serán
semejantes con la elección de la otra.
× = 5 × + ⇒ 5 = 5 × + 5 ⇒ = + 5 ⇒ × − 1 = 5 ⇒ =
5
− 1
∈
Evidentemente solo puede ser par: si fuera impar, − 1 (par) no podría dividir a 5 , producto de dos
impares. Además, dada la estructura de la fracción, debe ser − 1 = ±1 o − 1 = ±5
Los únicos casos posibles son, entonces, los siguientes:
• = 0 ⇒ = 0, = 0
• = 2 ⇒ = 10, = 10
• = 6 ⇒ = 6, = 30
• = −4 ⇒ = 4, = −20.
Puede obtenerse el correspondiente a este último par haciendo la otra elección inicial para que sea válido
≤ , por lo que los pares son
(0 , 0); (10 , 10); (6 , 30); (-20 , 4)
Escribe una lista de cinco números naturales distintos y mayores
que 1 de manera que cada número sea múltiplo del anterior y
que la suma de los cinco números sea 517.
SOLUCIÓN
Los números serán , × , × × , × × × , × × × × tales que > 1 , , , , ∈
Además, + × + × × + × × × + × × × × = 517 = 11 × 47 ⇒
⇒ × 1 + + × + × × + × × × = 11 × 47
Tomamos = 11 ⇒ 1 + + × + × × + × × × = 47 ⇒
⇒ + × + × × + × × × = 46 ⇒ × 1 + + × + × × = 46 = 2 × 23
Tomamos = 2 ⇒ 1 + + × + × × = 23 ⇒ + × + × × = 22 ⇒
⇒ × 1 + + × = 22 = 2 × 11
Tomamos = 2 ⇒ 1 + + × = 11 ⇒ + × = 10 ⇒ × 1 + = 10 = 2 × 5, y tomamos
= 2, = 4
De esta manera, los números son = 11, × = 11 × 2 = 22, × × = 22 × 2 = 44,
× × × = 44 × 2 = 88, × × × × = 88 × 4 = 352 tales que
11 + 22 + 44 + 88 + 352 = 517
La posibilidad de que fuera = 47 no permitiría llegar a ninguna lista válida, por lo que los números son
11, 22, 44, 88, 352
Si f es la función, definida en el intervalo (0, 1) y 0 < α < π/2, tal que
halla el valor de f (tg2
α)
SOLUCIÓN
Hacemos =
Entonces, = = ⇒ − = ⇒ + = ⇒ 1 + × = ⇒ = ⇒ =
De ahí, = = = √ = =
!
"# !
!
"# !
=
!
"# !
"# !$ !
"# !
=
!
"# !
%
"# !
= √&'( =
sen α
Una hormiga sale de su hormiguero y recorre en línea recta un tramo de x cm,
luego gira 90o
y recorre en línea recta otro tramo de x/2 cm, luego vuelve a girar
90o
y recorre un tramo de x/22
cm y, así sucesivamente, siempre recorriendo un
tramo de longitud igual a la mitad del anterior.
El sentido en que gira lo decide en cada vértice.
¿Cuál es la menor distancia al hormiguero a la que puede estar la hormiga después de haber recorrido 100
tramos?
SOLUCIÓN
La trayectoria más óptima para el enunciado del problema es hacer
los dos primeros tramos alejándose, inevitablemente, del
hormiguero para, después, ir acercándose a él con una trayectoria
‘en escalera’, como se aprecia en la figura.
Llamamos a la distancia pedida en centímetros, hipotenusa del
triángulo rectángulo cuyos catetos tienen, de medidas, y
centímetros
= − − − + … =⏞ − − − − … − = − − − − … − ⇒
⇒ = − × !1 + + # $ + … + # $
%
&, y la expresión encerrada entre paréntesis es la suma de los
primeros 49 términos de una progresión geométrica cuyo primer término es 1 y la razón es
Por lo tanto, = − × #) ×
* +
*+
$ = − ×
#
,
$ +
,
+
= −
×! +#
,
$ &
-
≅ −
-
⇒ ≅
-
cm, pues
1 − # $
/
≅ 1
= − %
− -
− %
− … =⏞ − − 0 − 1 − … − = − %
× !1 + + # $ + … + # $
%
&
De manera similar al caso anterior, tenemos que = −
%
× #) ×
* +
*+
$ = × 2 − × #) ×
* +
*+
$3 ⇒
⇒ = ⇒ ≅ -
cm
Aplicando ahora el teorema de Pitágoras en el triángulo rectángulo con las aproximaciones propuestas,
= + = + # $ = ≅ × #
-
$ = ×
/
× =
/
× ⇒ ≅
√
-
× =
√5x/3 cm = 0,745x cm
En un cajón hay calcetines de ocho colores y ocho de cada color.
Si sacamos dos calcetines al azar del cajón, ¿cuál es la probabilidad de que
sean del mismo color?
SOLUCIÓN
La probabilidad de sacar dos calcetines del mismo color será la probabilidad de sacar un segundo calcetín del
mismo color que el extraído en primer lugar.
Al ir a sacar el segundo quedan 63 calcetines en el cajón, de los cuales hay 7 con el mismo color del ya
extraído, por lo que la probabilidad es = =
1/9 = 11,11 %
Sea ABCD un paralelogramo tal que el lado BC mide 13 cm, la altura
correspondiente a la base AB mide 12 cm y el ángulo ABC es agudo.
Sea E un punto en la prolongación de BC tal que el ángulo DEC = 90o
Sabiendo que CE = 5 cm, calcula el área del cuadrilátero ABED.
SOLUCIÓN
Al construir la figura se puede observar que el cuadrilátero es un
trapecio rectángulo.
Sus bases miden = = 13 cm y = + = 13 + 5 = 18 cm
y su altura es = cm
Podemos también observar que los triángulos y son semejantes
por ser rectángulos y, además, los ángulos respectivos y son
iguales
Por lo tanto, = ⇒ = ⇒ = cm
Aplicando el teorema de Pitágoras en el triángulo tenemos que = − = 13 − 12 = 25 ⇒
⇒ = 5 ⇒ = = 12 cm
El área del trapecio es × =
! "
× 12 = 31 × 6 =
186 cm2
Si el cociente entre el radio del sector circular y el radio del círculo inscrito es 3, ¿cuál
es el cociente entre sus áreas?
SOLUCIÓN
Construimos el esquema nombrando vértices y centros de sector y círculo
inscrito.
Llamamos al radio del sector y al radio del círculo: = 3, y sea 2 el
ángulo que abarca el sector circular.
Se observa que hay dos triángulos rectángulos iguales, y , en los que
se cumple que = = = = ⇒ = ⇒ = 30°
El sector circular abarca entonces 2 = 60°, sexta parte del círculo al que pertenece por lo que su superficie
será
La proporción que pide el enunciado es = × " # = × 3 =
3/2
¿Cuántos números de 15 dígitos que utilizan exclusivamente
los dígitos 3 y 8 son múltiplos de 11?
SOLUCIÓN
Un número es múltiplo de 11 si la diferencia de las sumas de los dígitos que ocupan lugares pares y la suma de
los dígitos que ocupan lugares impares es 0 o múltiplo de 11
Si suponemos que hay cifras 8 ocupando lugares impares e cifras 8 ocupando lugares pares en el número
de 15 cifras, deberá cumplirse que 8 + 3 × 8 − − 8 + 3 × 7 − = 11 , siendo ∈
8 + 3 × 8 − − 8 + 3 × 7 − = 11 ⇒ 5 + 24 − 5 − 21 = 11 ⇒ 5 − 5 = 11 − 3 ⇒
⇒ 5 − 5 = 11 − 3 ⇒ − = ⇒ − = 2 + , siendo , ∈ ∋ 0 ≤ ≤ 8 y 0 ≤ ≤ 7
A partir de la igualdad anterior, y con las restricciones indicadas, los únicos valores factibles son:
1. = −2 ⇒ − = −5
• = 2, = 7: 2 cifras 8 y 6 cifras 3 ocupando lugares impares y 7 cifras 8 ocupando lugares pares. La
cantidad de números de 15 cifras es !"
#,$
× 1 =
"!
#!×$!
× 1 =
"×&
#
= 28 números
• = 1, = 6: 1 cifra 8 y 7 cifras 3 ocupando lugares impares y 6 cifras 8 y 1 cifra 3 ocupando lugares
pares. La cantidad de números de 15 cifras es !"
,&
× !&
$,
=
"!
!×&!
×
&!
$!× !
= 8 × 7 = 56 números
• = 0, = 5: 8 cifras 3 ocupando lugares impares y 5 cifras 8 y 2 cifras 3 ocupando lugares pares. La
cantidad de números de 15 cifras es 1 × !&
,#
= 1 ×
&!
!×#!
=
&×$
#
= 21 números
2. = 3 ⇒ − = 6
• = 8, = 2: 8 cifras 8 ocupando lugares impares y 2 cifras 8 y 5 cifras 3 ocupando lugares pares. La
cantidad de números de 15 cifras es 1 × !&
#,
= 1 ×
&!
#!× !
=
&×$
#
= 21 números
• = 7, = 1: 7 cifras 8 y 1 cifra 3 ocupando lugares impares y 1 cifra 8 y 6 cifra 3 ocupando lugares
pares. La cantidad de números de 15 cifras es !"
&,
× !&
,$
=
"!
&!× !
×
&!
!×$!
= 8 × 7 = 56 números
• = 6, = 0: 6 cifras 8 y 2 cifras 3 ocupando lugares impares y 7 cifras 3 ocupando lugares pares. La
cantidad de números de 15 cifras es !"
$,#
× 1 =
"!
$!×#!
× 1 =
"×&
#
= 28 números
El total es 28 + 56 + 21 + 21 + 56 + 28 =
210 números
Tengo escritos diez números naturales consecutivos y borro uno de ellos.
Si la suma de los restantes es 2017, ¿qué número he borrado?
SOLUCIÓN
Sean desde a + 9 los diez números naturales consecutivos que tengo y borro el número + , tal que
0 ≤ ≤ 9 y ∈
Entonces, + + 1 + ⋯ + + 9 − + = 2017 ⇒
×
− − = 2017, aplicando fórmula de la
suma de los términos de una progresión geométrica.
×
− + = 2017 ⇒ 10 + 45 − − = 2017 ⇒ 9 − = 1972 ⇒ 9 = 1972 − ⇒
⇒ = = 219 + ∈ . Como 0 ≤ ≤ 9 y ∈ ⇒ = 8 ⇒ = 219 + = 220, y el número
borrado es + = 220 + 8 =
228
El desarrollo decimal de la fracción
tiene un período muy largo.
Halla las tres últimas cifras del período.
SOLUCIÓN
Suponemos = 0, … y hacemos 10 × = 10 × 0, … = … × 0, … ⇒
999. . .999999
!
× = … ⇒ 999. . .999999
!
= 97 × …
Como el producto debe acabar en 9 ⇒ = 7, última cifra del período.
Como 97 × 7 = 679 y el producto debe acabar en 99 ⇒ $ = 6, penúltima cifra del período.
Como 97 × 67 = 6499 y el producto debe acabar en 999 ⇒ $ = 5, antepenúltima cifra del período.
Así, 97 × 567 = 54999
Concluyendo, el período de la fracción acaba en
567
Si a, b y c son números positivos tales que b = a30
y c = a42
, ¿cuánto vale logb c ?
SOLUCIÓN
Como =
ó
= =
7/5
Dado un conjunto de diez números enteros positivos no necesariamente
distintos, se realizan las siguientes diez operaciones: se descarta el primero y
se suman los nueve restantes, se descarta el segundo y se suman los nueve
restantes y se sigue así hasta descartar el último y sumar los nueve restantes.
De esta manera se obtienen sólo nueve resultados distintos, que son 86, 87, 88, 89, 90, 91, 92, 93, 96.
Halla los diez números iniciales.
SOLUCIÓN
Hacemos las diez operaciones citadas y, si son , , … los números buscados, se cumple que
9 × + + ⋯ + = 86 + 87 + 88 + 89 + 90 + 91 + 92 + 93 + 96 + = 812 +
tal que = 9 + 7 (para que la suma sea divisible por 9) y, además, debe ser uno de los valores indicados
como resultados obtenidos.
La única posibilidad es que = 88 = 9 × 9 + 7 y, así,
9 × + + ⋯ + = 86 + 87 + 88 + 89 + 90 + 91 + 92 + 93 + 96 + = 812 + 88 = 900 y
= + + ⋯ + = 100
De esta manera, los números son 100 − 86 = 14, 100 − 87 = 13, 100 − 88 = 12 (dos veces), 100 − 89 =
11, 100 − 90 = 10, 100 − 91 = 9, 100 − 92 = 8, 100 − 93 = 7 y 100 − 96 = 4
Resumiendo,
14, 13, 12, 12, 11, 10, 9, 8, 7 y 4
Sean a y b dos números naturales.
Si a+b y a3
+b3
acaban en 3, ¿en cuánto termina a2
+b2
?
SOLUCIÓN
Como + = + 3 + 3 + = + + 3 × + y
+ = + + 3 × + ⇒ 3 × + acaba en 4 ⇒ acaba en 6
En resumen, + = + 2 + ⇒ + = + − 2
!
= + − 2"
por lo que + acaba en
7
Reemplazando x e y por dígitos, halla todos los números naturales de cinco cifras 65x1y
que son múltiplos de 12.
SOLUCIÓN
Un número es divisible por 12 si lo es, a la vez, por 3 y por 4
Para que sea divisible por 3, la suma de sus cifras debe ser múltiplo de 3 ⇒ 6 + 5 + + 1 + = 3 ⇒
⇒ 12 + + = 3 ⇒ + = 3
Para que sea divisible por 4, la dos últimas cifras deben formar un número múltiplo de 4⇒ 1 = 4 ⇒
= 2
= 6
Entonces,
• = 2; + = 3 ⇒
= 1
= 4
= 7
• = 6; + = 3 ⇒
= 0
= 3
= 6
= 9
y los números son
65016, 65112, 65316, 65412, 65616, 65712, 65916
En la figura, A y B son los puntos medios de los lados del hexágono regular.
El hexágono interior, no regular, tiene los lados paralelos e iguales dos a dos.
Si el área del hexágono regular es de 180 cm2
, ¿cuál es la superficie de la zona
verde?
SOLUCIÓN
Dividimos el hexágono regular en triángulos iguales, como se indica en la imagen
adjunta, viendo que el hexágono se divide en 24 triángulos de los cuales 14 están
en la zona verde, por lo que la superficie de esa zona es × 180 = × 180 =
105 cm2
Un barco navega entre dos orillas paralelas siguiendo el recorrido de la
figura.
Se conocen estas igualdades entre ángulos: ABC=CDE y CBD=CDB
Si sale con un ángulo de 105o
respecto de la primera orilla, ¿ qué ángulo
sigue la trayectoria con la que atraca en la otra orilla?
SOLUCIÓN
Llamamos al ángulo solicitado y al ángulo determinado por la segunda
igualdad.
Según la imagen adjunta, el ángulo = 180° − 105° = 75° y, según el
triángulo , 2 + 40° = 180° ⇒ 2 = 140° ⇒ = 70°
Además, en el triángulo , = = 180° − − por lo que
+ + = 180° ⇒ 2 × 180° − − + 75° = 180°
Como = 70° ⇒ 2 × 180° − − 70° + 75° = 180° ⇒ 220° − 2 + 75° = 180° ⇒ 2 = 115° ⇒ =
°
=
57o
30’ o 180o
-57o
30’=122o
30’
Se inscribe en un hexágono regular otro cuyos vértices son los puntos medios de los
lados del primero.
Halla la proporción entre sus áreas.
SOLUCIÓN
Llamamos , a la apotema y el radio (y lado) del hexágono anterior, y , a la
apotema y radio (y lado) del hexágono exterior. Es evidente que la apotema del
exterior coincide con el radio del exterior.
Si dividimos los hexágonos en seis triángulos equiláteros cada uno, de lados los
radios respectivos, y tomamos uno de cada uno dividiéndolo por la mitad tenemos
dos triángulos equiláteros.
Uno de ellos de lados , , y el otro de lados , ,
Aplicando en ambos el teorema de Pitágoras,
+ =
+ =
⇒
=
=
⇒
=
√
=
√
⇒
=
=
√
La razón entre las superficies es, entonces, = = =
4/3
Juan saca 2700 € en un banco. Le pide al cajero que le entregue cierta
cantidad de billetes de 10 €, 20 veces esa cantidad de billetes de 20 € y el
resto en billetes de 50 €.
¿Cuántos billetes de cada clase le entrega el cajero?
SOLUCIÓN
Llamamos al número de billetes de 10 € que solicita e al número de billetes de 50 € que pide.
Entonces, 10 + 20 × 20 + 50 = 2700 ⇒ 410 + 50 = 2700 ⇒ 41 + 5 = 270 ⇒
⇒ =
270 − 41
5
⇒ = 54 − 8 −
5
Como , ∈ , los únicos valores admisibles en esta ecuación son = 5 ⇒ = 54 − 8 × 5 − = 13, por lo
que los billetes entregados son
5 billetes de 10 €, 100 billetes de 20 € y 13 billetes de 50 €
El área de un trapecio es de 1400 cm2
y su altura mide 50 cm.
Si sus dos bases son valores naturales múltiplos de ocho, ¿cuántos
posibles trapecios distintos verifican esos datos?
SOLUCIÓN
Llamando a las longitudes de bases 8 y 8 , siendo < y , ∈ ⇒ × 50 = 1400 ⇒
⇒ 8 × + =
×
= 56 ⇒ + = = 7, de donde obtenemos las siguientes posibilidades:
• = 1, = 6
• = 2, = 5
• = 3, = 4
En conclusión, los posibles trapecios distintos son
3
Un automóvil viaja de A a C a una velocidad constante de 90 kilómetros
por hora y, en el camino entre A y C, pasa por B.
Cuando son las 8 de la mañana ha recorrido 1/4 de la distancia entre A y
B y cuando son las 10 de la mañana ya ha recorrido 3/4 del camino entre
B y C.
Calcula la distancia entre A y C.
SOLUCIÓN
Sean , las distancias respectivas, en kilómetros, de A a B y de B a C. Es evidente que la distancia buscada, de
A a C, es +
Si hacemos un esquema del trayecto observamos que de las 8
horas a las 10 horas de la mañana el automóvil ha recorrido
+ : 2 horas a 90 k/h
∙
+ = 90 × 2 = 180 ⇒
⇒ 3 + 3 = 180 × 4 = 720 ⇒ + = =
240 kilómetros
Si AB = AC y AE = AD, ¿cuánto vale el ángulo x ?
SOLUCIÓN
Sean , los ángulos iguales de los triángulos isósceles respectivos y
En el triángulo , = 180° − 30° + ⇒ = 150° −
En el triángulo , = 180° − 2 ⇒ = − 30° = 150° − 2
Por lo tanto, en el triángulo , 2 + 150° − 2 = 180° ⇒ 2 = 30° + 2 ⇒ = 15° +
Para acabar, como + + = 180° ⇒ 150° − + 15° + + = 180° ⇒ 165° + = 180° ⇒ =
15o
Sea ABCD un rectángulo con AB = 30 cm y BC = 16 cm.
Si E y F son puntos, respectivamente, en los lados AB y CD tales que el
cuadrilátero AFCE es un rombo, calcula la medida de EF.
SOLUCIÓN
Llamamos a la diferencia de longitudes del lado mayor del
rectángulo y del lado del rombo. Entonces, el lado del rombo mide
30 − cm.
Aplicamos el teorema de Pitágoras en el triángulo rectángulo :
+ = ⇒ + 16 = 30 − ⇒
⇒ + 256 = 900 − 60 + ⇒ 60 = 644 ⇒ = = cm
Aplicando ahora el teorema de Pitágoras en el triángulo rectángulo , = + ⇒
⇒ = 30 − 2 + 16 ⇒ = 900 − 120 + 4 + 256 ⇒ = 4 − 120 + 1156 ⇒
⇒ = 4 × − 30 + 289 ⇒ = 4 ×
161
15
− 30 ×
161
15
+ 289 = 4 ×
18496
225
⇒ = 2 ×
136
15
=
272/15 = 18,13 cm
¿Cuántos triángulos escalenos hay cuyo perímetro sea menor que 13 y que
tengan las medidas de todos sus lados expresadas con números naturales?
SOLUCIÓN
Teniendo en cuenta que, en cualquier triángulo, la longitud de un lado debe ser menor que la suma de las
longitudes, las posibilidades son:
• Perímetro: 12 – máxima longitud de un lado: 5
o Lados 2, 5, 5 (isósceles); 3, 4, 5 (escaleno); 4, 4, 4 (equilátero)
• Perímetro: 11 – máxima longitud de un lado: 5
o Lados 1, 5, 5 (isósceles); 2, 4, 5 (escaleno); 3, 3, 5 (isósceles); 3, 4, 4 (isósceles)
• Perímetro: 10 – máxima longitud de un lado: 4
o Lados 2, 4, 4 (isósceles); 3, 3, 4 (isósceles)
• Perímetro: 9 – máxima longitud de un lado: 4
o Lados 1, 4, 4 (isósceles); 2, 3, 4 (escaleno); 3, 3, 3 (equilátero)
• Perímetro: 8 – máxima longitud de un lado: 3
o Lados 2, 3, 3 (isósceles)
• Perímetro: 7 – máxima longitud de un lado: 3
o Lados 1, 3, 3 (isósceles); 2, 2, 3 (isósceles)
• Perímetro: 6 – máxima longitud de un lado: 2
o Lados 2, 2, 2 (equilátero)
• Perímetro: 5 – máxima longitud de un lado: 2
o Lados 1, 2, 2 (isósceles)
• Perímetro: 4 – máxima longitud de un lado: 1
o No puede haber triángulos
• Perímetro: 3 – máxima longitud de un lado: 1
o Lados 1, 1, 1 (equilátero)
Hay, con las condiciones fijadas por el enunciado,
3 triángulos escalenos
Halla todos los números de cuatro cifras abcd tales que ab + cd = bc y b – c = d
SOLUCIÓN
+ = ⇒ 10 + + 10 + = 10 + ⇒ 10 + = 9 − 9 10 + = 9 ⇒ 10 = 8 ⇒
⇒ 5 = 4 ⇒ = 4 = 5
Además, − = 5 ⇒ = + 5
Por tanto, los números posibles son,
= 4; = 5; = 0; = 5 ⇒
= 4; = 5; = 1; = 6 ⇒
= 4; = 5; = 2; = 7 ⇒ #$
= 4; = 5; = 3; = 8 ⇒ &'
= 4; = 5; = 4; = 9 ⇒ (
El resultado son los números
4505; 4615; 4725; 4835; 4945
Halla el mayor número natural n para el que 5n
es divisor del número
SOLUCIÓN
= 98! + 99! + 100! = 1 + 99 + 99 × 100 × 98! = 10000 × 98! = 5 × 2 × 98!
Ahora bien, 98! contiene, como factores, a todos los múltiplos de 5 menores de 100:
98! = 1 × … × 5 × … × 10 × … × 15 × … × 20 × … … … … … × 90 × … × 95 × 96 × 97 × 98
Como hay 10 números acabados en 5 y dos de ellos tienen, en su factorización, dos 5 25 75 , todos esos
números multiplicados tendrán 5 como factor con la mayor potencia de 5.
Hay 9 números acabados en 0 que tienen un 5 en su factorización excepto uno 50 que tiene dos 5.
Entonces, todos esos números multiplicados tendrán 5 como factor con la mayor potencia de 5.
Por lo tanto 98! = 5 × 5 × =⏞
! "#$ % $ & ∗
5 × , siendo ∈ no divisible por 5
De ahí = 98! + 99! + 100! = 5 × 2 × 98! = 5 × 2 × 5 × =⏞
∗
5 )
× 2 × , por lo que * =
26
Una hoja de papel rectangular se divide mediante un solo corte en un triángulo y un
pentágono.
Las longitudes de los lados del pentágono son 17, 25, 28, 33 y 43, en algún orden.
Calcula el área del pentágono.
SOLUCIÓN
Representamos el corte con la hoja horizontal y señalamos las cinco
longitudes como , , , , no necesariamente en el mismo orden de
los valores expuestos 17, 25, 28, 33, 43 pero, lógicamente, = 43
Según la imagen, debe cumplirse que
>
>
− + − =
Teniendo en cuenta que , deben ser valores grandes de entre los
propuestos y , valores pequeños (y no puede ser nunca 25 al ver las diferencias que quedan restantes), se
pueden cribar posibilidades hasta dar con la válida: haciendo = 28, = 33, = 25, = 17 se cumplen las
tres condiciones
43 > 28
33 > 25
43 − 28 + 33 − 25 = 17
y la superficie del pentágono es igual a la del rectángulo menos la del triángulo rectángulo:
Sup á = a× c −
a− b × c − d
2
= 43 × 33 −
43 − 28 × 33 − 25
2
= 1419 −
15 × 8
2
=
1359 unidades cuadradas
Se elige al azar un número de cuatro cifras.
¿Cuál es la probabilidad de que sea impar y con todas sus cifras diferentes?
SOLUCIÓN
Números de cuatro cifras hay (de 1000 a 9999) 9999 − 1000 + 1 = 9000
Veamos los números de cuatro cifras que pueden crearse con todas sus cifras diferentes: la cifra de los miles
debe ser una de las nueve cifras no nulas. Por cada una de las cifras elegidas quedan nueve restantes (ahora se
incluye el 0) para las centenas. Para las decenas quedan ocho y para las unidades siete. Por tanto, la cantidad
de números de cuatro cifras distintas que pueden formarse son 9 × 9 × 8 × 7
De ellas acaban en 1, 3, 5, 7, 9 la cantidad de 8 × 8 × 7 números en cada caso por las mismas razones que en el
cálculo anterior y habiendo prefijado la cifra de las unidades. El total de números impares de cuatro cifras
distintas es, entonces, 5 × 8 × 8 × 7, por lo que la probabilidad pedida es =
× × ×
=
× × ×
× ×
=
×
×
=
56/225 ≈ 0,2489
Halla cuantos números naturales n menores que 1000 hay tales que n2
termina en
SOLUCIÓN
Si acaba en 44 ⇒ acaba en 2 o en 8
• Si = 100 + 10 + 2, siendo entero no negativo y ∈ ∋ 0 ≤ ≤ 9,
= 100 + 10 + 2 × 100 + 10 + 2 = ⋯ + 100 + 20 + 20 + 4 = ⋯ + 40 + 4 ⇒
40 + 4 = ⋯ 44 ⇒
= 1 ⇒ los números pedidos son los que acaban en 12
= 6 ⇒ los números pedidos son los que acaban en 62
)
• Si = 100 + 10 + 8, siendo entero no negativo y ∈ ∋ 0 ≤ ≤ 9,
= 100 + 10 + 8 × 100 + 10 + 8 = ⋯ + 100 + 80 + 80 + 64 = ⋯ + 160 + 64 ⇒
160 + 64 = ⋯ 44 ⇒
= 3 ⇒ los números pedidos son los que acaban en 38
= 8 ⇒ los números pedidos son los que acaban en 88
)
De todo lo anterior,
• acaban en 12 menores de 1000 ⇒ 12, 112, 212, … , 912: 10 números
• acaban en 62 menores de 1000 ⇒ 62, 162, 262, … , 962: 10 números
• acaban en 38 menores de 1000 ⇒ 38, 138 238, … , 938: 10 números
• acaban en 88 menores de 1000 ⇒ 88, 188, 288, … , 988: 10 números
En total, los números son
40 números
Sean a, b, c tres números reales no nulos tales que a + b + c = 0
Calcula todos los posibles valores de la expresión
SOLUCIÓN
Como + + = 0, uno o dos números de los tres son negativos y los restantes (o el restante) positivos.
Además,
=
| |
+
| |
+
| |
+
∙ ∙
| | ∙ | | ∙ | |
=
∙ | | ∙ | | + | | ∙ ∙ | | + | | ∙ | | ∙ + ∙ ∙
| | ∙ | | ∙ | |
y los cuatro sumandos del numerador son iguales en valor absoluto.
Estudiemos los dos casos posibles y fijémonos en los sumandos del numerador:
1. , positivos y negativo ⇒
∙| |∙| |
| |∙ ∙| |
| |∙| |∙
∙ ∙
⇒ =
∙| |∙| | | |∙ ∙| | | |∙| |∙ ∙ ∙
| |∙| |∙| |
=⏞ | |∙| |∙| |
= 0
2. positivo y , negativos ⇒
∙| |∙| |
| |∙ ∙| |
| |∙| |∙
∙ ∙
⇒ =
∙| |∙| | | |∙ ∙| | | |∙| |∙ ∙ ∙
| |∙| |∙| |
=⏞ | |∙| |∙| |
= 0
Por tanto, en todos los casos, =
0
Sean a y b números reales distintos tales que 2a2
+ 2b2
= 5ab.
Halla todos los posibles valores de
SOLUCIÓN
2 + 2 = 5 ⇒ 2 + 2 − 4 = 5 − 4 ⇒ 2 × + − 2 = ⇒ − =
2
2 + 2 = 5 ⇒ 2 + 2 + 4 = 5 + 4 ⇒ 2 × + + 2 = 9 ⇒ + =
9
2
Entonces,
+
−
=
+
−
=
9
2
2
= 9 ⇒
+
−
=
±3
En un triángulo rectángulo de catetos 5 y 12 se inscribe una
semicircunferencia como se muestra en la figura.
¿Cuál es su radio?
SOLUCIÓN
Nombramos todos los puntos y líneas interesantes y llamamos al radio
pedido.
Evidentemente, = = 5; = = ; = − = 12 −
Como el triángulo es rectángulo, por el teorema de Pitágoras cumple
que = + = 12 + 5 = 144 + 25 = 169 ⇒ = 13
Entonces = − = 13 − 5 = 8
De todo lo anterior, aplicando también el teorema de Pitágoras en el triángulo rectángulo obtenemos que
= + ⇒ 12 − = + 8 ⇒ 144 − 24 + = + 64 ⇒ 24 = 144 − 64 = 80 ⇒ = =
10/3 unidades
ABC es un triángulo tal que A > 90o
y M es el punto medio del lado BC.
Si BAM = 90o
, AB = 35 y AC = 77, calcula BC.
SOLUCIÓN
Señalamos lo que nos indica el enunciado y, además, hacemos = 2 , valor
buscado.
Prolongando el lado y trazando su perpendicular por el vértice , llamamos
al vértice intersección de ambas rectas.
Observamos que los triángulos y son rectángulos y semejantes, por lo
que = = = = 2 ⇒ = 2 × = 2 × 35 = 70 ⇒ = 35
Por lo tanto, aplicando el teorema de Pitágoras en el triángulo rectángulo , = − ⇒
⇒ = 77 − 35 = 4704
Aplicamos ahora el teorema de Pitágoras en el triángulo : = + = 4704 + 70 = 9604 ⇒
⇒ = √9604 =
98 unidades
Si m y n son números impares tales que m > n, determina el mayor número natural que divide a todos los
números de la forma
SOLUCIÓN
Como y son impares, = 2 + 1 y = 2 + 1, siendo > naturales.
Entonces, − = + × − = 2 + 2 + 2 × 2 − 2 = 4 × + + 1 × −
Evidentemente, + + 1 y − tienen distinta paridad por lo que uno de ellos (y solo uno) es par y de ahí se
deduce que todo número del tipo − es divisible por
8
Halla todos los números naturales x, y, z tales que
SOLUCIÓN
= 5 + ⇒ = − 5 = ⇒ + = y < 1 al ser , ∈
La única posibilidad es, entonces, que 9 + = ⇒ = 9 y = ⇒ + = 2 ⇒ = = 1
x = 9, y = 1, z = 1
Sea la sucesión
¿Cuál es el menor número natural n tal que el producto de los n primeros términos de la sucesión es mayor
que 100000?
SOLUCIÓN
Buscamos el menor número natural tal que
10 × 10 × … … × 10 > 10 = 100000 ⇒ 10
⋯
> 10 ⇒
1
13
+
2
13
+ ⋯
13
> 5
Por tanto,
⋯
> 5 ⇒ 1 + 2 + ⋯ + > 65
é" # $
%"$&" #ó "# é #(
)********************+
×
> 65 ⇒ 1 + × > 130
Si = 10 ⇒ 1 + × = 11 × 10 = 110 < 130 y si = 11 ⇒ 1 + × = 12 × 11 = 132 > 130, por lo
que el valor pedido es =
11
Sea ABCD un paralelogramo de lados AB, BC, CD y DA.
Se traza por D una recta que corta al lado BC en P y a la prolongación del
lado AB en Q.
Si el área del cuadrilátero ABDP es 29 y el área del triángulo DPC es 8, halla el
área del triángulo CPQ.
SOLUCIÓN
Dibujamos todos los elementos que nos da el enunciado y llamamos ,
a los lados del paralelogramo; = ; base de los triángulos y
; es la altura del paralelogramo y del triángulo , y es la
altura del triángulo .
Está claro que la superficie del paralelogramo es = 29 + 8 = 37
Además, la diferencia entre las superficies de los triángulos semejantes
y es
×
− = 29 ⇒ + − = 58 ⇒ 37 + − = 58 ⇒ − = 21
Finalmente, la superficie del triángulo es la diferencia de las superficies de los triángulos y :
"#$%&' =
2
−
2
=
−
2
=
21/2 unidades cuadradas
Si a, b, c son tres números racionales tales que ab = 26, ac = 128, bc = 52, calcula
SOLUCIÓN
= 26 = 2 × 13
= 128 = 2
= 52 = 2 × 13
⇒ × × = = 2 × 13 ⇒ = 2 × 13
Entonces, + + = + + =
×
×
+
×
+
×
×
= 2 + + 2 = 8 + + 16 =
109/4
Un triángulo equilátero se divide en cuatro triangulitos equilateros iguales, quedando
determinados 9 segmentos que son lados de triangulitos.
Distribuye los números 1, 2, 3, 4, 5, 6, 7, 8, 9 en los lados de los triangulitos, sin
repeticiones, de modo que la suma de los tres números correspondientes a cada
triangulito sea siempre la misma.
SOLUCIÓN
La suma de los lados de los tres triangulitos exteriores es (no necesariamente en el
mismo orden que los valores indicados,
+ + + + + + + ℎ + = 1 + 2 + 3 + 4 + 5 + 6 + 7 + 8 + 9 = 45
Entonces, los lados de cada triangulito suman = 15
Cuatro posibles resultados (hay más) son:
Sean m y n dos números naturales tales que n > 1 y mn
= 225
×340
Halla
SOLUCIÓN
> 1 ⇒ = 25,40 = 5 ⇒ = 2 × 3 = 2 × 3 ⇒ = 2 × 3 = 32 × 6561 = 209952
Entonces, + = 209952 + 5
209957

Más contenido relacionado

La actualidad más candente

100 problemas maravillosos de matemáticas - Libro 1
100 problemas maravillosos de matemáticas - Libro 1100 problemas maravillosos de matemáticas - Libro 1
100 problemas maravillosos de matemáticas - Libro 1José Mari Melgarejo Lanero
 
100 problemas maravillosos de matemáticas - Libro 12
100 problemas maravillosos de matemáticas - Libro 12100 problemas maravillosos de matemáticas - Libro 12
100 problemas maravillosos de matemáticas - Libro 12
José Mari Melgarejo Lanero
 
Matemática 8° Básico, tomo 2
Matemática 8° Básico, tomo 2Matemática 8° Básico, tomo 2
Matemática 8° Básico, tomo 2
Alejandra
 
Domino de factorizació
Domino de factorizacióDomino de factorizació
Domino de factorizació
David Valdez López
 
100 problemas maravillosos de matemáticas - Libro 7
100 problemas maravillosos de matemáticas - Libro 7100 problemas maravillosos de matemáticas - Libro 7
100 problemas maravillosos de matemáticas - Libro 7
José Mari Melgarejo Lanero
 
Evaluacion potencias 8vo basico 2007
Evaluacion  potencias 8vo basico 2007Evaluacion  potencias 8vo basico 2007
Evaluacion potencias 8vo basico 2007teresara
 
100 problemas maravillosos de matemáticas - Libro 15
100 problemas maravillosos de matemáticas - Libro 15100 problemas maravillosos de matemáticas - Libro 15
100 problemas maravillosos de matemáticas - Libro 15
José Mari Melgarejo Lanero
 
Operaciones enteros con fichas colores
Operaciones enteros con fichas coloresOperaciones enteros con fichas colores
Operaciones enteros con fichas coloresKarina Ruiz
 
Problemas con fracciones resueltos
Problemas con fracciones resueltosProblemas con fracciones resueltos
Problemas con fracciones resueltosColegio Vedruna
 
9. Prueba DiagnóStico I
9. Prueba DiagnóStico   I9. Prueba DiagnóStico   I
9. Prueba DiagnóStico IJuan Galindo
 
Cuadros magicos
Cuadros magicosCuadros magicos
Cuadros magicos
Рикардо Скот
 
Operaciones con-monomios (1)
Operaciones con-monomios (1)Operaciones con-monomios (1)
Operaciones con-monomios (1)
Cristopher Sanhueza
 
Cuestionario de matematica grado 8
Cuestionario de matematica grado 8Cuestionario de matematica grado 8
Cuestionario de matematica grado 8marcelariosolorzano
 
Ecuaciones de segundo grado
Ecuaciones de segundo gradoEcuaciones de segundo grado
Ecuaciones de segundo gradocesar canal mora
 
Acertijo-rompecabezas matemático con operaciones con fracciones para formar l...
Acertijo-rompecabezas matemático con operaciones con fracciones para formar l...Acertijo-rompecabezas matemático con operaciones con fracciones para formar l...
Acertijo-rompecabezas matemático con operaciones con fracciones para formar l...
JAVIER SOLIS NOYOLA
 
100 problemas maravillosos de matemáticas - Libro 6
100 problemas maravillosos de matemáticas - Libro 6100 problemas maravillosos de matemáticas - Libro 6
100 problemas maravillosos de matemáticas - Libro 6
José Mari Melgarejo Lanero
 
Taller general sistemas de ecuaciones lineales ejercicios para resolver
Taller general sistemas de ecuaciones lineales ejercicios para resolverTaller general sistemas de ecuaciones lineales ejercicios para resolver
Taller general sistemas de ecuaciones lineales ejercicios para resolver
Marvy Henao
 
Guía nº ecuaciones 1,2,3 8° básico
Guía  nº ecuaciones 1,2,3 8° básicoGuía  nº ecuaciones 1,2,3 8° básico
Guía nº ecuaciones 1,2,3 8° básiconigatiti
 

La actualidad más candente (20)

100 problemas maravillosos de matemáticas - Libro 1
100 problemas maravillosos de matemáticas - Libro 1100 problemas maravillosos de matemáticas - Libro 1
100 problemas maravillosos de matemáticas - Libro 1
 
100 problemas maravillosos de matemáticas - Libro 12
100 problemas maravillosos de matemáticas - Libro 12100 problemas maravillosos de matemáticas - Libro 12
100 problemas maravillosos de matemáticas - Libro 12
 
Matemática 8° Básico, tomo 2
Matemática 8° Básico, tomo 2Matemática 8° Básico, tomo 2
Matemática 8° Básico, tomo 2
 
Domino de factorizació
Domino de factorizacióDomino de factorizació
Domino de factorizació
 
100 problemas maravillosos de matemáticas - Libro 7
100 problemas maravillosos de matemáticas - Libro 7100 problemas maravillosos de matemáticas - Libro 7
100 problemas maravillosos de matemáticas - Libro 7
 
Evaluacion potencias 8vo basico 2007
Evaluacion  potencias 8vo basico 2007Evaluacion  potencias 8vo basico 2007
Evaluacion potencias 8vo basico 2007
 
100 problemas maravillosos de matemáticas - Libro 15
100 problemas maravillosos de matemáticas - Libro 15100 problemas maravillosos de matemáticas - Libro 15
100 problemas maravillosos de matemáticas - Libro 15
 
Operaciones enteros con fichas colores
Operaciones enteros con fichas coloresOperaciones enteros con fichas colores
Operaciones enteros con fichas colores
 
Problemas con fracciones resueltos
Problemas con fracciones resueltosProblemas con fracciones resueltos
Problemas con fracciones resueltos
 
9. Prueba DiagnóStico I
9. Prueba DiagnóStico   I9. Prueba DiagnóStico   I
9. Prueba DiagnóStico I
 
Cuadros magicos
Cuadros magicosCuadros magicos
Cuadros magicos
 
Operaciones con-monomios (1)
Operaciones con-monomios (1)Operaciones con-monomios (1)
Operaciones con-monomios (1)
 
Cuestionario de matematica grado 8
Cuestionario de matematica grado 8Cuestionario de matematica grado 8
Cuestionario de matematica grado 8
 
2 Ecuaciones CuadráTicas
2  Ecuaciones CuadráTicas2  Ecuaciones CuadráTicas
2 Ecuaciones CuadráTicas
 
Ecuaciones de segundo grado
Ecuaciones de segundo gradoEcuaciones de segundo grado
Ecuaciones de segundo grado
 
Acertijo-rompecabezas matemático con operaciones con fracciones para formar l...
Acertijo-rompecabezas matemático con operaciones con fracciones para formar l...Acertijo-rompecabezas matemático con operaciones con fracciones para formar l...
Acertijo-rompecabezas matemático con operaciones con fracciones para formar l...
 
100 problemas maravillosos de matemáticas - Libro 6
100 problemas maravillosos de matemáticas - Libro 6100 problemas maravillosos de matemáticas - Libro 6
100 problemas maravillosos de matemáticas - Libro 6
 
Taller general sistemas de ecuaciones lineales ejercicios para resolver
Taller general sistemas de ecuaciones lineales ejercicios para resolverTaller general sistemas de ecuaciones lineales ejercicios para resolver
Taller general sistemas de ecuaciones lineales ejercicios para resolver
 
Guía nº ecuaciones 1,2,3 8° básico
Guía  nº ecuaciones 1,2,3 8° básicoGuía  nº ecuaciones 1,2,3 8° básico
Guía nº ecuaciones 1,2,3 8° básico
 
Ecuaciones
EcuacionesEcuaciones
Ecuaciones
 

Similar a 100 problemas maravillosos de matemáticas - Libro 11

100 problemas maravillosos de matemáticas - Libro 5
100 problemas maravillosos de matemáticas - Libro 5100 problemas maravillosos de matemáticas - Libro 5
100 problemas maravillosos de matemáticas - Libro 5
José Mari Melgarejo Lanero
 
Preguntas para la segunda practica
Preguntas para la segunda practicaPreguntas para la segunda practica
Preguntas para la segunda practica
Cesar Dieguez
 
Problemas de ONEM Fase 2
Problemas de ONEM Fase 2Problemas de ONEM Fase 2
Problemas de ONEM Fase 2
Alvaro Miguel Naupay Gusukuma
 
10 problemas de ecuaciones cuadráticas
10 problemas de ecuaciones cuadráticas10 problemas de ecuaciones cuadráticas
10 problemas de ecuaciones cuadráticasCecy Felix
 
SOLUCIONARIO ONEM 2019 F1 N1
SOLUCIONARIO ONEM 2019 F1 N1SOLUCIONARIO ONEM 2019 F1 N1
SOLUCIONARIO ONEM 2019 F1 N1
arquimedesyeuclides
 
Olimpiada matemática
Olimpiada matemáticaOlimpiada matemática
Olimpiada matemática
Wilfrido Valiente Lopez
 
Fórmula para encontrar la suma total de las n combinaciones posibles com m dí...
Fórmula para encontrar la suma total de las n combinaciones posibles com m dí...Fórmula para encontrar la suma total de las n combinaciones posibles com m dí...
Fórmula para encontrar la suma total de las n combinaciones posibles com m dí...
Ernesto Silva
 
Leyes de exponentes whatsmath
Leyes de exponentes whatsmathLeyes de exponentes whatsmath
Leyes de exponentes whatsmath
Victor Jhanpierre Rivera Chavez
 
Longitud de una curva
Longitud de una curvaLongitud de una curva
Longitud de una curva
Luis Alejandro Arocha
 
MATEMÁTICA SEMANA 1
MATEMÁTICA SEMANA 1MATEMÁTICA SEMANA 1
MATEMÁTICA SEMANA 1
JoseFernandez774935
 
Universidad de oriente
Universidad de orienteUniversidad de oriente
Universidad de oriente
Catherin Rengel
 
Aplicaciones
AplicacionesAplicaciones
Aplicaciones
Gicella Véliz Tapia
 
Congruencia
CongruenciaCongruencia
Congruencia
Eve Capi
 
Metodo de simpsons y de los trapecios
Metodo de simpsons y de los trapeciosMetodo de simpsons y de los trapecios
Metodo de simpsons y de los trapeciosFranklin Gualán
 
CIRCUNFERENCIA Y PARÁBOLA.pptx
CIRCUNFERENCIA Y PARÁBOLA.pptxCIRCUNFERENCIA Y PARÁBOLA.pptx
CIRCUNFERENCIA Y PARÁBOLA.pptx
JOHANANALEJANDROSEVI
 
Algebra Pdf
Algebra PdfAlgebra Pdf
Algebra Pdf
sergioflores21
 

Similar a 100 problemas maravillosos de matemáticas - Libro 11 (20)

100 problemas maravillosos de matemáticas - Libro 5
100 problemas maravillosos de matemáticas - Libro 5100 problemas maravillosos de matemáticas - Libro 5
100 problemas maravillosos de matemáticas - Libro 5
 
Preguntas para la segunda practica
Preguntas para la segunda practicaPreguntas para la segunda practica
Preguntas para la segunda practica
 
Problemas de ONEM Fase 2
Problemas de ONEM Fase 2Problemas de ONEM Fase 2
Problemas de ONEM Fase 2
 
Probl geom con_ecuac_soluc
Probl geom con_ecuac_solucProbl geom con_ecuac_soluc
Probl geom con_ecuac_soluc
 
10 problemas de ecuaciones cuadráticas
10 problemas de ecuaciones cuadráticas10 problemas de ecuaciones cuadráticas
10 problemas de ecuaciones cuadráticas
 
SOLUCIONARIO ONEM 2019 F1 N1
SOLUCIONARIO ONEM 2019 F1 N1SOLUCIONARIO ONEM 2019 F1 N1
SOLUCIONARIO ONEM 2019 F1 N1
 
Olimpiada matemática
Olimpiada matemáticaOlimpiada matemática
Olimpiada matemática
 
Fórmula para encontrar la suma total de las n combinaciones posibles com m dí...
Fórmula para encontrar la suma total de las n combinaciones posibles com m dí...Fórmula para encontrar la suma total de las n combinaciones posibles com m dí...
Fórmula para encontrar la suma total de las n combinaciones posibles com m dí...
 
Leyes de exponentes whatsmath
Leyes de exponentes whatsmathLeyes de exponentes whatsmath
Leyes de exponentes whatsmath
 
Longitud de una curva
Longitud de una curvaLongitud de una curva
Longitud de una curva
 
MATEMÁTICA SEMANA 1
MATEMÁTICA SEMANA 1MATEMÁTICA SEMANA 1
MATEMÁTICA SEMANA 1
 
Universidad de oriente
Universidad de orienteUniversidad de oriente
Universidad de oriente
 
Aplicaciones
AplicacionesAplicaciones
Aplicaciones
 
Congruencia
CongruenciaCongruencia
Congruencia
 
Matematicas
Matematicas Matematicas
Matematicas
 
Metodo de simpsons y de los trapecios
Metodo de simpsons y de los trapeciosMetodo de simpsons y de los trapecios
Metodo de simpsons y de los trapecios
 
Fc
FcFc
Fc
 
Circunferencia analitica
Circunferencia analiticaCircunferencia analitica
Circunferencia analitica
 
CIRCUNFERENCIA Y PARÁBOLA.pptx
CIRCUNFERENCIA Y PARÁBOLA.pptxCIRCUNFERENCIA Y PARÁBOLA.pptx
CIRCUNFERENCIA Y PARÁBOLA.pptx
 
Algebra Pdf
Algebra PdfAlgebra Pdf
Algebra Pdf
 

Más de José Mari Melgarejo Lanero

100 problemas maravillosos de matemáticas - Libro 17
100 problemas maravillosos de matemáticas - Libro 17100 problemas maravillosos de matemáticas - Libro 17
100 problemas maravillosos de matemáticas - Libro 17
José Mari Melgarejo Lanero
 
100 problemas maravillosos de matemáticas - Libro 16
100 problemas maravillosos de matemáticas - Libro 16100 problemas maravillosos de matemáticas - Libro 16
100 problemas maravillosos de matemáticas - Libro 16
José Mari Melgarejo Lanero
 
100 problemas maravillosos de matemáticas - Libro 13
100 problemas maravillosos de matemáticas - Libro 13100 problemas maravillosos de matemáticas - Libro 13
100 problemas maravillosos de matemáticas - Libro 13
José Mari Melgarejo Lanero
 
100 problemas maravillosos de matemáticas - Libro 4
100 problemas maravillosos de matemáticas - Libro 4100 problemas maravillosos de matemáticas - Libro 4
100 problemas maravillosos de matemáticas - Libro 4
José Mari Melgarejo Lanero
 
100 problemas maravillosos de matemáticas - Libro 3
100 problemas maravillosos de matemáticas - Libro 3100 problemas maravillosos de matemáticas - Libro 3
100 problemas maravillosos de matemáticas - Libro 3José Mari Melgarejo Lanero
 
100 problemas maravillosos de matemáticas - Libro 2
100 problemas maravillosos de matemáticas - Libro 2100 problemas maravillosos de matemáticas - Libro 2
100 problemas maravillosos de matemáticas - Libro 2José Mari Melgarejo Lanero
 

Más de José Mari Melgarejo Lanero (6)

100 problemas maravillosos de matemáticas - Libro 17
100 problemas maravillosos de matemáticas - Libro 17100 problemas maravillosos de matemáticas - Libro 17
100 problemas maravillosos de matemáticas - Libro 17
 
100 problemas maravillosos de matemáticas - Libro 16
100 problemas maravillosos de matemáticas - Libro 16100 problemas maravillosos de matemáticas - Libro 16
100 problemas maravillosos de matemáticas - Libro 16
 
100 problemas maravillosos de matemáticas - Libro 13
100 problemas maravillosos de matemáticas - Libro 13100 problemas maravillosos de matemáticas - Libro 13
100 problemas maravillosos de matemáticas - Libro 13
 
100 problemas maravillosos de matemáticas - Libro 4
100 problemas maravillosos de matemáticas - Libro 4100 problemas maravillosos de matemáticas - Libro 4
100 problemas maravillosos de matemáticas - Libro 4
 
100 problemas maravillosos de matemáticas - Libro 3
100 problemas maravillosos de matemáticas - Libro 3100 problemas maravillosos de matemáticas - Libro 3
100 problemas maravillosos de matemáticas - Libro 3
 
100 problemas maravillosos de matemáticas - Libro 2
100 problemas maravillosos de matemáticas - Libro 2100 problemas maravillosos de matemáticas - Libro 2
100 problemas maravillosos de matemáticas - Libro 2
 

Último

Semana 10-TSM-del 27 al 31 de mayo 2024.pptx
Semana 10-TSM-del 27 al 31 de mayo 2024.pptxSemana 10-TSM-del 27 al 31 de mayo 2024.pptx
Semana 10-TSM-del 27 al 31 de mayo 2024.pptx
LorenaCovarrubias12
 
evalaución de reforzamiento de cuarto de secundaria de la competencia lee
evalaución de reforzamiento de cuarto de secundaria de la competencia leeevalaución de reforzamiento de cuarto de secundaria de la competencia lee
evalaución de reforzamiento de cuarto de secundaria de la competencia lee
MaribelGaitanRamosRa
 
El fundamento del gobierno de Dios. Lec. 09. docx
El fundamento del gobierno de Dios. Lec. 09. docxEl fundamento del gobierno de Dios. Lec. 09. docx
El fundamento del gobierno de Dios. Lec. 09. docx
Alejandrino Halire Ccahuana
 
Automatización de proceso de producción de la empresa Gloria SA (1).pptx
Automatización de proceso de producción de la empresa Gloria SA (1).pptxAutomatización de proceso de producción de la empresa Gloria SA (1).pptx
Automatización de proceso de producción de la empresa Gloria SA (1).pptx
GallardoJahse
 
CALENDARIZACION DEL MES DE JUNIO - JULIO 24
CALENDARIZACION DEL MES DE JUNIO - JULIO 24CALENDARIZACION DEL MES DE JUNIO - JULIO 24
CALENDARIZACION DEL MES DE JUNIO - JULIO 24
auxsoporte
 
El Liberalismo económico en la sociedad y en el mundo
El Liberalismo económico en la sociedad y en el mundoEl Liberalismo económico en la sociedad y en el mundo
El Liberalismo económico en la sociedad y en el mundo
SandraBenitez52
 
T3-Instrumento de evaluacion_Planificación Analìtica_Actividad con IA.pdf
T3-Instrumento de evaluacion_Planificación Analìtica_Actividad con IA.pdfT3-Instrumento de evaluacion_Planificación Analìtica_Actividad con IA.pdf
T3-Instrumento de evaluacion_Planificación Analìtica_Actividad con IA.pdf
eliecerespinosa
 
Junio 2024 Fotocopiables Ediba actividades
Junio 2024 Fotocopiables Ediba actividadesJunio 2024 Fotocopiables Ediba actividades
Junio 2024 Fotocopiables Ediba actividades
cintiat3400
 
corpus-christi-sesion-de-aprendizaje.pdf
corpus-christi-sesion-de-aprendizaje.pdfcorpus-christi-sesion-de-aprendizaje.pdf
corpus-christi-sesion-de-aprendizaje.pdf
YolandaRodriguezChin
 
PPT: El fundamento del gobierno de Dios.
PPT: El fundamento del gobierno de Dios.PPT: El fundamento del gobierno de Dios.
PPT: El fundamento del gobierno de Dios.
https://gramadal.wordpress.com/
 
SESION ORDENAMOS NÚMEROS EN FORMA ASCENDENTE Y DESCENDENTE 20 DE MAYO.docx
SESION ORDENAMOS NÚMEROS EN FORMA ASCENDENTE Y DESCENDENTE 20 DE MAYO.docxSESION ORDENAMOS NÚMEROS EN FORMA ASCENDENTE Y DESCENDENTE 20 DE MAYO.docx
SESION ORDENAMOS NÚMEROS EN FORMA ASCENDENTE Y DESCENDENTE 20 DE MAYO.docx
QuispeJimenezDyuy
 
True Mother's Speech at THE PENTECOST SERVICE..pdf
True Mother's Speech at THE PENTECOST SERVICE..pdfTrue Mother's Speech at THE PENTECOST SERVICE..pdf
True Mother's Speech at THE PENTECOST SERVICE..pdf
Mercedes Gonzalez
 
Fase 3; Estudio de la Geometría Analítica
Fase 3; Estudio de la Geometría AnalíticaFase 3; Estudio de la Geometría Analítica
Fase 3; Estudio de la Geometría Analítica
YasneidyGonzalez
 
INFORME MINEDU DEL PRIMER SIMULACRO 2024.pdf
INFORME MINEDU DEL PRIMER SIMULACRO 2024.pdfINFORME MINEDU DEL PRIMER SIMULACRO 2024.pdf
INFORME MINEDU DEL PRIMER SIMULACRO 2024.pdf
Alejandrogarciapanta
 
Libro infantil sapo y sepo un año entero pdf
Libro infantil sapo y sepo un año entero pdfLibro infantil sapo y sepo un año entero pdf
Libro infantil sapo y sepo un año entero pdf
danitarb
 
PRÁCTICAS PEDAGOGÍA.pdf_Educación Y Sociedad_AnaFernández
PRÁCTICAS PEDAGOGÍA.pdf_Educación Y Sociedad_AnaFernándezPRÁCTICAS PEDAGOGÍA.pdf_Educación Y Sociedad_AnaFernández
PRÁCTICAS PEDAGOGÍA.pdf_Educación Y Sociedad_AnaFernández
Ruben53283
 
Mapa_Conceptual de los fundamentos de la evaluación educativa
Mapa_Conceptual de los fundamentos de la evaluación educativaMapa_Conceptual de los fundamentos de la evaluación educativa
Mapa_Conceptual de los fundamentos de la evaluación educativa
TatianaVanessaAltami
 
ACERTIJO DE CARRERA OLÍMPICA DE SUMA DE LABERINTOS. Por JAVIER SOLIS NOYOLA
ACERTIJO DE CARRERA OLÍMPICA DE SUMA DE LABERINTOS. Por JAVIER SOLIS NOYOLAACERTIJO DE CARRERA OLÍMPICA DE SUMA DE LABERINTOS. Por JAVIER SOLIS NOYOLA
ACERTIJO DE CARRERA OLÍMPICA DE SUMA DE LABERINTOS. Por JAVIER SOLIS NOYOLA
JAVIER SOLIS NOYOLA
 
3° UNIDAD 3 CUIDAMOS EL AMBIENTE RECICLANDO EN FAMILIA 933623393 PROF YESSENI...
3° UNIDAD 3 CUIDAMOS EL AMBIENTE RECICLANDO EN FAMILIA 933623393 PROF YESSENI...3° UNIDAD 3 CUIDAMOS EL AMBIENTE RECICLANDO EN FAMILIA 933623393 PROF YESSENI...
3° UNIDAD 3 CUIDAMOS EL AMBIENTE RECICLANDO EN FAMILIA 933623393 PROF YESSENI...
rosannatasaycoyactay
 
c3.hu3.p3.p2.Superioridad e inferioridad en la sociedad.pptx
c3.hu3.p3.p2.Superioridad e inferioridad en la sociedad.pptxc3.hu3.p3.p2.Superioridad e inferioridad en la sociedad.pptx
c3.hu3.p3.p2.Superioridad e inferioridad en la sociedad.pptx
Martín Ramírez
 

Último (20)

Semana 10-TSM-del 27 al 31 de mayo 2024.pptx
Semana 10-TSM-del 27 al 31 de mayo 2024.pptxSemana 10-TSM-del 27 al 31 de mayo 2024.pptx
Semana 10-TSM-del 27 al 31 de mayo 2024.pptx
 
evalaución de reforzamiento de cuarto de secundaria de la competencia lee
evalaución de reforzamiento de cuarto de secundaria de la competencia leeevalaución de reforzamiento de cuarto de secundaria de la competencia lee
evalaución de reforzamiento de cuarto de secundaria de la competencia lee
 
El fundamento del gobierno de Dios. Lec. 09. docx
El fundamento del gobierno de Dios. Lec. 09. docxEl fundamento del gobierno de Dios. Lec. 09. docx
El fundamento del gobierno de Dios. Lec. 09. docx
 
Automatización de proceso de producción de la empresa Gloria SA (1).pptx
Automatización de proceso de producción de la empresa Gloria SA (1).pptxAutomatización de proceso de producción de la empresa Gloria SA (1).pptx
Automatización de proceso de producción de la empresa Gloria SA (1).pptx
 
CALENDARIZACION DEL MES DE JUNIO - JULIO 24
CALENDARIZACION DEL MES DE JUNIO - JULIO 24CALENDARIZACION DEL MES DE JUNIO - JULIO 24
CALENDARIZACION DEL MES DE JUNIO - JULIO 24
 
El Liberalismo económico en la sociedad y en el mundo
El Liberalismo económico en la sociedad y en el mundoEl Liberalismo económico en la sociedad y en el mundo
El Liberalismo económico en la sociedad y en el mundo
 
T3-Instrumento de evaluacion_Planificación Analìtica_Actividad con IA.pdf
T3-Instrumento de evaluacion_Planificación Analìtica_Actividad con IA.pdfT3-Instrumento de evaluacion_Planificación Analìtica_Actividad con IA.pdf
T3-Instrumento de evaluacion_Planificación Analìtica_Actividad con IA.pdf
 
Junio 2024 Fotocopiables Ediba actividades
Junio 2024 Fotocopiables Ediba actividadesJunio 2024 Fotocopiables Ediba actividades
Junio 2024 Fotocopiables Ediba actividades
 
corpus-christi-sesion-de-aprendizaje.pdf
corpus-christi-sesion-de-aprendizaje.pdfcorpus-christi-sesion-de-aprendizaje.pdf
corpus-christi-sesion-de-aprendizaje.pdf
 
PPT: El fundamento del gobierno de Dios.
PPT: El fundamento del gobierno de Dios.PPT: El fundamento del gobierno de Dios.
PPT: El fundamento del gobierno de Dios.
 
SESION ORDENAMOS NÚMEROS EN FORMA ASCENDENTE Y DESCENDENTE 20 DE MAYO.docx
SESION ORDENAMOS NÚMEROS EN FORMA ASCENDENTE Y DESCENDENTE 20 DE MAYO.docxSESION ORDENAMOS NÚMEROS EN FORMA ASCENDENTE Y DESCENDENTE 20 DE MAYO.docx
SESION ORDENAMOS NÚMEROS EN FORMA ASCENDENTE Y DESCENDENTE 20 DE MAYO.docx
 
True Mother's Speech at THE PENTECOST SERVICE..pdf
True Mother's Speech at THE PENTECOST SERVICE..pdfTrue Mother's Speech at THE PENTECOST SERVICE..pdf
True Mother's Speech at THE PENTECOST SERVICE..pdf
 
Fase 3; Estudio de la Geometría Analítica
Fase 3; Estudio de la Geometría AnalíticaFase 3; Estudio de la Geometría Analítica
Fase 3; Estudio de la Geometría Analítica
 
INFORME MINEDU DEL PRIMER SIMULACRO 2024.pdf
INFORME MINEDU DEL PRIMER SIMULACRO 2024.pdfINFORME MINEDU DEL PRIMER SIMULACRO 2024.pdf
INFORME MINEDU DEL PRIMER SIMULACRO 2024.pdf
 
Libro infantil sapo y sepo un año entero pdf
Libro infantil sapo y sepo un año entero pdfLibro infantil sapo y sepo un año entero pdf
Libro infantil sapo y sepo un año entero pdf
 
PRÁCTICAS PEDAGOGÍA.pdf_Educación Y Sociedad_AnaFernández
PRÁCTICAS PEDAGOGÍA.pdf_Educación Y Sociedad_AnaFernándezPRÁCTICAS PEDAGOGÍA.pdf_Educación Y Sociedad_AnaFernández
PRÁCTICAS PEDAGOGÍA.pdf_Educación Y Sociedad_AnaFernández
 
Mapa_Conceptual de los fundamentos de la evaluación educativa
Mapa_Conceptual de los fundamentos de la evaluación educativaMapa_Conceptual de los fundamentos de la evaluación educativa
Mapa_Conceptual de los fundamentos de la evaluación educativa
 
ACERTIJO DE CARRERA OLÍMPICA DE SUMA DE LABERINTOS. Por JAVIER SOLIS NOYOLA
ACERTIJO DE CARRERA OLÍMPICA DE SUMA DE LABERINTOS. Por JAVIER SOLIS NOYOLAACERTIJO DE CARRERA OLÍMPICA DE SUMA DE LABERINTOS. Por JAVIER SOLIS NOYOLA
ACERTIJO DE CARRERA OLÍMPICA DE SUMA DE LABERINTOS. Por JAVIER SOLIS NOYOLA
 
3° UNIDAD 3 CUIDAMOS EL AMBIENTE RECICLANDO EN FAMILIA 933623393 PROF YESSENI...
3° UNIDAD 3 CUIDAMOS EL AMBIENTE RECICLANDO EN FAMILIA 933623393 PROF YESSENI...3° UNIDAD 3 CUIDAMOS EL AMBIENTE RECICLANDO EN FAMILIA 933623393 PROF YESSENI...
3° UNIDAD 3 CUIDAMOS EL AMBIENTE RECICLANDO EN FAMILIA 933623393 PROF YESSENI...
 
c3.hu3.p3.p2.Superioridad e inferioridad en la sociedad.pptx
c3.hu3.p3.p2.Superioridad e inferioridad en la sociedad.pptxc3.hu3.p3.p2.Superioridad e inferioridad en la sociedad.pptx
c3.hu3.p3.p2.Superioridad e inferioridad en la sociedad.pptx
 

100 problemas maravillosos de matemáticas - Libro 11

  • 2. El poema “No hay uno que valga” contiene 1000 versos. El autor los ha numerado por orden, absteniéndose de usar los números cuya escritura decimal use uno o más dígitos 1. El primer verso es numerado, así, como 2, el segundo 3, ... de 9 vamos a 20, ..., de 99 a 200 ... ¿Cuál es el número del último verso? SOLUCIÓN Los números válidos para usar son: De 1 a 99 hay que eliminar los que empiezan por 1 y, de los restantes, los que acaban en 1: el 1, los que indican decenas son 10 (10 a 19) y los restantes son 8 ( 21 a 91). En total, 80810199 =−−− De 100 a 199 se eliminan todos pues todos tienen una cifra 1 al menos. De 200 a 299 hay uno más que de 1 a 99 (se cuenta el 200 que no tiene correspondiente en la lista anterior) : 81180 =+ En cada centena, hasta 999, hay también 81, por lo que de 1 a 999: 72881880 =×+ (de 1 a 99 y ocho centenas más). De 1000 a 1999 tienen todos, al menos, una cifra 1, por lo que no cuentan y cada centena, a partir de 2000 , y quitando la centena de 2100 a 2199 , tienen todas 81 números válidos. Por tanto, usando tres centenas, de 1 a 2399 hay 971813728 =×+ a los que deberemos añadir la primera decena válida 2400 a 2409 (todos menos 9:2401 ), la segunda se ignora porque tiene cifra 1 en todos sus números, y con la tercera y la cuarta tenemos que de 1 a 2439 hay 9982797193971 =+=×+ números. Para llegar a mil faltan dos: el que hace 999, 2440 , y el que hace 1000, 2442
  • 3. Halla el cociente entre el área del triángulo equilátero grande PQR y el área del triángulo equilátero pequeño LMN ? SOLUCIÓN Llamamos x al lado del triángulo equilátero pequeño. Como 651 =+ es la longitud del lado del triángulo equilátero grande, la proporción entre las áreas de ambos es la misma que entre los cuadrados de las longitudes de los lados: 2 2 6 x Hallamos la longitud del lado del triángulo pequeño: aplicamos el teorema del coseno en el triángulo LRN y tenemos que 2121526 2 1 10251º60cos51251 222 ==−=×−+=×××−+= xx La proporción pedida es == 21 366 2 2 x 12/7
  • 4. Marta tiene un medallón hecho con 11 hexágonos. Ha hecho un dibujo del medallón y se divierte rodeándolo con otros hexágonos que marca con números desde 1, girando en la dirección opuesta a la de las agujas del reloj. Se da cuenta que los números 1, 20, 45, 76,… se colocan uno encima del otro y se pregunta, ¿cuál será el número que ocupe el lugar 100 en esa serie? Averígualo. SOLUCIÓN Consideramos la sucesión { } { }...,76,45,20,1=na y la sucesión { } { } { } Nnaab nnn ∈∀=−= + ,...,31,25,191 Esta última sucesión es una progresión aritmética de diferencia 6 : Nnnbn ∈∀+= ,613 Como Nnbaaaab nnnnnn ∈∀+=−= ++ ,11 . Podemos escribir entonces +++++++++=+++++           += += += += = −− −− 113322114321 11 334 223 112 1 .........1......... ......... 1 nnn nnn babababaaaaaa baa baa baa baa a ( ) ( ) ( )( ) ( )  −×−×++ += −×+ +=+++++= − − 2 1161319 1 2 1 1.........1 11 1321 nn a nbb abbbba n n nnn ( ) ( ) Nnnna nnnn a nn ∈∀−+= −+ += −×+ += ,12103 2 26206 1 2 1626 1 2 2 Entonces, =−×+×= 12100101003 2 100a 30988
  • 5. La sucesión a1 , a2 , a3 , ... verifica que a1 = 19, a2018 = 99 y para n ≥ 3, an es la media aritmética de los n – 1 primeros términos. ¿Cuál es el valor de a2 ? SOLUCIÓN 191 =a y 992018 =a . Sea xa =2 Según indica el enunciado, 2 19 2 21 3 xaa a + = + = ; 2 19 3 2 19 19 3 321 4 x x x aaa a + = + ++ = ++ = Según se observa, podemos suponer que todos los demás términos de la serie serán iguales. Demostrémoslo por inducción: Suponemos que niNi x ai ≤≤∈∀ + = 3,, 2 19 . Entonces,  + + + ++ + ++ = +++++ = − + n xxx x n aaaaa a nn n 2 19 2 19 ... 2 19 19 ... 1321 1 ( ) ( ) ( ) ( ) ( ) 2 19 2 19 2 1921922 19 219 11 x a n xn n xnx n x nx a nn + = +× = +×−++× = + ×−++ = ++ Hemos demostrado que todos los términos de la serie, a partir del tercero, valen =−===+= + = + 191981981999 2 19 2 19 22018 xax x a x 179
  • 6. Al laberinto adjunto se entra por la [E]ntrada y se sale por la [S]alida. Cuando se llega a una casilla • se puede subir si el número es múltiplo de 3 • se puede bajar si el número es múltiplo de 4 • se puede avanzar hacia la derecha si el número es múltiplo de 5 Da un camino que permita ir desde la [E]ntrada hasta la [S]alida. SOLUCIÓN El camino adecuado es →20→9↑35→12↓16↓24↓25→15→36↑21↑30→
  • 7. Calcula el valor de SOLUCIÓN Si b a m abaabmb a bb mm a log 1 log 1 loglog 1 ===== , por lo que ( ) ( ) ( ) ( ) =++++=++++ 100log...4log3log2log )!100(log 1 ... )!100(log 1 )!100(log 1 )!100(log 1 !100!100!100!100 100432 ( )( ) ( )( )==××××= !100log100...432log !100!100 1
  • 8. Juan tiene el doble de la edad de su hija Carmen, y estamos en el año 2018. En 2029, el mismo día que hoy, tendrán entre los dos un siglo de años. ¿Qué edades tienen ambos ahora mismo? SOLUCIÓN Sea x la edad actual de Juan e y la edad actual de Carmen. Teniendo en cuenta que habrán pasado 1120182029 =− años en la segunda afirmación, el enunciado dice que     =+++ = 1001111 2 yx yx 52262 26 3 78 2 783 2 782 2 7822100 2 =×=     == =     = =     =+ =     =−=+ =  x y yx y yx yy yx yx yx Actualmente, Juan tiene 52 años y Carmen tiene 26 años
  • 9. Con centro en el vértice B del cuadrado ABCD se traza un arco de circunferencia de radio igual a la longitud del lado del cuadrado. Un punto P de dicho arco dista 8 cm del lado AD y 1 cm del lado DC. ¿Cuál es la longitud, en centímetros, del lado del cuadrado? SOLUCIÓN Sea x la longitud, en centímetros, del lado del cuadrado. Construímos el triángulo rectángulo BPC' y pueden verse, en la figura, las longitudes de sus lados en función de x Aplicando el teorema de Piotágoras a dicho triángulo tenemos que ( ) ( ) 0651812641618 2222222 =+−=+−++−=−+− xxxxxxxxxx De lo anterior,    = =  ± = ×−± = 5 13 2 818 2 6541818 2 x x x Desechando, por el contexto del problema, el segundo valor, se deduce que el lado del cuadrado mide 13 centímetros
  • 10. Angelines, Rocío y Tere viven en la misma comarca. En línea recta, las casas de Angelines y Rocío están separadas por un número entero de hectómetros y las casas de Angelines y Tere en un número entero de kilómetros. Además, siempre en línea recta, hay exactamente diez kilómetros más entre la casa de Rocío y la de Tere que entre la casa de Rocío y la de Angelines. Finalmente, el triángulo formado por las casas de las tres vecinas es un triángulo rectángulo. ¿Cuál es, al menos, la distancia entre la casa de Angelines y la de Tere? SOLUCIÓN Llamamos a a la distancia, en hectómetros, entre las casas de Angelines y de Rocío, y llamamos b a la distancia, en kilómetros, entre las casas de Angelines y de Tere, por lo que en hectómetros será b10 Tanto a como b son valores enteros y se pide el menor valor posible de b Pasada a hectómetros, la distancia entre las casas de Rocío y Tere será a+100 En resumen, lo que sabemos es que la distancia entre Rocío y Angelines no es la mayor de las tres. Como las casas forman un triángulo rectángulo, la hipotenusa puede ser cualquiera de los lados que tienen a la casa de Tere como vértice. Vemos los dos casos posibles aplicando el teorema de Pitágoras: 1. Hipotenusa Rocío-Tere: ( ) ( ) +=+++=+ 222222 1002001000010100 baaabaa 1002100210020010000 22 +=+==+ ababba entero positivo. El menor valor posible, siendo también a un valor entero positivo, es 12100222 =+×=b 2. Hipotenusa Angelines-Tere: ( ) ( ) ++=++= 10000200210010010 22222 aabaab 1002 50100 100002002 2 2 2 2 ++= ++ = a a b aa b , debiendo ser a y b valores enteros positivos. Una simple comprobación para valores probables iniciales permite deducir que 121442 >> bb , si existe, pues a debería ser múltiplo de 10 En conclusión, la menor distancia posible entre las casas de Angelines y Tere es 12 kilómetros
  • 11. El rectángulo adjunto está compuesto por dos cuadrados y un rectángulo pequeño. Si el rectángulo pequeño es semejante al rectángulo original y el lado de cada cuadrado mide 1 cm, ¿qué longitud tiene el lado mayor del rectángulo grande? SOLUCIÓN Sea x la longitud, en centímetros, del rectángulo pequeño no igual a la unidad. La longitud que se busca es xx +=++ 211 Como los dos rectángulos son semejantes se verifica que ( ) 01212 1 1 2 2 =−+=×+= + xxxx x x Entonces, 2 222 2 82 2 442 ±− = ±− = +±− =x . Desechamos, según el contexto, la solución negativa de la ecuación por lo que 12 2 222 −= +− = xx y el lado pedido es =−+=+ 1222 x 1 + √2 cm
  • 12. Calcula el valor de la integral SOLUCIÓN Sea ( ) ( ) ( ) −+− − = 8 4 . 3ln9ln 9ln dx xx x I y hacemos el cambio de variable 6+= tx Entonces, dtdx = y los límites son 264600 −=−=−= xt y 268611 =−=−= xt por lo que ( ) ( ) ( ) ( ) ( ) ( ) ]1[. 3ln3ln 3ln . 36ln69ln 69ln 2 2 2 2  −− ++− − = −++−− −− = dt tt t dt tt t I Hacemos ahora el cambio dudtut −=−= y los límites son 200 =−= tu y 211 −=−= tu por lo que ( ) ( ) ( ) ( ) ( ) ( ) ( ) ( ) ( ) ( ) − − − −++ + =− −++ + = ++− − = 2 2 2 2 2 2 ]2[. 3ln3ln 3ln . 3ln3ln 3ln . 3ln3ln 3ln du uu u du uu u dt tt t I Como el nombre de la variable es arbitrario podemos designar, sin perder generalidad, las variables t y u como y en las igualdades ]1[ y ]2[ : ( ) ( ) ( ) ( ) ( ) ( ) ( ) ( ) ( ) ( ) ( ) ( )  −++ + + ++− − =+       −++ + = ++− − =    −− + − − 2 2 2 2 ª2ª1 2 2 2 2 . 3ln3ln 3ln . 3ln3ln 3ln . 3ln3ln 3ln ]2[ . 3ln3ln 3ln ]1[ dy yy y dy yy y II dy yy y I dy yy y I ( ) ( ) ( ) ( ) ] ( ) ==−−=== −++ ++− = −−−  IIydydy yy yy I 4222. 3ln3ln 3ln3ln 2 2 2 2 2 2 2 2
  • 13. Las flotas de la Federación Intergaláctica siempre se componen de naves dispuestas en formación de cuadrados perfectos cuando atacan. El general Black Vader une las flotas A y B para formar una flota C con la que ataca al sistema planetario Nobelius y sufre una fuerte derrota. Solo la última fila sobrevive. Con las naves restantes puede formar una flota D, pero prefiere subdividirla en dos flotas E y F. E y F van al ataque y, de nuevo, solo las últimas filas de E y F escapan de la matanza. Quedan 23 naves. ¿Cuál fue el número inicial de naves de cada flota A y B de Black Vader? SOLUCIÓN Interpretamos las condiciones que nos plantea el problema: Si e y f es la respectiva cantidad de naves por fila de las flotas E y F , la cantidad de naves de ambas flotas es, respectivamente, 2 e y 2 f , siendo 222 fed += el número de naves de la flota D . Los tres valores anteriores deben formar una terna pitagórica con 823 ==+ efe y 15=f , suponiendo que fe < (en caso contrario sería igual) 17= d El cuadrado de este último número, 2891722 === dc , es el número de naves por fila de la flota C : la cantidad de naves de la flota C es ( ) 835212892222 === dc Para finalizar, si las flotas A y B pueden formar la flota C , 2222 28983521===+ cba , siendo ( ) ( ) ( ) 13617828917171715178 2222 =×==×=×+× a y 2551715 =×=b (otra terna pitagórica derivada de la anterior) el número de naves respectivas, por fila, de las flotas A y B . La cantidad de naves de cada una de estas últimas flotas es 1849613622 ==a y 6502525522 ==b 18496 y 65025 naves Otro resultado posible es 2592116122 ==a y 5760024022 ==b , pues también verifican que 2222 28983521===+ cba . La cantidad de naves de las flotas A y B puede ser 25921 y 57600 naves
  • 14. Los puntos A, B y C de la figura dividen a los lados del triángulo MNP en dos partes que están en la relación 1:3 ¿Qué fracción de la superficie del triángulo MNP está coloreada de rojo? SOLUCIÓN Según el enunciado, CPCM ×= 3 , BNBP ×= 3 y 3 3 AN AMAMAN =×= Llamamos WÁreaCBP = y VÁreaABN = Si consideramos los triángulos MBC y CBP y la altura común c , la relación entre sus áreas es =×= × ×= ×× = × = WÁrea cCPcCPcCM Área CBPMBC 33 2 3 2 3 2 WWWÁreaÁreaÁrea CBPMBCMBP 43 =+=+= Y considerando los triángulos MBA y ABN con altura común a , la relación entre sus áreas es =×= × ×= × = × = 33 1 23 1 2 3 2 V Área aAN a AN aAM Área ABNMBA 3 4 3 V V V ÁreaÁreaÁrea ABNMBAMBN =+=+= Por último, los triángulos MBP y MBN , con altura común b , tienen sus áreas relacionadas así: ×= × ×= ×× = × = MBNMBP Área bBNbBNbBP Área 3 2 3 2 3 2 VW V ÁreaWÁrea MBNMBP =×=×== 3 4 334 Entonces, la superficie roja es 3 10 3 3 3 3 WW W V WÁreaÁreaÁrea MBAMBCr =+=+=+= , y la superficie total del triángulo es 3 16 3 4 4 3 4 4 WW W V WÁreaÁreaÁrea MBNMBPt =+=+=+= La razón es == 16 10 3 16 3 10 W W 5/8
  • 15. Dos hermanos, Ángel y Benito, heredaron dos campos cuadrados cada uno, y un lado de cada uno de los cuatro campos forma la orilla del lago cuadrilátero Raum, que tiene dos ángulos opuestos rectos, precisamente en donde los hermanos construyeron sus casas. Por supuesto, ambas superficies heredadas miden lo mismo y las dimensiones de cada campo es, en decámetros, un número entero. Un día Benito sale de su casa corriendo a velocidad siempre constante, entre 15 y 20 km/h, y alcanza el muelle en treinta segundos. Pasa por la casa de Ángel un minuto y medio después de comenzar a correr y da la vuelta completa al lago en cuatro minutos. ¿Cuál es la superficie del lago en áreas? SOLUCIÓN Llamamos x a la velocidad constante de Benito en su recorrido, medida en km/h: 2015 ≤≤ x , y dcba ,,, a las longitudes, en decámetros, de los campos cuadrados según se ve en la figura adjunta. Según el enunciado, al tener las mismas superficies las herencias se verifica que 2222 cbda +=+ , lo cual es coherente con la construcción de dos triángulos rectángulos en el lago con la misma hipotenusa, y la superficie del lago es 222 bcad S bcad S + =+= dam2 La velocidad, en decámetros por minuto, es 3 5 60 100 xx = por lo que, como en medio minuto Benito alcanza el muelle , 6 5 2 1 3 5 x a x a =×= dam [A] En minuto y medio alcanza llega a casa de Ángel , luego 2 5 2 3 3 5 x ba x ba =+×=+ dam. De lo anterior, 3 5 6 10 6 5 2 5 2 5 6 5 xxxx b x b x ba ==−==+=+ dam [B] Por fin, como en cuatro minutos da la vuelta completa, 3 20 4 3 5 x dcba x dcba =+++×=+++ dam, luego 6 25 2 5 3 20 3 20 2 5 x dc xx dc x dc x =+−=+=++ dam Además,       −      =−+      =+      +=+ 22 222 2 2 2 2222 6 5 3 5 3 5 6 5 xx cdc x d x cbda ( ) ( ) ( ) 212 25 6 25 12 25 12 25 36 25 9 25 22222 22 x cd x cd xx cdcd xxx cd =−=−×=−×+=−=− En resumen,      = =              +×=       −×=       =− =+ 6 14 6 11 26 25 2 1 26 25 2 1 2 6 25 x d x c xx d xx c x cd x dc , [C] y [D]
  • 16. Según lo obtenido en [A], [B], [C] y [D], la superficie del lago es  ×+× = + = 2 6 11 3 5 6 14 6 5 2 xxxx bcad S 2 5 72 180 2 36 110 36 70 22 22 xx xx S == + = dam2 Como 6 5x a = y 6 11x c = deben ser valores enteros, siendo 2015 ≤≤ x 18= x , por lo que la superficie del lago es 810 2 185 2 = × =S dam2 = S 810 áreas
  • 17. En el dibujo aparece un cuarto de circunferencia de radio 1 y dos semicircunferencias tangentes. ¿Cuál es el radio de la semicircunferencia pequeña? SOLUCIÓN Según lo señalado en la imagen adjunta, siendo r el valor pedido, se verifica que ( )     −= =+      −= ++=+++       =+       +=      ++ ar rara ar rrrara ar rra 12 2 12 4 1 4 1 2 12 2 1 2 1 2222 22 2 ( ) ( )  +− = − ×−=     − = ×−=−=  2 231 2 1 21 2 1 212 2 2 2 aaa aaa r raarra 3 1 0312312 22 ==−+−= aaaaa Y como ===−=−= 6 2 3 2 3 1 112 rar 1/3 unidades
  • 18. Hace un año, las edades de Rosa y de su madre Luisa tenían las mismas cifras pero con el orden invertido cada una respecto de la otra. Este año sucede lo mismo con las edades de Rosa y de su padre Juan y, curiosamente, la suma de las edades actuales de sus padres es 93. ¿Cuál es la edad actual de Rosa? SOLUCIÓN Llamamos ab a la edad de Rosa. La edad de su padre es, entonces, ba y es lógico pensar que 9<a , 1>b y ab > Además hace un año la edad de Rosa era ( )1−ba y la edad de su madre ab )1( − por lo que este año su madre tiene ( )1)1( +− ab años. Así, ( ) ( ) =+=−+=++++−×=++− 10220293922093101110931)1( baabababbaab baba 10515110 −==+ y, como ambas son cifras, 1510515 =×−== ab Por lo tanto, Juan tiene 51 años, Luisa tiene 42 años y Rosa tiene 15 años
  • 19. ¿Cuáles de las siguientes desigualdades no tienen soluciones reales? SOLUCIÓN La respuesta se encuentra en la representación gráfica de las funciones xy 2= , 2 xy = e x y 2= : Pueden apreciarse cinco zonas en las que se establece el conjunto de valores que cumplen determinadas desigualdades: Zona A. Se cumple la desigualdad 1: 2 22 xx x << , ( )7665,0, −∞−∈∀x Zona B. Se cumple la desigualdad 6 : x xx 22 2 << , ( )0,7665,0−∈∀x Zona C. Se cumple la desigualdad 2 : x xx 222 << , ( )1,0∈∀x Zona D. Se cumple la desigualdad 4 : xx x 222 << , ( )2,1∈∀x Zona E. Se cumple la desigualdad 1: 2 22 xx x << , ( )∞+∈∀ ,2x En resumen, las desigualdades 3 y 5 no tienen soluciones reales
  • 20. Cuatro antiguos piratas, ahora convertidos en comerciantes pacíficos y honorables, se encuentran y conversan. − Cada uno de nosotros tiene un hijo, dice el primero, ¡y todos sueñan con ir al mar! − Sí, pero los hijos de ustedes tres son exactamente de la misma edad, a diferencia del mío, expone el segundo. − Es cierto, y la suma de las edades de los cuatro suman ya 50 años, observa el tercero. − ¡Cómo pasa el tiempo!, hace diez años la misma suma era únicamente 10 años, manifiesta el cuarto. ¿Cuál es la edad actual del hijo del segundo pirata? SOLUCIÓN Sean dcba ,,, las edades de los hijos de los cuatro piratas en el orden mostrado en el enunciado, que tienen, evidentemente, más de 10 años. La conversación se traduce en las siguientes igualdades: ( ) ( ) ( ) ( )      >> − =     >> =+       >> =+++ ==       =−+−+−+− =+++ == 10,10 3 50 10,10 503 10,10 50 1010101010 50 ba b a ba ba ba aaba dca dcba dcba dca         = − == = − == = − ==  11 3 50 ,17 12 3 50 ,14 13 3 50 ,11 b ab b ab b ab , posibilidades admisibles. El hijo del segundo pirata puede tener, actualmente, 11, 14 o 17 años
  • 21. El círculo y el rectángulo de la figura tienen el mismo centro. Si las dimensiones del rectángulo son 6x12 cm y los dos lados pequeños del rectángulo son tangentes al círculo, ¿cuál es el área de la región común (en amarillo) al rectángulo y al círculo? SOLUCIÓN Evidentemente, el radio del círculo mide 6 cm. Marcamos con letras algunos vértices para determinar superficies. El área pedida será el doble de la suma de las áreas del triángulo AOD (de marrón) y el sector circular AOB (de azul), pues es igual a la superficie restante (de amarillo). En el triángulo rectángulo OEA aplicamos el teorema de Pitágoras para obtener EA : ===−=−=−= 33272793636 22222 EAOEOAEA 362 == EAAD cm El área del triángulo AOD es, entonces, 39 2 336 2 = × = ×OEAD cm2 [i] Como el triángulo AOB es equilátero, el ángulo que abarca en el centro del círculo es de º60 , sexta parte del ángulo completo, por lo que el área del sector circular AOB es la sexta parte del área del círculo: π ππ 6 6 6 6 22 == r cm2 [ii] La superficie resultante de sumar las áreas del triángulo AOD y del sector circular AOB es [i]+[ii]: π639 + cm2 y el área pedida es el doble. 18√3 + 12π cm2 = 68,876 cm2
  • 22. Carlos ha dibujado, con puntos, la silueta de un barco en un papel con trama triangular como se ve en la figura adjunta. ¿Cuántos triángulos equiláteros distintos pueden dibujarse con todos los vértices en puntos de la silueta? SOLUCIÓN Los triángulos equiláteros, clasificados según tipos, son por lo que el total es =+++++++ 244413416 38 triángulos equiláteros
  • 23. En un triángulo rectángulo la bisectriz de un ángulo agudo corta al cateto opuesto en dos trozos de longitudes 1 y 2. ¿Cuál es la longitud del segmento de bisectriz interior al triángulo? SOLUCIÓN Nombramos puntos, ángulos y longitudes, y debemos hallar x Es evidente que, en el triángulo BAC , b 3 2tan =α y, en el triángulo DAC , b 1 tan =α Entonces, =− − =       − × = − × = 2 3 3 1 1 2 3 1 1 1 2 3 tan1 tan2 2tan 2 2 22 b bb b bα α α 33233 222 ===− bbbb Aplicando el teorema de Pitágoras en el triángulo rectángulo DAC , ( ) ==+=+=+= xbx 413131 2222 2
  • 24. Se construyen un octógono regular y un hexágono también regular, aunque se han tenido que utilizar lados ligeramente más grandes que los del octógono para que, al superponerlos, coincidan los dos vértices como muestra la figura. ¿Cuál es la amplitud del ángulo x ? SOLUCIÓN Designamos α y β a los ángulos interiores respectivos de un hexágono regular y de un octógono regular y dibujamos diagonales en ambos polígonos para construir cuadriláteros, cuyos ángulos suman siempre º360 Según el cuadrilátero de la derecha, =+++ º360 223 º360 αα α º120º240º120º3602º3602º120 ==−==+ ααα Según el cuadrilátero de la izquierda, =+++ º360 224 º360 ββ β º135º270º90º3602º3602º90 ==−==+ βββ Entonces, en el cuadrilátero central, −==++=+++ º255º3602º360º135º1202º360 xxxx βα == xx º1052 52,5o
  • 25. En el siguiente criptograma cada dígito ha sido reemplazado por una letra. Como siempre a dígitos distintos corresponden letras distintas y, además, ningún número comienza con cero. Halla la diferencia entre el valor más pequeño posible y el mayor valor posible de HUIT. SOLUCIÓN Para el valor más pequeño posible serán 1=D y 2=H . Además, está claro que 9=U o 0=U , y elegimos el segundo por ser el valor más pequeño. Así, tenemos la suma que aparece a la derecha. Como se ve, debe ser 10=+ SE ( 46 + o 37 + ) y 5<X . Si 63 == TX y SE, no podrían tener valores admisibles == 84 TX 53784 =+=+== ISETX (para dar el menor valor). En resumen, la suma es la que se ve a la derecha y el menor valor posible de HUIT es 2058 El valor más grande será para 8=D y 9=H . 9=U o 0=U , y elegimos el segundo por ser tener asignado ya el otro valor. Así, tenemos la suma que aparece a la derecha. Debe ser 10=+ SE ( 46 + o 37 + ), 5<X y T par. El mayor valor posible 7=I hace que 146 =+=+ XSE y 2=T . En resumen, la suma es la que se ve a la derecha y el mayor valor posible de HUIT es 9072 De todo lo anterior deducimos que la diferencia entre los valores menor y mayor de HUIT es =− 20589072 7014
  • 26. ¿Cuántos números hay que colocar en el recinto de color negro? SOLUCIÓN Mostrando la distribución en la imagen derecha se observa que la cantidad de números a colocar en el recinto negro es 5
  • 27. En Spanystán las monedas se identifican fácilmente si uno no es daltónico: 13 monedas lilas valen 17 euros, 13 monedas amarillas son 18 euros y 13 monedas rojas valen 19 euros. Juan sale a la calle con 13 monedas de cada tipo para hacer unas compras. Compra un libro de 16 euros y luego una camiseta por 15 euros. Por cada uno de estos dos gastos, Juan da varias monedas y no se le devuelve ningún cambio. En otra tienda compra una cartera con 2 monedas amarillas y 5 monedas rojas. Para volver a casa coge un taxi, cuyo recorrido le cuesta un número entero de euros. Juan paga con todas las monedas amarillas y rojas que tiene y el taxista le da el cambio en monedas lilas. ¿Cuántas monedas lilas le devuelve el taxista? SOLUCIÓN Al comprar el libro paga con x monedas lilas, y monedas amarillas y z monedas rojas, por lo que, convirtiéndolas en euros,  −− ==++=++ 17 1918208 20819181716 13 19 13 18 13 17 zy xzyx zyx      = = =  −− +−−= 1 2 9 17 24 12 z y x zy zyx , al ser todos los valores, evidentemente, enteros positivos. Le quedan entonces 4913 =− monedas lilas, 11213 =− monedas amarillas y 12113 =− monedas rojas. Luego compra una camiseta con x monedas lilas, y monedas amarillas y z monedas rojas, por lo que, convirtiéndolas en euros,  −− ==++=++ 17 1918195 19519181715 13 19 13 18 13 17 zy xzyx zyx 17 28 11 zy zyx −− +−−= En este caso hay dos posibilidades (en los demás casos rebasa el número de monedas lilas que le quedan): a)      = = = 0 8 3 z y x , y le quedan 134 =− moneda lila, 3811 =− monedas amarillas y 12 monedas rojas. b)      = = = 1 6 4 z y x , y le quedan 044 =− monedas lilas, 5611 =− monedas amarillas y 11112 =− monedas rojas. Por fin, al comprar la cartera le quedan a) 1 moneda lila, 123 =− moneda amarilla y 7512 =− monedas rojas o b) 325 =− monedas amarillas y 6511 =− monedas rojas Analizamos los dos casos al coger el autobús. Sea k el precio, en euros, del billete de autobús y m el número de monedas lilas que le devuelven (ambos valores enteros positivos): a) Al pagar con todas las monedas amarillas y rojas que posee,  × += × + × 13 17 13 719 13 118 m k 92 13 48 11 13 17151 171313318 == − +−= − =+=+ km m mk m kmk
  • 28. b) Al pagar con todas las monedas amarillas y rojas que posee,  × += × + × 13 17 13 619 13 318 m k 102 13 48 12 13 17168 171311454 == − +−= − =+=+ km m mk m kmk En ambos casos, en los que el precio del taxi es 9 o 10 euros, el taxista le devuelve 2 monedas lilas
  • 29. Si el área del cuadrado BESO es 16 cm2 y el área del triángulo SOL es el doble, ¿cuál es, en cm2 , el área del trapecio naranja? SOLUCIÓN Si la superficie del cuadrado BESO es 16 cm2 , esu lado es 416 ==a cm2 Como la superficie del triángulo SOL es 32 cm2 , ( ) ( ) = +× = +× 32 2 44 32 2 haha ( ) 121643242 ==+=+× hhh cm Establecemos ahora una relación de semejanza entre las alturas y las bases de los triángulos SOL y EBL : 3 16 124 4412 12 = × == + = + x x a x ah h cm De lo anterior, la superficie del trapecio naranja es =× + =× + 4 2 43 2 a ax 14 cm2
  • 30. Un aspersor de riego, giratorio, está ubicado en el centro de un jardín rectangular cuyos lados son números enteros de metros estrictamente mayores de 2. Este aspersor, en todo momento, rocía una superficie cuadrada de la cual es una de las esquinas, siendo el lado del cuadrado regado más grande que la mitad de la diagonal del jardín. La porción máxima de jardín, regada en un momento dado (de verde en el dibujo), tiene un área de 1991 m². ¿Cuál es la superficie del jardín? SOLUCIÓN Tomamos x2 y y2 como las medidas del jardín en metros, siendo yx ≤<1 La porción de jardín máxima regada se produce en el pentágono que se muestra en la imagen derecha, compuesto por dos trapecios iguales cuyas bases menor y mayor miden xy − e y metros y la altura x metros. La superficie es ( ) ( ) 1811119912 2 2 ×==×−= ×+− ×= xxy xyxy S m2 22211 == xx m y 1921118118121812 =+=+==− xyxy m Entonces, la superficie del jardín es =×=× 1922222 yx 4224 m2
  • 31. El 25% de los libros de Merche son novelas y 1/9 del total son libros de poesía. Si el número de libros que tiene está entre 50 y 100, ¿cuántos de ellos no son ni novelas ni de poesía? SOLUCIÓN El %25 representa la cuarta parte, por lo que el número de libros de Merche, x , es divisible por 4 Además, 9 1 de los libros son de poesía, por lo que el número de libros también es divisible por 9 En resumen, el número de libros de Merche se puede esxpresar así: nxnx 3694 =××= , siendo Nn∈ Como 72236;21003650 =×==≤=≤ xnnx Tiene, por tanto, 18 4 72 = novelas y 8 9 72 = libros de poesía. El resto es ( )=+− 81872 46 libros
  • 32. La serie se construye así: se eligen y escriben las cifras 8 y 7 y, después, escribimos las cifras de su producto, 56. Después se escriben sucesivamente las cifras del producto de 7 (segundo término) y 5 (tercer término), 35, las cifras del producto de 5 (tercer término) y 6 (cuarto término), 30, … continuando con las cifras (o única cifra) de los productos determinados por dos términos consecutivos. Después de un tiempo, solo se obtienen ceros. ¿Cuál es el último dígito distinto de cero? SOLUCIÓN La secuencia completa es ...8103536578 Desde el primero cero que aparece hasta el siguiente mostramos la secuencia que aparece y las respectivas que se deducen, cada una de la anterior:  ...0555880......05151810...  ...042230......06480......00101010061420......0525204460...  ...022610......021280......062180......023420......08460...  ...02140......06220......061210......08260......042160... ...080......0240..  El último dígito distinto de cero es 8
  • 33. El lado del cuadrado ABCD mide 4 cm. Si el triángulo EAD tiene igual área que el cuadrado, ¿cuál es la distancia del vértice E a la recta determinada por B y C ? SOLUCIÓN Llamamos EFd = a la longitud pedida. Está claro que 4+= hd , siendo h la altura del triángulo EAD Como las áreas de triángulo y cuadrado son iguales, = 2 4 2 4h 842 =×= h cm Por lo tanto, =+=+= 484hd 12 cm
  • 34. Sea ABCD un cuadrilátero tal que el ángulo en C mide 76o y el ángulo en D mide 128o . Se trazan las bisectrices interiores de los ángulos en A y en B que se cortan en P. Halla el ángulo APB. SOLUCIÓN Dibujamos las bisectrices, el punto P y nombramos los ángulos correspondientes. Se pide γ=BPA ˆ La suma de los ángulos de un pentágono es º540º1803 =× pues siempre se puede descomponer en tres triángulos. Entonces, en el pentágono APBCD , se verifica que ( ) º24º128º76º360º540º540º128º76º360 −=−−−=−+=+++−+ γβαβγα En el triángulo APB , γβαγβα −=+=++ º180º180 y sustituyendo en la igualdad anterior se obtiene que ===+=−=−−−=−+ 2 º204 º204º24º1802º24º180º24 γγγγγβα 102o
  • 35. Con los dígitos 1, 2, 3, 4 y 5, escritos en algún orden, formamos el número de cinco cifras abcde. Si el número de tres cifras abc es divisible por 4, el bcd es divisible por 5 y el cde es divisible por 3, ¿qué cifra representa la letra a ? SOLUCIÓN Si bcd es divisible por 55 = d Entonces, si ec5 es divisible por 133 +=+ nec , y c es par porque abc es divisible por 4 2≠c porque, en caso contrario, dec === 52 para que 713 =+=+ nec , lo cual es una contradicción. Por lo tanto, 34 == ec para que 713 =+=+ nec y 2=b pues 4ab es divisible por 4 De todo lo anterior, como ===== aedcb 3,5,4,2 1
  • 36. Sea ABCD un rectángulo y A', B', C' y D' en las prolongaciones de sus lados tales que AA' = k.AD ; BB' = k.AB ; CC' = k.BC ; DD' = k.CD Halla k de modo que el área del cuadrilátero A'B'C'D' sea 25 veces el área del rectángulo ABCD SOLUCIÓN Representamos en la figura las longitudes. Entonces, ×=×+×+= ABCDCDCABAABCDDCBA ÁreaÁreaÁreaÁreaÁrea 2522 '''''''' ( ) ( ) = +× ×+ +× ×+ ab kaakbkbbka ab 25 2 2 2 2 012242224 2222 =−+=+=+++ kkkkababkkababkkab Entonces, 3 2 71 2 71 2 12411 = +− = ±− = ×+±− = kk en el contexto del problema: un valor negativo no tiene sentido. Por lo tanto, =k 3
  • 37. En el triángulo ABC las medianas trazadas desde B y desde C son perpendiculares entre sí. Si AC mide 15 cm y AB mide 10 cm, calcula cuánto mide BC SOLUCIÓN Nombramos, en la figura, los puntos clave y las longitudes. Debemos calcular la longitud BCx = En los triángulos equiláteros que se construyen alrededor del punto en donde se cortan las medianas applicamos el teorema de Pitágoras: En BOC , 222 xba =+ [i] En FOC , 2 22 2 15       =+ da [ii] En BOE , 222 5=+ cb [iii] En FOE , 222 EFdc =+ , pero los triángulos AEF y ABC son semejantes (un ángulo igual y los lados que lo forman son proporcionales en proporción 1 a 2 ) por lo que 2 22 22       =+= x dc x EF [iv] Haciendo ahora [i]-[ii]-[iii]+[iv] tenemos ( ) ( )       +−      −=+++−+−+ 2 2 2 222222222 2 5 2 15 x xdccbdaba ====+=+=      +−      − xxx x x x x 65 5 325 325525 4 225 4 0 2 5 2 15 22 2 2 2 2 2 2 √65 cm = 8,06 cm
  • 38. Se divide un cuadrado de 125 cm2 de área en cinco regiones, cuatro cuadrados y un polígono en forma de L, de rojo en la figura, todas de igual área. ¿Cuál es la longitud, en cm, del lado más corto del polígono en forma de L? SOLUCIÓN Llamamos x a la longitud pedida, en cm, y a a la longitud del lado de los cuadrados, tambien en cm. La superficie de cada región es 25 5 125 = cm2 , por lo que 5252 == aa cm Por otro lado, la región en forma de L , de la misma superficie, se compone de dos rectángulos, por lo que su área será ( ) =+=+=×++× 2520422 22 xxaxxaxxax 1251025101002520 22 ±−=+±−==−+ xxx En el contexto del problema x debe ser positivo, por lo que =−= 10125x 5√5 – 10 cm = 1,18 cm
  • 39. Halla la suma de los ángulos de los vértices A, B, C, D y E de la estrella de la figura. SOLUCIÓN Sean edcba ,,,, los ángulos interiores del pentágono central. Está claro que º540º1803 =×=++++ edcba , suma de los ángulos de un pentágono cualquiera. Observemos que • en el triángulo de vértice A , dcAdcA +=+=−+−+ º180ˆº180º180º180ˆ • en el triángulo de vértice B , edBedB +=+=−+−+ º180ˆº180º180º180ˆ • en el triángulo de vértice C , aeCaeC +=+=−+−+ º180ˆº180º180º180ˆ • en el triángulo de vértice D , baDbaD +=+=−+−+ º180ˆº180º180º180ˆ • en el triángulo de vértice E , cbEcbE +=+=−+−+ º180ˆº180º180º180ˆ Sumando ashora miembro a miembro las cinco igualdades, ( ) =×=+++++++++×=×+++++ º1080º5402º900ˆˆˆˆˆ2º1805ˆˆˆˆˆ EDCBAedcbaEDCBA =−=++++ º900º1080ˆˆˆˆˆ EDCBA 180o
  • 40. Escribe en cada casilla de la pirámide un número natural mayor que 1 de modo que • la casilla superior tenga escrito el número 560105280 • el número escrito en cada casilla sea igual al producto de los números escritos en las dos casillas sobre las que está apoyada. SOLUCIÓN Como 466 7532560105280 ×××= , vamos a intentar rellenarla de abajo hasta arriba con los divisores más elementales. Ponemos 5 en una esquina, al ser el factor que menos debe intervenir y 3 en la casilla central. Como hay que repetir uno de los factores elegimos el 2 para que ocupe un ala completa (dos casillas seguidas). Así, rellenamos de abajo hacia arriba la pirámide y queda Que no determina, en la casilla superior, el número indicado, pero que da la pista fundamental para obtener una pirámide (no la úníca) que se busca. Colocamos un 2 2 en vez de un 2 en la primjera casilla de abajo y obtenemos que sí cumple las condiciones del problema. Una solución es, pues,
  • 41. Todos los estudiantes de una clase hicieron una prueba. Cinco de ellos obtuvieron la puntuación máxima, 100 puntos, ninguno obtuvo menos de 60 puntos y la media de la clase fue de 76 puntos. ¿Cuántos estudiantes, como poco, había en la clase? SOLUCIÓN Sea n el número de estudiantes que no obtuvieron 100 puntos. El total de estudiantes pedido es 5+n Si el promedio de puntos de esos n estudiantes fue de ( )60>x , la media total fue = + +× 76 5 1005 n nx ( ) x nnxnnx − ==×−+=+ 76 120 1207638076500 A menor valor de ( )60>x corresponde menor valor de n , siempre que ambos valores sean enteros. Esto corresponde a 8 15 120 6176 120 6061 == − =>= nx , que corresponde a =+=+ 585n 13 estudiantes
  • 42. Quique intercambia los dígitos de un número de 3 cifras distintas, y distintas de cero, de modo que ningún dígito queda en su posición original. Después resta el número viejo menos el nuevo y el resultado es un número de 2 cifras que es un cuadrado perfecto. Halla todos los resultados que puede obtener Quique. SOLUCIÓN Sea el número abc con las tres cifras distintas entre sí. Se pueden dar dos casos: 1. Volteamos el número obteniendo bca y restamos: 2 1010010100 Aacbcbabcaabc =−−−++=− , siendo ba > y 2 A un número entero de dos cifras. De lo anterior, ( ) 3691011999099 222 ===−−×=−− • AAAcbacba o 812 =A a) Si ( ) 41110410113610119362 −=+=−−=−−×= acbcbacbaA y se cumple para los valores • 8,118422102 ===−=+= cbcba • 9,229433103 ===−=+= cbcba b) Si ( ) 91110910118110119812 −=+=−−=−−×= acbcbacbaA y se cumple para los valores • 3,113922102 ===−=+= cbcba • 4,224933103 ===−=+= cbcba • 5,335944104 ===−=+= cbcba • 6,446955105 ===−=+= cbcba • 7,557966106 ===−=+= cbcba • 8,668977107 ===−=+= cbcba • 9,779988108 ===−=+= cbcba 2. Volteamos el número obteniendo cab y restamos: 2 1010010100 Abaccbacababc =−−−++=− , siendo ca > y 2 A un número entero de dos cifras. De lo anterior, ( ) 3691110999990 222 ===−+×=−+ • AAAcbacba o 812 =A a) Si ( ) 41110411103611109362 +=+=−+=−+×= cbacbacbaA y se cumple para los valores • 1,881477107 ===+=+= babac • 2,992488108 ===+=+= babac b) Si ( ) 91110911108111109812 +=+=−+=−+×= cbacbacbaA y se cumple para los valores • 1,331922102 ===+=+= babac • 2,442933103 ===+=+= babac
  • 43. • 3,553944104 ===+=+= babac • 4,664955105 ===+=+= babac • 5,775966106 ===+=+= babac • 6,886977107 ===+=+= babac • 7,997988108 ===+=+= babac Entonces, todos los resultados que puede obtener Quique son: 218 – 182 = 36; 329 – 293 = 36; 817 – 781 = 36; 928 – 892 = 36; 213 – 132 = 81; 324 – 243 = 81; 435 – 354 = 81; 546 – 465 = 81; 657 – 576 = 81; 768 – 687 = 81; 879 – 798 = 81; 312 – 231 = 81; 423 – 342 = 81; 534 – 453 = 81; 645 – 564 = 81; 756 – 675 = 81; 867 – 786 = 81; 978 – 897 = 81 18 resultados
  • 44. Los puntos A, B, C y D, en este orden, determinan un cuadrilátero inscrito en una circunferencia. Los lados AB y CD son paralelos, el ángulo ADC = 50o y los ángulos BAC y BCA son iguales. ¿Cuánto mide el ángulo DAC ? SOLUCIÓN Como AB y CD son paralelos, el cuadrilátero inscrito en la circunferencia debe ser un trapecio isósceles. Por lo tanto, === º50ˆˆº50ˆ CDABCDCDA º130 2 º50º50º360ˆˆ = −− === αDABABC Como == βACBCAB ˆˆ el triángulo ABC es isósceles y −= αβ º1802 º25 2 º50 2 º130º180 2 º180 == − = − = β α β Ahora bien, =−=−==+ º25º130βαγαγβ 105o
  • 45. La familia Álvarez, la familia Benítez y el matrimonio Cáceres almorzaron juntos. Los Álvarez, que comieron 3 chuletones, 2 ensaladas y 5 gaseosas, gastaron 85 euros, los Benítez, que comieron 5 chuletones, 3 ensaladas y 9 gaseosas, gastaron 142 euros. ¿Cuánto gastaron los Cáceres que comieron, entre los dos, 1 chuletón, 1 ensalada y 1 gaseosa? SOLUCIÓN Llamamos zyx ,, a los precios unitarios respectivos, en euros, de chuletón, ensalada y gaseosa. Según el enunciado, ( ) ( ) =++−=++−++    =++ =++ −× zyxzyxzyx zyx zyx 1421709351046 142935 85523 ª2ª12 28 euros
  • 46. Si Presen se sube en una mesa y Roque se queda en el suelo, Presen es 80 cm más alta que Roque. Si Roque se sube a la mesa y Presen se queda en el suelo entonces Roque es un metro más alto que Presen. ¿Cuál es, en centímetros, la altura de la mesa? SOLUCIÓN Tomamos yx, como las alturas respectivas, en centímetros, de Roque y Presen y z la altura, en centímetros, de la mesa. Según el enunciado tenemos que ( ) 1020220 100 80 ª1ª2 =−=−×+−=−    +=+ +=+ − yxyxxyyx yzx xzy De la primera condición, =+=+−=+=+ 80108080 yxzxzy 90 centímetros
  • 47. Halla el menor número natural que es suma de 9 naturales consecutivos, es suma de 10 naturales consecutivos y además es suma de 11 naturales consecutivos. SOLUCIÓN Según el enunciado, el número buscado es = × = × = × , siendo , , ∈ N De lo anterior, = 9 + 36 = 10 + 45 = 11 + 55 Entonces, !" !" "" # ⇒ % % # ⇒ % # ⇒ &'()* * ( * &'(+&' && , Como , , ∈ N ⇒ - % . #⇒ - . # ⇒ - &&/)&' &' 0, siendo 1, 2 ∈ N Por lo tanto, . = 91 ⇒ 112 + 10 = 901 ⇒ 2 = -% ⇒ 2 = 81 + -% El valor natural más pequeño de 1 que hace a 2 natural es 1 = 5 ⇒ 2 = 8 × 5 + ×"% = 40 De ahí, el valor más pequeño de ⇒ = 91 = 9 × 5 = 45 ⇒ = 10 + 45 = 10 × 45 + 45 = 495
  • 48. En la figura se observa un triángulo isósceles dividido en cuatro regiones de las que conocemos las áreas de los tres triángulos: 3, 3 y 6 cm2 . Si M y N son los puntos medios de los lados iguales, ¿cuál es el área del cuadrilátero verde? SOLUCIÓN Señalámos los vértices del triángulo grande y su altura. Por semejanza, la altura del triángulo ABC es el doble de la del triángulo BMC y, como ambos triángulos tienen la misma base, el área del triángulo ABC es el doble de la del área del triángulo BMC : ( ) 186322 =+×=×= BMCABC ÁreaÁrea unidades cuadradas Entonces, la superficie del cuadrilátero verde es la superficie del triángulo ABC menos las superficies de los tres triángulos interiores: =−−− 63318 6 unidades cuadradas
  • 49. En un hotel de Menorca hay 120 personas distribuidas entre la recepción, el bar, el comedor y el salón de reuniones. La cantidad de personas que hay en el bar es un quinto de la que hay en el comedor, y en la recepción hay un octavo de las que hay en el salón. Al pasar diez personas del comedor al salón y seis del bar a la recepción, en la recepción hay un sexto de las que quedan en el comedor. ¿Cuántas personas había inicialmente en cada uno de los lugares mencionados del hotel? SOLUCIÓN Llamamos dcba ,,, a la cantidad de personas que hay, respectivamente, en la recepción, en el bar, en el comedor y en el salón de reuniones. Según el enunciado, 120=+++ dcba , 5 c b = , 8 d a = Al moverse las personas como indica el enunciado, 6 46 6 6 10 6 10 6 − =− − = − =+ cc a c a Además, 3 1844 6 46 88 − = − == c d cdd a y, sustituyendo en la primera ecuación, se tiene que = −+++− = − +++ − =+++ 120 30 1840403062305 120 3 1844 56 46 120 ccccc c cc dcba 70 81 5670 5670813600207081 ====− cccc De lo anterior, 4 6 4670 6 46 = − = − = a c a ; 14 5 70 5 === b c b ; 32 3 184704 3 1844 = −× = − = d c d En resumen hay 4 personas en la recepción 14 personas en el bar 70 personas en el comedor 32 personas en el salón de reuniones
  • 50. “¿Qué edad tienes?” le preguntan a Joaquín, y contesta: “Si yo viviera 100 años, mi edad actual sería los cuatro tercios de la mitad de lo que me quedaría por vivir”. ¿Cuál es la edad de Joaquín? SOLUCIÓN Sea x la edad actual de Joaquín. Si viviera 100 años le quedarían por vivir x−100 años. Entonces, según lo que contesta Joaquín, ( ) −= − =−××= xx x xxx 22003 3 2200 100 2 1 3 4 === 5 200 2005 xx 40 años
  • 51. Halla todos los números naturales de dos cifras tales que al elevarlos al cubo se obtienen números que terminan en dos cifras iguales. SOLUCIÓN Sea xy uno de esos números. Como ( ) ( ) 322333 30300100010 yxyyxxyxxy +++=+= , los sumandos que determinan las dos últimas cifras son 32 30 yxy + Analizamos los casos: xyxyy ∀=+= ,0300 32 cifra, por lo que todos los números de dos cifras acabados en cero cumplen la condición: 90,80,70,60,50,40,30,20,10 130301 32 +=+= xyxyy debe acabar en 711 = x para que 211130 =+x . Sólo hay un número cumpliendo la condición: 71 8120302 32 +=+= xyxyy debe acabar en    =+= =+=  10888120;9 4888120;4 88 xx xx . Hay dos números cumpliendo la condición: 92,42 27270303 32 +=+= xyxyy debe acabar en 137727270;577 =+= xx . Hay un número cumpliendo la condición: 53 64480304 32 +=+= xyxyy debe acabar en    =+= =+=  294464480;6 54464480;1 44 xx xx . Hay dos números cumpliendo la condición: 64,14 125750305 32 +=+= xyxyy debe acabar en 55: imposible 2161080306 32 +=+= xyxyy debe acabar en 66 : imposible 3431470307 32 +=+= xyxyy debe acabar en 106333431470;733 =+= xx . Hay un número cumpliendo la condición: 77 5121920308 32 +=+= xyxyy debe acabar en 22 : imposible 7292430309 32 +=+= xyxyy debe acabar en 225997292430;999 =+= xx . Hay un número cumpliendo la condición: 99 Concluyendo, hay 17 números cumpliendo las condiciones del problema: 10, 14, 20, 30, 40, 42, 50, 53, 60, 64, 70, 71, 77, 80, 90, 92, 99
  • 52. En la figura adjunta los segmentos AB, FC y ED son paralelos. Si la longitud del segmento AB es 10 y la del ED es 7, ¿cuál es la longitud del segmento FC ? SOLUCIÓN Como los triángulos AED y AFC son semejantes, 7 7 FCAE AF AF AE FCAF AE FC ED × === Como los triángulos ABE y FCE son semejantes, 10 10 FCAE FE FE AE FCFE AE FC AB × === Sumando las dos expresiones, AE FCAEAE FCAEFCAE FEAF ÷       +××= × + × =+ 10 1 7 1 107 ×= ÷ 70 17 1 FC AE FC = 70/17
  • 53. Para hacer un castillo de naipes de 1 piso se usan 2 naipes, para hacerlo de 2 pisos se usan 7 naipes, para hacerlo de 3 pisos se usan 15 naipes… ¿Cuántos naipes hay que usar para hacer un castillo de 100 pisos? SOLUCIÓN El problema propone una sucesión na tal que 21 =a , 72 =a , 153 =a , … donde na es la cantidad de naipes necesarios para completar un castillo de n pisos. Experimentalmente puede verse que 264 =a , 405 =a , …, siempre un incremento de 3 naipes en la diferencia con el castillo anterior. Construímos la sucesión :1 nnn aab −= + ( ) 13213 ............ 243142640 233111526 2238715 213527 11 454 343 232 121 −=+−×=−=         +×==−=−= +×==−=−= +×==−=−= +×==−=−= −− nnaab aab aab aab aab nnn Se puede observar que esta última sucesión es una progresión aritmética de diferencia 3 y, sumando sus 1−n primeros términos, tenemos que ( ) ( ) ( ) ( ) ( ) ( ) ( ) ( ) ( ) ( )      −×+ = −×−+ = −×+ =++++ −=−=−++−+−+−=++++ − − −− 2 143 2 1135 2 1 ...... 2............ 11 1321 113423121321 nnnnnbb bbbb aaaaaaaaaaabbbb n n nnnnn ( ) ( ) ( ) ( ) 2 3 2 443 2 2 143 2 143 2 22 nn a nnnn a nn a nnn + = +−+ =+ −×+ = −×+ =− Por lo tanto, el número de naipes necesarios para construir un castillo de 100 pisos es = +× = 2 1001003 2 100a 15050 naipes
  • 54. Sabiendo que calcula el valor de SOLUCIÓN 1 + + + + … = 2 ⇒ + + + … = 2 − 1= 1 Buscamos el valor de = + 2 + 3 + 4 + … = + + + + 2 + + 3 + … ⇒ ⇒ = + + + + … + + 2 + 3 + … = 1 + × + 2 + 3 + … = 1 + ⇒ ⇒ − = 1 ⇒ 1 − × = 1 ⇒ = Como 1 + + + + … es la suma de los términos de una progresión geométrica de razón ∋ | | < 1, pues de lo contrario la suma no sería un valor real, 1 + + + + … = = = 2 ⇒ = = 2
  • 55. Calcula, en la figura adjunta, el área de la forma de color rojo. SOLUCIÓN Para referenciar todas las formas que aparecen en las operaciones, señalamos los 12 vértices con letras consecutivas del alfabeto. Calculamos, en principio, la superficie de toda la forma señalada de amarillo. Dicha superficie es igual a la superficie del sector circular , de radio 12, menos la superficie del sector circular , de radio 8, y menos la superficie del cuadrado , de lado 4: = × − × − 4 = 20 − 16 La superficie de la forma azul es igual a la superficie del cuadrado , de lado 4, menos la superficie del sector circular , de radio 4: !" = 4 − × = 16 − 4 Hallamos ahora la superficie señalada de verde. Es igual a a la superficie del cuadrado # , de lado 8, menos la superficie del sector circular # , de radio 8: $% &% = 8 − × 8 4 = 64 − 16 La superficie buscada es '( = + !" − $% &% = *20 − 16+ + *16 − 4 + − *64 − 16 + = 32π – 64 = 36,53 unidades cuadradas de superficie
  • 56. Se inscribe una semicircunferencia en un triángulo isósceles de base 16 cm y altura 15 cm, como muestra la figura. ¿Cuál es su radio? SOLUCIÓN Se observa en la imagen derecha que hay tres triángulos rectángulos en los que aplicamos el teorema de Pitágoras: 22222222 6488 ararra −=−=+= 1728964225815 222 ==+=+= xx cm ( ) =+−=−++−+−= = 225643428922564215 17 222222 aaaaxxrax x 17 64 34 128 12834 === aa cm De ahí, =×=×=      −×=      −=−= 17 15 8 289 225 64 289 64 164 17 64 6464 2 2 22 rrar 120/17 = 7,0588 cm
  • 57. En una circunferencia de centro O, AB es un diámetro y P un punto de AB que dista 9 cm. de O. Se trazan dos cuerdas perpendiculares a AB que miden 18 cm y 14 cm respectivamente, dejan a O entre ambas y distan 8 cm entre sí. Calcula la medida de la cuerda paralela a las otras que pasa por P. SOLUCIÓN Intentamos calcular el radio de la circunferencia según las condiciones del segundo párrafo. Siendo r el radio de la circunferencia, aplicamos el teorema de la altura en los triángulos rectángulos ADB y AEB : ( ) ( )     =+ +==−     =+ =−     =+ =+     =×+ =+× 498 44 498 3288 498 818 78 98 2 2 bab baba bab ba bab aba ba ba ( ) 68549366049124984 2 −=++−==−+=+×+ bbbbbbb cm De ahí, 28568544 −=−+=+= aba cm Entonces, 85 2 8685285 2 8 = +−+− = ++ = r ba r cm Mostramos ahora la cuerda paralela a las anteriores. Su longitud, x2 , es el doble de la altura del triángulo rectángulo AFB . Aplicamos en dicho triángulo en teorema de la altura y tenemos que ( ) ( ) 24985985985 22 ==−=−×+= xx cm Por lo tanto, =x2 4 cm
  • 58. Halla el resto de la división SOLUCIÓN Tengamos en cuenta que, aplicando la regla de Ruffini, ( ) ( )21232 −×−=+− xxxx y que ( ) ( ) ( ) ( ) ( ) ( ) RuffiniR xxxxxxxxx . 99999999100 422142114242 +−+−×−=+−×+−=+−×=+− ( ) ( ) ( ) ++−×−×++++++=+− 2211......42 296979899100 . xxxxxxxxxx RuffiniR ( ) ( ) 2231......42 2296979899100 +++−×++++++=+− xxxxxxxxxx , luego el resto solicitado es x + 2
  • 59. ¿Cuántos números naturales de 4 cifras terminan en 36 y son múltiplos de 36? SOLUCIÓN Para que sean múltiplos de 9436 ×= deben ser múltiplos de 4 y de 9 Todos los números acabados en 36 son múltiplos de 4 pues el número formado por sus dos últimas cifras ( )36 es múltiplo de 4 Para que sean, también, múltiplos de 9 la suma de sus cifras debe ser múltiplo de 9: si el número es •• =+=++ 993636 babaab , siendo 0≠a Esto se cumple en estos casos: 9936,9036,8136,7236,6336,5436,4536,3636,2736,1836 10 números
  • 60. Halla el valor exacto de sin usar calculadora. SOLUCIÓN Llamamos = 2 + √5 + 2 − √5 ⇒ = 2 + √5 + 2 − √5 ⇒ ⇒ = 2 + √5 + 3 × 2 + √5 × 2 − √5 + 3 × 2 + √5 × 2 − √5 + 2 − √5 Ahora bien, como 3 × 2 + √5 × 2 − √5 = 3 × 2 + √5 × 2 + √5 × 2 − √5 = = 3 × 2 + √5 × 2 + √5 × 2 − √5 = 3 × 2 + √5 × √−1 = −3 × 2 + √5 y 3 × 2 + √5 × 2 − √5 = 3 × 2 + √5 × 2 − √5 × 2 − √5 = = 3 × 2 + √5 × 2 − √5 × 2 − √5 = 3 × √−1 × 2 − √5 = −3 × 2 − √5 , se sigue que = 2 + √5 − 3 × 2 + √5 − 3 × 2 − √5 + 2 − √5 = 4 − 3 × 2 + √5 + 2 − √5 ⇒ ⇒ = 4 − 3 ⇒ + 3 − 4 = 0 Usando la Regla de Ruffini, 1 0 3 −4 1 ) 1 1 4 1 1 4 ( 0 ⇒ + 3 − 4 = ( − 1) × ( + + 4) = 0 Como + + 4 = 0 no posee soluciones reales pues = ±√ , la única solución real se obtiene de − 1 = 0 ⇒ = 1
  • 61. Empezando con 46 se forma una secuencia de dígitos colocando en cada paso, a continuación del ultimo número escrito, el producto de los dos últimos dígitos que se escribieron. Los primeros dígitos pueden verse en la imagen adjunta. Calcula el dígito que está en la posición 1089 SOLUCIÓN La secuencia es 462483261224832612 … y puede observarse que, a partir del tercer dígito de la secuencia, se repite la serie, de 8 dígitos, 24832612: hay dos dígitos iniciales y, posteriormente, series iguales de 8 dígitos. Entonces, como 1089 = 136 × 8 + 1 = 2 + 135 × 8 + 7, el dígito correspondiente a esa posición será el séptimo de la serie que se repite: 1
  • 62. La gráfica de la función f está compuesta por tres trozos rectilíneos, como muestra la figura. ¿Cuántas soluciones tiene la ecuación f ( f ( f ( x ) ) ) = 0 ? SOLUCIÓN La función está compuesta de tres tramos rectos. Los dos extremos de pendiente 1 y el del medio de pendiente −1 Como pasa por los puntos −4 , 0 y 0 , 0 , su expresión analítica es = + 4, < −2 – , − 2 ≤ < 0 , ≥ 0 Como −4 = 0 = 0 , = 0 ⇒ ⎩ ⎪ ⎨ ⎪ ⎧ = −4 ⇒ = −8 ⇒ = −12 = 0 ⇒ = −4 ⇒ = −8 = 0 ⇒ = −4 = 0 La ecuación planteada posee 4 soluciones
  • 63. Sea t una recta y P un punto exterior. Sobre la recta se marcan de izquierda a derecha los puntos A, B, C, D, E de modo que PA=PB, PB=BC, PC=CD y PD=DE. Se traza por P la paralela a t y se marca en esta paralela el punto Q tal que PQED es un paralelogramo. Si los ángulos QED y APB son iguales, ¿cuánto mide el ángulo BAP? SOLUCIÓN Construimos el esquema que nos plantea el enunciado: Llamamos = = y seguimos el siguiente razonamiento: Como es un paralelogramo, = = es un triángulo isósceles: = = ⇒ = 180° − − = 180° − 2 ⇒ = 2 es un triángulo isósceles: = = 2 ⇒ = 180° − − = 180° − 4 ⇒ = 4 es un triángulo isósceles: = = 4 ⇒ = 180° − − = 180° − 8 , pero = ⇒ = 180° − 8 ⇒ 9 = 180° ⇒ = 20° ⇒ = 4 = 80o
  • 64. Si f es una función periódica de periodo T = 5 y en el intervalo [3 , 8) verifica que ¿cuál es el valor de f (2019) ? SOLUCIÓN + 5 = , y como 2019 = 5 × 403 + 4 ⇒ 2019 = 4 + 5 × 403 = 4 ⇒ 2019 = 4 = 4 − 10 × 4 + 25 = 1
  • 65. El druida Panoramix desea preparar 24 cucharones de una pócima mágica que contenga las sustancias A, B, C por partes iguales. Dispone de un recipiente donde hay A y C mezclados por partes iguales; otro en el que hay A y B mezclados en la proporción 2:3 y un tercero en el que hay B y C mezclados en la proporción 1:2. ¿Cuántos cucharones de cada recipiente debe mezclar para obtener la pócima deseada? SOLUCIÓN Sean , , los cucharones que tienen que contribuir, de cada recipiente, a la mezcla deseada. Está claro, entonces, que + + = 24 Los cucharones del primer recipiente aportarán de A y de C, medidos en “unidades de cucharón”. Los cucharones del segundo recipiente aportarán de A y de B, medidos “en unidades de cucharón”. Los cucharones del tercer recipiente aportarán de B y de C, medidos en “unidades de cucharón”. Como las tres sustancias deben ser aportadas en la misma cantidad, deberá ser + = + = + ⇒ ⇒ 15 + 12 = 18 + 10 = 15 + 20 tomando denominador común. Se obtiene el sistema + + = 24 15 + 12 = 18 + 10 18 + 10 = 15 + 20 ⇒ + + = 24 15 − 6 − 10 = 0 15 − 18 + 10 = 0 ª→ ª ª ª→ ª ª ! + + = 24 30 − 24 = 0 12 − 20 = 0 ⇒ ⇒ + + = 24 5 − 4 = 0 3 − 5 = 0 ⇒ # + + = 24 = $ = ⇒ ⎩ ⎪ ⎨ ⎪ ⎧ $ + + = 24⇒ ) = 24 ⇒ = 10 = $ ⇒ = $×)+ ⇒ = 8 = ⇒ = ×)+ ⇒ = 6 Se obtienen así + = + = + = 8 “unidades de cucharón” de cada sustancia A, B y C mezclando 8, 10 y 6 cucharones de cada recipiente, siguiendo el orden
  • 66. Halla todos los pares de enteros (x, y), con x ≤ y, que verifican que su producto es igual a cinco veces su suma. SOLUCIÓN Sea × = 5 × + . Entonces, ∃ ∈ ∋ = 5 . Elegimos esta incógnita porque los resultados serán semejantes con la elección de la otra. × = 5 × + ⇒ 5 = 5 × + 5 ⇒ = + 5 ⇒ × − 1 = 5 ⇒ = 5 − 1 ∈ Evidentemente solo puede ser par: si fuera impar, − 1 (par) no podría dividir a 5 , producto de dos impares. Además, dada la estructura de la fracción, debe ser − 1 = ±1 o − 1 = ±5 Los únicos casos posibles son, entonces, los siguientes: • = 0 ⇒ = 0, = 0 • = 2 ⇒ = 10, = 10 • = 6 ⇒ = 6, = 30 • = −4 ⇒ = 4, = −20. Puede obtenerse el correspondiente a este último par haciendo la otra elección inicial para que sea válido ≤ , por lo que los pares son (0 , 0); (10 , 10); (6 , 30); (-20 , 4)
  • 67. Escribe una lista de cinco números naturales distintos y mayores que 1 de manera que cada número sea múltiplo del anterior y que la suma de los cinco números sea 517. SOLUCIÓN Los números serán , × , × × , × × × , × × × × tales que > 1 , , , , ∈ Además, + × + × × + × × × + × × × × = 517 = 11 × 47 ⇒ ⇒ × 1 + + × + × × + × × × = 11 × 47 Tomamos = 11 ⇒ 1 + + × + × × + × × × = 47 ⇒ ⇒ + × + × × + × × × = 46 ⇒ × 1 + + × + × × = 46 = 2 × 23 Tomamos = 2 ⇒ 1 + + × + × × = 23 ⇒ + × + × × = 22 ⇒ ⇒ × 1 + + × = 22 = 2 × 11 Tomamos = 2 ⇒ 1 + + × = 11 ⇒ + × = 10 ⇒ × 1 + = 10 = 2 × 5, y tomamos = 2, = 4 De esta manera, los números son = 11, × = 11 × 2 = 22, × × = 22 × 2 = 44, × × × = 44 × 2 = 88, × × × × = 88 × 4 = 352 tales que 11 + 22 + 44 + 88 + 352 = 517 La posibilidad de que fuera = 47 no permitiría llegar a ninguna lista válida, por lo que los números son 11, 22, 44, 88, 352
  • 68. Si f es la función, definida en el intervalo (0, 1) y 0 < α < π/2, tal que halla el valor de f (tg2 α) SOLUCIÓN Hacemos = Entonces, = = ⇒ − = ⇒ + = ⇒ 1 + × = ⇒ = ⇒ = De ahí, = = = √ = = ! "# ! ! "# ! = ! "# ! "# !$ ! "# ! = ! "# ! % "# ! = √&'( = sen α
  • 69. Una hormiga sale de su hormiguero y recorre en línea recta un tramo de x cm, luego gira 90o y recorre en línea recta otro tramo de x/2 cm, luego vuelve a girar 90o y recorre un tramo de x/22 cm y, así sucesivamente, siempre recorriendo un tramo de longitud igual a la mitad del anterior. El sentido en que gira lo decide en cada vértice. ¿Cuál es la menor distancia al hormiguero a la que puede estar la hormiga después de haber recorrido 100 tramos? SOLUCIÓN La trayectoria más óptima para el enunciado del problema es hacer los dos primeros tramos alejándose, inevitablemente, del hormiguero para, después, ir acercándose a él con una trayectoria ‘en escalera’, como se aprecia en la figura. Llamamos a la distancia pedida en centímetros, hipotenusa del triángulo rectángulo cuyos catetos tienen, de medidas, y centímetros = − − − + … =⏞ − − − − … − = − − − − … − ⇒ ⇒ = − × !1 + + # $ + … + # $ % &, y la expresión encerrada entre paréntesis es la suma de los primeros 49 términos de una progresión geométrica cuyo primer término es 1 y la razón es Por lo tanto, = − × #) × * + *+ $ = − × # , $ + , + = − ×! +# , $ & - ≅ − - ⇒ ≅ - cm, pues 1 − # $ / ≅ 1 = − % − - − % − … =⏞ − − 0 − 1 − … − = − % × !1 + + # $ + … + # $ % & De manera similar al caso anterior, tenemos que = − % × #) × * + *+ $ = × 2 − × #) × * + *+ $3 ⇒ ⇒ = ⇒ ≅ - cm Aplicando ahora el teorema de Pitágoras en el triángulo rectángulo con las aproximaciones propuestas, = + = + # $ = ≅ × # - $ = × / × = / × ⇒ ≅ √ - × = √5x/3 cm = 0,745x cm
  • 70. En un cajón hay calcetines de ocho colores y ocho de cada color. Si sacamos dos calcetines al azar del cajón, ¿cuál es la probabilidad de que sean del mismo color? SOLUCIÓN La probabilidad de sacar dos calcetines del mismo color será la probabilidad de sacar un segundo calcetín del mismo color que el extraído en primer lugar. Al ir a sacar el segundo quedan 63 calcetines en el cajón, de los cuales hay 7 con el mismo color del ya extraído, por lo que la probabilidad es = = 1/9 = 11,11 %
  • 71. Sea ABCD un paralelogramo tal que el lado BC mide 13 cm, la altura correspondiente a la base AB mide 12 cm y el ángulo ABC es agudo. Sea E un punto en la prolongación de BC tal que el ángulo DEC = 90o Sabiendo que CE = 5 cm, calcula el área del cuadrilátero ABED. SOLUCIÓN Al construir la figura se puede observar que el cuadrilátero es un trapecio rectángulo. Sus bases miden = = 13 cm y = + = 13 + 5 = 18 cm y su altura es = cm Podemos también observar que los triángulos y son semejantes por ser rectángulos y, además, los ángulos respectivos y son iguales Por lo tanto, = ⇒ = ⇒ = cm Aplicando el teorema de Pitágoras en el triángulo tenemos que = − = 13 − 12 = 25 ⇒ ⇒ = 5 ⇒ = = 12 cm El área del trapecio es × = ! " × 12 = 31 × 6 = 186 cm2
  • 72. Si el cociente entre el radio del sector circular y el radio del círculo inscrito es 3, ¿cuál es el cociente entre sus áreas? SOLUCIÓN Construimos el esquema nombrando vértices y centros de sector y círculo inscrito. Llamamos al radio del sector y al radio del círculo: = 3, y sea 2 el ángulo que abarca el sector circular. Se observa que hay dos triángulos rectángulos iguales, y , en los que se cumple que = = = = ⇒ = ⇒ = 30° El sector circular abarca entonces 2 = 60°, sexta parte del círculo al que pertenece por lo que su superficie será La proporción que pide el enunciado es = × " # = × 3 = 3/2
  • 73. ¿Cuántos números de 15 dígitos que utilizan exclusivamente los dígitos 3 y 8 son múltiplos de 11? SOLUCIÓN Un número es múltiplo de 11 si la diferencia de las sumas de los dígitos que ocupan lugares pares y la suma de los dígitos que ocupan lugares impares es 0 o múltiplo de 11 Si suponemos que hay cifras 8 ocupando lugares impares e cifras 8 ocupando lugares pares en el número de 15 cifras, deberá cumplirse que 8 + 3 × 8 − − 8 + 3 × 7 − = 11 , siendo ∈ 8 + 3 × 8 − − 8 + 3 × 7 − = 11 ⇒ 5 + 24 − 5 − 21 = 11 ⇒ 5 − 5 = 11 − 3 ⇒ ⇒ 5 − 5 = 11 − 3 ⇒ − = ⇒ − = 2 + , siendo , ∈ ∋ 0 ≤ ≤ 8 y 0 ≤ ≤ 7 A partir de la igualdad anterior, y con las restricciones indicadas, los únicos valores factibles son: 1. = −2 ⇒ − = −5 • = 2, = 7: 2 cifras 8 y 6 cifras 3 ocupando lugares impares y 7 cifras 8 ocupando lugares pares. La cantidad de números de 15 cifras es !" #,$ × 1 = "! #!×$! × 1 = "×& # = 28 números • = 1, = 6: 1 cifra 8 y 7 cifras 3 ocupando lugares impares y 6 cifras 8 y 1 cifra 3 ocupando lugares pares. La cantidad de números de 15 cifras es !" ,& × !& $, = "! !×&! × &! $!× ! = 8 × 7 = 56 números • = 0, = 5: 8 cifras 3 ocupando lugares impares y 5 cifras 8 y 2 cifras 3 ocupando lugares pares. La cantidad de números de 15 cifras es 1 × !& ,# = 1 × &! !×#! = &×$ # = 21 números 2. = 3 ⇒ − = 6 • = 8, = 2: 8 cifras 8 ocupando lugares impares y 2 cifras 8 y 5 cifras 3 ocupando lugares pares. La cantidad de números de 15 cifras es 1 × !& #, = 1 × &! #!× ! = &×$ # = 21 números • = 7, = 1: 7 cifras 8 y 1 cifra 3 ocupando lugares impares y 1 cifra 8 y 6 cifra 3 ocupando lugares pares. La cantidad de números de 15 cifras es !" &, × !& ,$ = "! &!× ! × &! !×$! = 8 × 7 = 56 números • = 6, = 0: 6 cifras 8 y 2 cifras 3 ocupando lugares impares y 7 cifras 3 ocupando lugares pares. La cantidad de números de 15 cifras es !" $,# × 1 = "! $!×#! × 1 = "×& # = 28 números El total es 28 + 56 + 21 + 21 + 56 + 28 = 210 números
  • 74. Tengo escritos diez números naturales consecutivos y borro uno de ellos. Si la suma de los restantes es 2017, ¿qué número he borrado? SOLUCIÓN Sean desde a + 9 los diez números naturales consecutivos que tengo y borro el número + , tal que 0 ≤ ≤ 9 y ∈ Entonces, + + 1 + ⋯ + + 9 − + = 2017 ⇒ × − − = 2017, aplicando fórmula de la suma de los términos de una progresión geométrica. × − + = 2017 ⇒ 10 + 45 − − = 2017 ⇒ 9 − = 1972 ⇒ 9 = 1972 − ⇒ ⇒ = = 219 + ∈ . Como 0 ≤ ≤ 9 y ∈ ⇒ = 8 ⇒ = 219 + = 220, y el número borrado es + = 220 + 8 = 228
  • 75. El desarrollo decimal de la fracción tiene un período muy largo. Halla las tres últimas cifras del período. SOLUCIÓN Suponemos = 0, … y hacemos 10 × = 10 × 0, … = … × 0, … ⇒ 999. . .999999 ! × = … ⇒ 999. . .999999 ! = 97 × … Como el producto debe acabar en 9 ⇒ = 7, última cifra del período. Como 97 × 7 = 679 y el producto debe acabar en 99 ⇒ $ = 6, penúltima cifra del período. Como 97 × 67 = 6499 y el producto debe acabar en 999 ⇒ $ = 5, antepenúltima cifra del período. Así, 97 × 567 = 54999 Concluyendo, el período de la fracción acaba en 567
  • 76. Si a, b y c son números positivos tales que b = a30 y c = a42 , ¿cuánto vale logb c ? SOLUCIÓN Como = ó = = 7/5
  • 77. Dado un conjunto de diez números enteros positivos no necesariamente distintos, se realizan las siguientes diez operaciones: se descarta el primero y se suman los nueve restantes, se descarta el segundo y se suman los nueve restantes y se sigue así hasta descartar el último y sumar los nueve restantes. De esta manera se obtienen sólo nueve resultados distintos, que son 86, 87, 88, 89, 90, 91, 92, 93, 96. Halla los diez números iniciales. SOLUCIÓN Hacemos las diez operaciones citadas y, si son , , … los números buscados, se cumple que 9 × + + ⋯ + = 86 + 87 + 88 + 89 + 90 + 91 + 92 + 93 + 96 + = 812 + tal que = 9 + 7 (para que la suma sea divisible por 9) y, además, debe ser uno de los valores indicados como resultados obtenidos. La única posibilidad es que = 88 = 9 × 9 + 7 y, así, 9 × + + ⋯ + = 86 + 87 + 88 + 89 + 90 + 91 + 92 + 93 + 96 + = 812 + 88 = 900 y = + + ⋯ + = 100 De esta manera, los números son 100 − 86 = 14, 100 − 87 = 13, 100 − 88 = 12 (dos veces), 100 − 89 = 11, 100 − 90 = 10, 100 − 91 = 9, 100 − 92 = 8, 100 − 93 = 7 y 100 − 96 = 4 Resumiendo, 14, 13, 12, 12, 11, 10, 9, 8, 7 y 4
  • 78. Sean a y b dos números naturales. Si a+b y a3 +b3 acaban en 3, ¿en cuánto termina a2 +b2 ? SOLUCIÓN Como + = + 3 + 3 + = + + 3 × + y + = + + 3 × + ⇒ 3 × + acaba en 4 ⇒ acaba en 6 En resumen, + = + 2 + ⇒ + = + − 2 ! = + − 2" por lo que + acaba en 7
  • 79. Reemplazando x e y por dígitos, halla todos los números naturales de cinco cifras 65x1y que son múltiplos de 12. SOLUCIÓN Un número es divisible por 12 si lo es, a la vez, por 3 y por 4 Para que sea divisible por 3, la suma de sus cifras debe ser múltiplo de 3 ⇒ 6 + 5 + + 1 + = 3 ⇒ ⇒ 12 + + = 3 ⇒ + = 3 Para que sea divisible por 4, la dos últimas cifras deben formar un número múltiplo de 4⇒ 1 = 4 ⇒ = 2 = 6 Entonces, • = 2; + = 3 ⇒ = 1 = 4 = 7 • = 6; + = 3 ⇒ = 0 = 3 = 6 = 9 y los números son 65016, 65112, 65316, 65412, 65616, 65712, 65916
  • 80. En la figura, A y B son los puntos medios de los lados del hexágono regular. El hexágono interior, no regular, tiene los lados paralelos e iguales dos a dos. Si el área del hexágono regular es de 180 cm2 , ¿cuál es la superficie de la zona verde? SOLUCIÓN Dividimos el hexágono regular en triángulos iguales, como se indica en la imagen adjunta, viendo que el hexágono se divide en 24 triángulos de los cuales 14 están en la zona verde, por lo que la superficie de esa zona es × 180 = × 180 = 105 cm2
  • 81. Un barco navega entre dos orillas paralelas siguiendo el recorrido de la figura. Se conocen estas igualdades entre ángulos: ABC=CDE y CBD=CDB Si sale con un ángulo de 105o respecto de la primera orilla, ¿ qué ángulo sigue la trayectoria con la que atraca en la otra orilla? SOLUCIÓN Llamamos al ángulo solicitado y al ángulo determinado por la segunda igualdad. Según la imagen adjunta, el ángulo = 180° − 105° = 75° y, según el triángulo , 2 + 40° = 180° ⇒ 2 = 140° ⇒ = 70° Además, en el triángulo , = = 180° − − por lo que + + = 180° ⇒ 2 × 180° − − + 75° = 180° Como = 70° ⇒ 2 × 180° − − 70° + 75° = 180° ⇒ 220° − 2 + 75° = 180° ⇒ 2 = 115° ⇒ = ° = 57o 30’ o 180o -57o 30’=122o 30’
  • 82. Se inscribe en un hexágono regular otro cuyos vértices son los puntos medios de los lados del primero. Halla la proporción entre sus áreas. SOLUCIÓN Llamamos , a la apotema y el radio (y lado) del hexágono anterior, y , a la apotema y radio (y lado) del hexágono exterior. Es evidente que la apotema del exterior coincide con el radio del exterior. Si dividimos los hexágonos en seis triángulos equiláteros cada uno, de lados los radios respectivos, y tomamos uno de cada uno dividiéndolo por la mitad tenemos dos triángulos equiláteros. Uno de ellos de lados , , y el otro de lados , , Aplicando en ambos el teorema de Pitágoras, + = + = ⇒ = = ⇒ = √ = √ ⇒ = = √ La razón entre las superficies es, entonces, = = = 4/3
  • 83. Juan saca 2700 € en un banco. Le pide al cajero que le entregue cierta cantidad de billetes de 10 €, 20 veces esa cantidad de billetes de 20 € y el resto en billetes de 50 €. ¿Cuántos billetes de cada clase le entrega el cajero? SOLUCIÓN Llamamos al número de billetes de 10 € que solicita e al número de billetes de 50 € que pide. Entonces, 10 + 20 × 20 + 50 = 2700 ⇒ 410 + 50 = 2700 ⇒ 41 + 5 = 270 ⇒ ⇒ = 270 − 41 5 ⇒ = 54 − 8 − 5 Como , ∈ , los únicos valores admisibles en esta ecuación son = 5 ⇒ = 54 − 8 × 5 − = 13, por lo que los billetes entregados son 5 billetes de 10 €, 100 billetes de 20 € y 13 billetes de 50 €
  • 84. El área de un trapecio es de 1400 cm2 y su altura mide 50 cm. Si sus dos bases son valores naturales múltiplos de ocho, ¿cuántos posibles trapecios distintos verifican esos datos? SOLUCIÓN Llamando a las longitudes de bases 8 y 8 , siendo < y , ∈ ⇒ × 50 = 1400 ⇒ ⇒ 8 × + = × = 56 ⇒ + = = 7, de donde obtenemos las siguientes posibilidades: • = 1, = 6 • = 2, = 5 • = 3, = 4 En conclusión, los posibles trapecios distintos son 3
  • 85. Un automóvil viaja de A a C a una velocidad constante de 90 kilómetros por hora y, en el camino entre A y C, pasa por B. Cuando son las 8 de la mañana ha recorrido 1/4 de la distancia entre A y B y cuando son las 10 de la mañana ya ha recorrido 3/4 del camino entre B y C. Calcula la distancia entre A y C. SOLUCIÓN Sean , las distancias respectivas, en kilómetros, de A a B y de B a C. Es evidente que la distancia buscada, de A a C, es + Si hacemos un esquema del trayecto observamos que de las 8 horas a las 10 horas de la mañana el automóvil ha recorrido + : 2 horas a 90 k/h ∙ + = 90 × 2 = 180 ⇒ ⇒ 3 + 3 = 180 × 4 = 720 ⇒ + = = 240 kilómetros
  • 86. Si AB = AC y AE = AD, ¿cuánto vale el ángulo x ? SOLUCIÓN Sean , los ángulos iguales de los triángulos isósceles respectivos y En el triángulo , = 180° − 30° + ⇒ = 150° − En el triángulo , = 180° − 2 ⇒ = − 30° = 150° − 2 Por lo tanto, en el triángulo , 2 + 150° − 2 = 180° ⇒ 2 = 30° + 2 ⇒ = 15° + Para acabar, como + + = 180° ⇒ 150° − + 15° + + = 180° ⇒ 165° + = 180° ⇒ = 15o
  • 87. Sea ABCD un rectángulo con AB = 30 cm y BC = 16 cm. Si E y F son puntos, respectivamente, en los lados AB y CD tales que el cuadrilátero AFCE es un rombo, calcula la medida de EF. SOLUCIÓN Llamamos a la diferencia de longitudes del lado mayor del rectángulo y del lado del rombo. Entonces, el lado del rombo mide 30 − cm. Aplicamos el teorema de Pitágoras en el triángulo rectángulo : + = ⇒ + 16 = 30 − ⇒ ⇒ + 256 = 900 − 60 + ⇒ 60 = 644 ⇒ = = cm Aplicando ahora el teorema de Pitágoras en el triángulo rectángulo , = + ⇒ ⇒ = 30 − 2 + 16 ⇒ = 900 − 120 + 4 + 256 ⇒ = 4 − 120 + 1156 ⇒ ⇒ = 4 × − 30 + 289 ⇒ = 4 × 161 15 − 30 × 161 15 + 289 = 4 × 18496 225 ⇒ = 2 × 136 15 = 272/15 = 18,13 cm
  • 88. ¿Cuántos triángulos escalenos hay cuyo perímetro sea menor que 13 y que tengan las medidas de todos sus lados expresadas con números naturales? SOLUCIÓN Teniendo en cuenta que, en cualquier triángulo, la longitud de un lado debe ser menor que la suma de las longitudes, las posibilidades son: • Perímetro: 12 – máxima longitud de un lado: 5 o Lados 2, 5, 5 (isósceles); 3, 4, 5 (escaleno); 4, 4, 4 (equilátero) • Perímetro: 11 – máxima longitud de un lado: 5 o Lados 1, 5, 5 (isósceles); 2, 4, 5 (escaleno); 3, 3, 5 (isósceles); 3, 4, 4 (isósceles) • Perímetro: 10 – máxima longitud de un lado: 4 o Lados 2, 4, 4 (isósceles); 3, 3, 4 (isósceles) • Perímetro: 9 – máxima longitud de un lado: 4 o Lados 1, 4, 4 (isósceles); 2, 3, 4 (escaleno); 3, 3, 3 (equilátero) • Perímetro: 8 – máxima longitud de un lado: 3 o Lados 2, 3, 3 (isósceles) • Perímetro: 7 – máxima longitud de un lado: 3 o Lados 1, 3, 3 (isósceles); 2, 2, 3 (isósceles) • Perímetro: 6 – máxima longitud de un lado: 2 o Lados 2, 2, 2 (equilátero) • Perímetro: 5 – máxima longitud de un lado: 2 o Lados 1, 2, 2 (isósceles) • Perímetro: 4 – máxima longitud de un lado: 1 o No puede haber triángulos • Perímetro: 3 – máxima longitud de un lado: 1 o Lados 1, 1, 1 (equilátero) Hay, con las condiciones fijadas por el enunciado, 3 triángulos escalenos
  • 89. Halla todos los números de cuatro cifras abcd tales que ab + cd = bc y b – c = d SOLUCIÓN + = ⇒ 10 + + 10 + = 10 + ⇒ 10 + = 9 − 9 10 + = 9 ⇒ 10 = 8 ⇒ ⇒ 5 = 4 ⇒ = 4 = 5 Además, − = 5 ⇒ = + 5 Por tanto, los números posibles son, = 4; = 5; = 0; = 5 ⇒ = 4; = 5; = 1; = 6 ⇒ = 4; = 5; = 2; = 7 ⇒ #$ = 4; = 5; = 3; = 8 ⇒ &' = 4; = 5; = 4; = 9 ⇒ ( El resultado son los números 4505; 4615; 4725; 4835; 4945
  • 90. Halla el mayor número natural n para el que 5n es divisor del número SOLUCIÓN = 98! + 99! + 100! = 1 + 99 + 99 × 100 × 98! = 10000 × 98! = 5 × 2 × 98! Ahora bien, 98! contiene, como factores, a todos los múltiplos de 5 menores de 100: 98! = 1 × … × 5 × … × 10 × … × 15 × … × 20 × … … … … … × 90 × … × 95 × 96 × 97 × 98 Como hay 10 números acabados en 5 y dos de ellos tienen, en su factorización, dos 5 25 75 , todos esos números multiplicados tendrán 5 como factor con la mayor potencia de 5. Hay 9 números acabados en 0 que tienen un 5 en su factorización excepto uno 50 que tiene dos 5. Entonces, todos esos números multiplicados tendrán 5 como factor con la mayor potencia de 5. Por lo tanto 98! = 5 × 5 × =⏞ ! "#$ % $ & ∗ 5 × , siendo ∈ no divisible por 5 De ahí = 98! + 99! + 100! = 5 × 2 × 98! = 5 × 2 × 5 × =⏞ ∗ 5 ) × 2 × , por lo que * = 26
  • 91. Una hoja de papel rectangular se divide mediante un solo corte en un triángulo y un pentágono. Las longitudes de los lados del pentágono son 17, 25, 28, 33 y 43, en algún orden. Calcula el área del pentágono. SOLUCIÓN Representamos el corte con la hoja horizontal y señalamos las cinco longitudes como , , , , no necesariamente en el mismo orden de los valores expuestos 17, 25, 28, 33, 43 pero, lógicamente, = 43 Según la imagen, debe cumplirse que > > − + − = Teniendo en cuenta que , deben ser valores grandes de entre los propuestos y , valores pequeños (y no puede ser nunca 25 al ver las diferencias que quedan restantes), se pueden cribar posibilidades hasta dar con la válida: haciendo = 28, = 33, = 25, = 17 se cumplen las tres condiciones 43 > 28 33 > 25 43 − 28 + 33 − 25 = 17 y la superficie del pentágono es igual a la del rectángulo menos la del triángulo rectángulo: Sup á = a× c − a− b × c − d 2 = 43 × 33 − 43 − 28 × 33 − 25 2 = 1419 − 15 × 8 2 = 1359 unidades cuadradas
  • 92. Se elige al azar un número de cuatro cifras. ¿Cuál es la probabilidad de que sea impar y con todas sus cifras diferentes? SOLUCIÓN Números de cuatro cifras hay (de 1000 a 9999) 9999 − 1000 + 1 = 9000 Veamos los números de cuatro cifras que pueden crearse con todas sus cifras diferentes: la cifra de los miles debe ser una de las nueve cifras no nulas. Por cada una de las cifras elegidas quedan nueve restantes (ahora se incluye el 0) para las centenas. Para las decenas quedan ocho y para las unidades siete. Por tanto, la cantidad de números de cuatro cifras distintas que pueden formarse son 9 × 9 × 8 × 7 De ellas acaban en 1, 3, 5, 7, 9 la cantidad de 8 × 8 × 7 números en cada caso por las mismas razones que en el cálculo anterior y habiendo prefijado la cifra de las unidades. El total de números impares de cuatro cifras distintas es, entonces, 5 × 8 × 8 × 7, por lo que la probabilidad pedida es = × × × = × × × × × = × × = 56/225 ≈ 0,2489
  • 93. Halla cuantos números naturales n menores que 1000 hay tales que n2 termina en SOLUCIÓN Si acaba en 44 ⇒ acaba en 2 o en 8 • Si = 100 + 10 + 2, siendo entero no negativo y ∈ ∋ 0 ≤ ≤ 9, = 100 + 10 + 2 × 100 + 10 + 2 = ⋯ + 100 + 20 + 20 + 4 = ⋯ + 40 + 4 ⇒ 40 + 4 = ⋯ 44 ⇒ = 1 ⇒ los números pedidos son los que acaban en 12 = 6 ⇒ los números pedidos son los que acaban en 62 ) • Si = 100 + 10 + 8, siendo entero no negativo y ∈ ∋ 0 ≤ ≤ 9, = 100 + 10 + 8 × 100 + 10 + 8 = ⋯ + 100 + 80 + 80 + 64 = ⋯ + 160 + 64 ⇒ 160 + 64 = ⋯ 44 ⇒ = 3 ⇒ los números pedidos son los que acaban en 38 = 8 ⇒ los números pedidos son los que acaban en 88 ) De todo lo anterior, • acaban en 12 menores de 1000 ⇒ 12, 112, 212, … , 912: 10 números • acaban en 62 menores de 1000 ⇒ 62, 162, 262, … , 962: 10 números • acaban en 38 menores de 1000 ⇒ 38, 138 238, … , 938: 10 números • acaban en 88 menores de 1000 ⇒ 88, 188, 288, … , 988: 10 números En total, los números son 40 números
  • 94. Sean a, b, c tres números reales no nulos tales que a + b + c = 0 Calcula todos los posibles valores de la expresión SOLUCIÓN Como + + = 0, uno o dos números de los tres son negativos y los restantes (o el restante) positivos. Además, = | | + | | + | | + ∙ ∙ | | ∙ | | ∙ | | = ∙ | | ∙ | | + | | ∙ ∙ | | + | | ∙ | | ∙ + ∙ ∙ | | ∙ | | ∙ | | y los cuatro sumandos del numerador son iguales en valor absoluto. Estudiemos los dos casos posibles y fijémonos en los sumandos del numerador: 1. , positivos y negativo ⇒ ∙| |∙| | | |∙ ∙| | | |∙| |∙ ∙ ∙ ⇒ = ∙| |∙| | | |∙ ∙| | | |∙| |∙ ∙ ∙ | |∙| |∙| | =⏞ | |∙| |∙| | = 0 2. positivo y , negativos ⇒ ∙| |∙| | | |∙ ∙| | | |∙| |∙ ∙ ∙ ⇒ = ∙| |∙| | | |∙ ∙| | | |∙| |∙ ∙ ∙ | |∙| |∙| | =⏞ | |∙| |∙| | = 0 Por tanto, en todos los casos, = 0
  • 95. Sean a y b números reales distintos tales que 2a2 + 2b2 = 5ab. Halla todos los posibles valores de SOLUCIÓN 2 + 2 = 5 ⇒ 2 + 2 − 4 = 5 − 4 ⇒ 2 × + − 2 = ⇒ − = 2 2 + 2 = 5 ⇒ 2 + 2 + 4 = 5 + 4 ⇒ 2 × + + 2 = 9 ⇒ + = 9 2 Entonces, + − = + − = 9 2 2 = 9 ⇒ + − = ±3
  • 96. En un triángulo rectángulo de catetos 5 y 12 se inscribe una semicircunferencia como se muestra en la figura. ¿Cuál es su radio? SOLUCIÓN Nombramos todos los puntos y líneas interesantes y llamamos al radio pedido. Evidentemente, = = 5; = = ; = − = 12 − Como el triángulo es rectángulo, por el teorema de Pitágoras cumple que = + = 12 + 5 = 144 + 25 = 169 ⇒ = 13 Entonces = − = 13 − 5 = 8 De todo lo anterior, aplicando también el teorema de Pitágoras en el triángulo rectángulo obtenemos que = + ⇒ 12 − = + 8 ⇒ 144 − 24 + = + 64 ⇒ 24 = 144 − 64 = 80 ⇒ = = 10/3 unidades
  • 97. ABC es un triángulo tal que A > 90o y M es el punto medio del lado BC. Si BAM = 90o , AB = 35 y AC = 77, calcula BC. SOLUCIÓN Señalamos lo que nos indica el enunciado y, además, hacemos = 2 , valor buscado. Prolongando el lado y trazando su perpendicular por el vértice , llamamos al vértice intersección de ambas rectas. Observamos que los triángulos y son rectángulos y semejantes, por lo que = = = = 2 ⇒ = 2 × = 2 × 35 = 70 ⇒ = 35 Por lo tanto, aplicando el teorema de Pitágoras en el triángulo rectángulo , = − ⇒ ⇒ = 77 − 35 = 4704 Aplicamos ahora el teorema de Pitágoras en el triángulo : = + = 4704 + 70 = 9604 ⇒ ⇒ = √9604 = 98 unidades
  • 98. Si m y n son números impares tales que m > n, determina el mayor número natural que divide a todos los números de la forma SOLUCIÓN Como y son impares, = 2 + 1 y = 2 + 1, siendo > naturales. Entonces, − = + × − = 2 + 2 + 2 × 2 − 2 = 4 × + + 1 × − Evidentemente, + + 1 y − tienen distinta paridad por lo que uno de ellos (y solo uno) es par y de ahí se deduce que todo número del tipo − es divisible por 8
  • 99. Halla todos los números naturales x, y, z tales que SOLUCIÓN = 5 + ⇒ = − 5 = ⇒ + = y < 1 al ser , ∈ La única posibilidad es, entonces, que 9 + = ⇒ = 9 y = ⇒ + = 2 ⇒ = = 1 x = 9, y = 1, z = 1
  • 100. Sea la sucesión ¿Cuál es el menor número natural n tal que el producto de los n primeros términos de la sucesión es mayor que 100000? SOLUCIÓN Buscamos el menor número natural tal que 10 × 10 × … … × 10 > 10 = 100000 ⇒ 10 ⋯ > 10 ⇒ 1 13 + 2 13 + ⋯ 13 > 5 Por tanto, ⋯ > 5 ⇒ 1 + 2 + ⋯ + > 65 é" # $ %"$&" #ó "# é #( )********************+ × > 65 ⇒ 1 + × > 130 Si = 10 ⇒ 1 + × = 11 × 10 = 110 < 130 y si = 11 ⇒ 1 + × = 12 × 11 = 132 > 130, por lo que el valor pedido es = 11
  • 101. Sea ABCD un paralelogramo de lados AB, BC, CD y DA. Se traza por D una recta que corta al lado BC en P y a la prolongación del lado AB en Q. Si el área del cuadrilátero ABDP es 29 y el área del triángulo DPC es 8, halla el área del triángulo CPQ. SOLUCIÓN Dibujamos todos los elementos que nos da el enunciado y llamamos , a los lados del paralelogramo; = ; base de los triángulos y ; es la altura del paralelogramo y del triángulo , y es la altura del triángulo . Está claro que la superficie del paralelogramo es = 29 + 8 = 37 Además, la diferencia entre las superficies de los triángulos semejantes y es × − = 29 ⇒ + − = 58 ⇒ 37 + − = 58 ⇒ − = 21 Finalmente, la superficie del triángulo es la diferencia de las superficies de los triángulos y : "#$%&' = 2 − 2 = − 2 = 21/2 unidades cuadradas
  • 102. Si a, b, c son tres números racionales tales que ab = 26, ac = 128, bc = 52, calcula SOLUCIÓN = 26 = 2 × 13 = 128 = 2 = 52 = 2 × 13 ⇒ × × = = 2 × 13 ⇒ = 2 × 13 Entonces, + + = + + = × × + × + × × = 2 + + 2 = 8 + + 16 = 109/4
  • 103. Un triángulo equilátero se divide en cuatro triangulitos equilateros iguales, quedando determinados 9 segmentos que son lados de triangulitos. Distribuye los números 1, 2, 3, 4, 5, 6, 7, 8, 9 en los lados de los triangulitos, sin repeticiones, de modo que la suma de los tres números correspondientes a cada triangulito sea siempre la misma. SOLUCIÓN La suma de los lados de los tres triangulitos exteriores es (no necesariamente en el mismo orden que los valores indicados, + + + + + + + ℎ + = 1 + 2 + 3 + 4 + 5 + 6 + 7 + 8 + 9 = 45 Entonces, los lados de cada triangulito suman = 15 Cuatro posibles resultados (hay más) son:
  • 104. Sean m y n dos números naturales tales que n > 1 y mn = 225 ×340 Halla SOLUCIÓN > 1 ⇒ = 25,40 = 5 ⇒ = 2 × 3 = 2 × 3 ⇒ = 2 × 3 = 32 × 6561 = 209952 Entonces, + = 209952 + 5 209957